Ευκλειδης β 93

Page 1


EMHNIKH ΜΑΘΗΜΑΤΙΚΗ ETAIPEIA Τεύχος 93 -Ιούλιος - Αύγουστος- Σεπτέμβριος 2014- Ευρώ: 3,50 www.hms.gr

e-mail: info@hms.gr,

ΜΑΘΗΜΑΤΙΚΟ ΠΕΡΙΟΔΙΚΟ ΓΙΑ ΤΟ ΛΥΚΕΙΟ

ΠΕΡΙΕΧΟΜΕΝΑ

Βοϊκό πρόβλημα, ..................................................................................................... Βραβείο Fields, ........................................................................................................ . Γενικά Θέματα

1

7

8

Γράμμα της Σύνταξης

Αγαπητοί μαθητές και συνάδελφοι, σας ευχόμαατε καλή σχολική Χρονιά,

Βιβλιοθήκη Αρyuρούπολη<;, ................................................................................... Μαθηματικέ<; Ολυμπιάδε-;,.................................................................................... Homo Mathematicus, .... . ...... . ...................................................................................

33

μεγαλ ύτερο

�����::�:�; �ρ;θ��i:::::::::::::::::::::::::::::::::::::::::::::::::::::::::::::::::::::::

��

Στέλιος Μιόγλου και ο αντιπρόεδρος Λευτέρης Ασβεατόπουλος.

Α 'τάξη

Άλγεβρα: Γεωμετρία:

Ασκήσει-;,.......................................................................................... Β 'τάξη

Τριγωνομετρικέ<; εξισώσει<;, ......... . ................................................. Γεωμετρία: Εyyράψιμα Τετράπλευρα, .........................•.................................... Κατεύθυνση: Διανύσματα, ....................................................................................

Άλγεβρα:

Συστήματα Γραμμικών Εξισώσεων, ....................................•...........

Γ Τάξη

Γενική Παιδεία: Συναρτήσει<;,Όρια,.................................................................... Κατεύθυνη: Μιγαδικοί αριθμοί, .....................................................•............... Γ ενικά Θέματα

Το Βήμα του Ευκλείδη, .............. . ............................................................................. Ευκλείδη<; Προτείνει,.............................................................................................. Επικαιρότητα, ....................................................... . .................................................

Τα μαθηματικά μα<; διασκεδάζουν,.......................................................................

9

40

46

49 50

53

Σας παραδίδουμε με χαρά το πρώτο τεύχος του Ευκλείδη Β. με το τμήμα

του

αποτελ ο ύ μενο

από

εργασίες

συναδέλφων του Παραρτήματος Ημαθίας. Τ ην επιμέλεια των εργασιών ανέλαβε ο πρόεδρος του πάραρτήματος Ημαθίας Στα επόμενα τεύχη θα δημοσιευθούν αταδιακά και οι υπόλοιπες εργασίες οπό το παράρτημα, που δεν δημοσιεύτηκαν σε αυτό το τεύχος, όπως εργασίες και από άλλα παραρτήματα,καθώς και εργασίες άλλων συναδέλφων. Σας περιμένουμε με χαρά ατη Βέροια ατσ Συνέδριο ατις 7, Β και 9 Νοεμβρίου 2014. Με ευχές γιο υγεία και πρόοδο

Ο πρόεδρος της Συντακτικής Επιτροπής: Γιώργος Τασσόπουλσς

57

Οι αντιπρόεδροι: Βαγγέλης Ευσταθίου, Γιάννης Κερασαρίδης

63

συνάδελφοι: Α. Λυκείου ΙΧρ. Λαζαρίδης. Γ. ΚατσούληςΙ Β' Λυκείου Καρκάνης, Σ. Λουρίδας! Γ Λυκείου IK. Βακαλόπουλος. Δ. Αργυράκης!

59

72

γ,Γ. Σ' αυτό το τεύχος την επιμέλεια της ύλης των τάξεων είχαν και οι

IB.

77

79

Εξf!.,φ..,.υ)!\.,2.�

Εικαατική σίίνθεση με τη δομή

του σίίμπαντος και φωτογραφία από την Ολυμπιάδα 2014

ΚάΘε Σάββατο γίνονται fΔi'}REAN μαθήματα,

στα γραφεία της Ε.Μ.Ε. 27 Σεπτεμβρίου 2014. Θυμίζουμε ότι: Ερωτήματα σχετικά με τα Θέματα Διαγωνισμών υποβάλλονται στην επιτροπή Διαγωνισμών της Ε.Μ.Ε. Η έγκαιρη πληρωμή της ' συν�Ρ,.�μ ής; � βοηθάει στην έκδοση του περιοδικού ΠΑΝmΙΣJΗΜIΟΥ 34 106 79ΑθΗΝΑ Τηλ.: 210 3617784- 210 3616532 Fox: 210 3641025

Εκδότης:

Διευθυντής:

Δημόκσς Γεώργιος Κρητικός Εμμανουήλ

Επιμέλεια Έκδοσης:

Ζώτος Βαγγέλης

Κωδικός ΕΛ.ΤΑ: 2054

ιSSN: 1105 - 8005

Αθσνσα6ποuλος Ιtώρyιος

Πρόεδρος:

Ταααόπουλος Γιώργος Αντιπρόεδροι:

Ευσταθίου Βαγγέλης Κεραααρίδης Γιάννης Γραμματέας:

Χριστόπουλος Παναγιώτης Μέλη: Αργυράκης Δημήτριος Λουρίδας Σωτήρης Στεφανής Παναγιώτης Ταπεινός Νικόλαος

ΜΕΓΑΛΟΣ ΧΟΡΗΓΟΣ ΔΙΑΓΩΝΙΣΜΩΝ Ε.Μ.Ε.

Ανδρουλσκάκης Νίκος Αντωνόπουλος Γεώργιος Αντωνόπουλος Νίκος Αργυράκης Δημήτριος Βακαλόπουλος Κώατας Γιιβράς Τάαος Ευσταθίου Βαγγέλης Ζσχορόπουλος Κων/νος Ζώτος Βσyyέλης ΚοκσβάςΑπ6ατολος Κολίκος Στομ6της Κομπούκος Κυριάκος Κανέλλας Χρήστος Κορκάνης Βασίλης Κοταούλης Γιώρyος Κεροσορίδης Γιάννης Κορδσμίτσης Σπύρος Κονόμηςίt.ρτι Κόντζιος Νίκος Κοτσιφάκης Γιώργος Κυριαζής Ιωάννης

Συντακτική Επιτροπή ΚυριακόπουλοςΑντώνης Κυρισκοπούλου Κων/νσ Κυβερνήτου Χρυστ. Λαζαρίδης Χρήατος Λόππας Λευτέρης Λουριδάς Γιάννης Λουρίδας Σωτήρης Μολοφέκσς Θανάαης Μσνι6της Ανδρέας Μουρογισννάκης Λεωνίδας Μενδριν6ς Γιάννης Μεταξάς Νικόλαος Μήλιος Γεώργιος Μπερσίμης Φρσγκίσκος Μπρίνος Πσνοyιώτης Μυλωνάς Δημήτρης Μώκος Χρήστος Πσνδής Χρήατος Σίσκου Μαρία Σσ'iτη Εύα Στσϊκος Κώστας Στάϊκος Πσναyιώτης

Στεφανής Παναγιώτης Στρατής Γιάννης Ταπεινός Νικόλαος Τοααόπουλος Γιώργος Τ ζελέπηςίt.λκης Τ ζιώτζιος Θονάαης

τριόντος Γεώργιος

Τασγκάρης Ανδρέας

Τσαγκάρης Κώστας Τσικολουδάκης Γιώργος Τσιούμσς Θσνάαης Τ υρλής Ιωάννης Φσνέληίt.ννυ Χσρσλομπάκης Ευατάθιος Χσρολομποπούλου Λίνο Χαραλάμπους Θάνος Χριατιάς Σπύρος Χριατ6πουλος Θανάσης Χριστόπουλος Παναγιώτης Ψύχος Βοyyέλης

Σχόλιο: Οι εργασίες για το περιοδικό στέλνονται και ηλεκτρονικά ατο e-mαii:'SteΊios@lims.gr. Τιμή Τεύχους: ευρώ 3,50 Τα διαφημιζόμενα βιβλία δε σημαίνει ότι προτείνονται από την Ε.Μ.Ε. Οι συνεργασίες, (τα άρθρα, οι προτεινόμενες ασκήσεις, οι λύσεις ασκήσεων κλπ.) πρέπει να στέλνονται έγκαιρα, στα γραφεία της Ε.Μ.Ε. με την ένδειξη "Για τον Ευκλείδη Β". Τα χειρόγραφα δεν επιστρέφονται. Όλα τα άρθρα υπόκεινται σε κρίση, αλλά την κύρια ευθύνη τη φέρει ο εισηγητής. Ετήσια συνδρομή ( 12,00 + 2,00 Ταχυδρομικά= ευρώ 14,00). Ετήσια συνδρομή για Σχολεία ευρώ 12,00

Το αντίτιμο για τα τεύχη που παραγγέλνονται στέλνεται: (1). Με απλή ταχυδρομική επιταγή σε διαταγή Ε.Μ.Ε. Ταχ. Γραφείο Αθήνα 54 Τ.Θ. 30044 (2). Στην ιστοσελίδα της Ε.Μ.Ε., όπου υπάρχει δυνατότητα τραπεζικής συναλλαγής με την τράπεζα EUROBANK (3). Πληρώνεται στα γραφεία της Ε.Μ.Ε. (4). Με αντικαταβολή, σε εταιρεία ταχυμεταφορών στο χώρο σας, κατά την παραλαβή.

Εκ1ύπωση: ROTOPRINΊ' (Α ΜΠΡΟΥΣΑΛΗ & ΣΙΑ ΕΕ). τηλ.: 2ΊΟ 6623778- 358 Υπεύθυνος ιυπογραφείου: Δ. Παπαδόπουλος


fo Βοεi 11ρόβΑ�1μα Ο Αρχuμήδης καu � Νiεόvωvα� Παναγιώτης Π. Χριστόπουλος

Π ρόσφατα με μια ομάδα του Νομισματικού Μουσείου, επισκεφτήκαμε τον Ιούνιο του 2014, την κάτω

Ιταλία και την Σικελία, η οποία είναι το μεγαλύτερο νησί της Μεσογείου. Επισκεφτήκαμε πόλεις που ί­

δρυσαν τον 8° π.Χ. αιώνα οι Έλληνες, όπως οι Συρακούσες, ο Σελινούντας, η Ιμέρα, η Μεσσήνα, η Γέλα, ο Ακράγας, η Νάξος, η Εγέστη, το Ιππώνιο, ο Τάραντας που μαζί με τις πόλεις της κάτω Ιταλίας αποτε­

λούσαν τη «Μεγάλη Ελλάδα)). Περάσαμε τα στενά της Σκύλας και της Χάρυβδης, τα νησιά του Αιόλου (γνωστά από τη μυθολογία). Επισκεφτήκαμε το Παλέρμο που είναι η πρωτεύουσα και άλλες πανέμορφες πόλεις με ιστορία όπως την Ταορμίνα,

Λέτσε, Ρήγιο, Ελληνόφωνα χωριά, Μεταπόντιο, Σύβαρη. Στον Ακράγαντα θαυμά­

σαμε το ναό της Ομόνοιας, της Ήρας, του Δία, των Διόσκουρων, του Ηρακλή. Στις

Συρακούσες το Βωμό του Ιέρωνα και το αρχαίο θέατρο, το νησί της Ορτυγίας με το

ναό του Απόλλωνα το μακροβιότερο λειτουργικό ναό στον κόσμο, το ναό της Αθη-

νάς, το μουσείο Αρχιμήδη, την πηγή Αρεθούσα. Στη Σικελία κατοικούσαν Έλληνες ,Φοίνικες αλλά είχαν

βλέψεις και οι Καρχηδόνιοι. Νησί πλούσιο με πολλές ομορφιές που νιώθεις θαυμασμό για την ιστορία

του και ένα δέος το οποίο συμπληρώνεται με την θέα τις επιβλητικής Αίτνας. Η Σικελία επειδή έχει σχή­

μα τριγώνου, νησί με τρία ακρωτήρια (τρία άκρα), ονομάστηκε «Θρινάκρια ή Τρινάκριω> ή Θρινακία

κατά τον Όμηρο. Οι Συρακούσες είναι πόλη της Σικελίας, στο νοτιοδυ­

τικό άκρο κοντά στην Αίτνα. Ιδρύθηκε το 733 π. Χ. από τους Κορίνθιους.

Ήταν η σημαντικότερη πόλη της Μεγάλης Ελλάδας. Ήταν μία από τις ακμαιότερες, πλουσιότερες και δυνατότερες Ελληνικές πόλεις της Σικε­

λίας μέχρι την υποδούλωσή της το

212 π. Χ. από τους Ρωμαίους. Ό Κι­

κέρων την αποκάλεσε «Ωραιοτάτην και καταστόλιστον πόλιν». Στις Συρακούσες έζησε ο Αρχιμήδης τον οποίο σκότωσε στην εξέγερση τους

το

2 1 2 π. Χ., στρατιώτης των Ρωμαίων τη

στιγμή μάλιστα που ο Αρχιμήδης έλυνε κάποιο γεωμετρικό

πρόβλημα λέγοντάς το περίφημο «Μη μου τους κύκλους τάραττε)) .

Ο Αρχιμήδης [287 π.Χ.-212 π.Χ.] ήταν ένας από τους μεγαλύτερους μαθηματι­

κούς, φυσικούς και μηχανικούς της αρχαιότητας. Γεννήθηκε, έζησε και πέθανε στις Συρακούσες. Πατέρας του ήταν ο aστρονόμος Φειδίας που είχε συγγένεια με το βα­

σιλιά των Συρακουσών, Ιέρωνα Α'. Παρ' όλο που καταγόταν από ευγενική γενιά, ο Αρχιμήδης δεν πήρε οποιοδήποτε αξίωμα, επιμένοντας να διαθέτει όλο του τον χρό­

νο στη σπουδή και τη μάθηση. Γι' αυτόν τον λόγο ταξίδεψε στην Αίγυπτο και ήρθε σε επαφή με τους εκεί επιστήμονες, τον Ερατοσθένη, τον Κόνωνα κ. ά. Το έργο του Αρχιμήδη είναι τε­ ράστιο. Το σημερινό επίπεδο γνώσεων στην επιστήμη και την τεχνολογία έχει τις ρίζες στα έργα του Αρ­

χιμήδη. Παλαιότερα όπως έκλεβαν αγάλματα από την Ελλάδα για να στολίσουν τα μουσεία τους, έτσι

έκλεψαν και τις ιδέες του Αρχιμήδη και άλλων μεγάλων της Αρχαίας Ελλάδας για να προβληθούν ότι τα επινόησαν. Ο Αρχιμήδης συνέδεσε το όνομά του με τη γένεση της μηχανικής, τη λύση μαθηματικών

προβλημάτων, καθώς και με τις αμυντικές εφευρέσεις που χρησιμοποιήθηκαν όταν οι Ρωμαίοι πολιορ­

κούσαν την πατρίδα του. Έγραψε τα πρώτα βιβλία για την επίπεδη γεωμετρία και στερεομετρία, την αΕΥΚΛΕΙΔΗΣ Β' 93 τ.l/1


------

Το Βοεικό Πρόβλημα. Ο Αρχιμή δης και ο Νεύτωνας

--------­

ριθμητική και τα Μαθηματικά. Επίσης ανακάλυψε την άνωση, το ειδικό βάρος και το μοχλό. Είναι γνω­

στή η ιστορία με το χρυσό στέμμα του βασιλιά, τη φράση «Εύρηκα- Εύρηκω) καθώς και το «Δως μοι πα

στώ και τα Γ αν κινάσω>) (= Δώστε μου ένα σημείο να στηριχθώ και θα κινήσω τη Γη). Ο Αρχιμήδης

εmνόησε τη μέθοδο της εξάντλησης για τον υπολογισμό της περιοχής κάτω

από τόξο παραβολής, το άθροισμα άπειρης σειράς, μεγάλη προσέγγιση για

τον αριθμό π, ένα ευφυές σύστημα για την έκφραση πολύ μεγάλων αριθ­ μών και ακόμα μηχανές, οδομετρητές, παιχνίδια, ένα πλανητάριο με γρα­

νάζια, το ρωμαϊκό ζυγό (καντάρι), ρολόγια σαν το μηχανισμό των Αντικυ­ θήρων, το γρανάζι της Σαρδηνίας, κ.ά. Ο Κικέρων κάνει λόγο για μια ου­

ράνια σφαίρα και ένα πλανητάριο, που έφερε στη Ρώμη ο Μαρκέλλος μετά

την κατάληψη των Συρακουσών, τα οποία είχε κατασκευάσει ο Αρχιμήδης. Το έργο «Σφαιροποιία)) στο

οποίο περιγράφονταν οι αρχές κατασκευής τους χάθηκε. Άραβες επιστήμονες, αντέγραψαν όλα τα έργα

του στα αραβικά και έτσι διασώθηκαν αρκετά, αφού τα πρωτότυπα είχαν χαθεί. Το κύριο έγγραφο που πε­

ριέχει το έργο του Αρχιμήδη είναι το Παλίμψηστο του Αρχιμήδη.

Το 1 906, ο Δανός καθηγητής

Johan Ludνig Heiberg εmσκέφ­

θηκε την Κωνσταντινούπολη και εξέτασε περγαμηνή (από δέρμα

κατσίκας προερχόμενη από την Ιερουσαλήμ), γραμμένη τον 1 3 ο

Συρακούσες: Τα ερείmα του κέντρου της αρχαίας πόλης μνημείο παγκόσμιας πολιτιστικής κληρονομιάς της Unesco. Το αρχαίο θέατρο είναι το μεγαλύτερο πού χτίστηκε ποτέ από τούς aρχαίους Έλληνες.

μ.Χ. αιώνα, η οποία περιείχε 1 74 σελίδες. Τελικά ανακάλυψε ότι η περγαμηνή ήταν ένα παλίμψηστο,

δηλαδή ένα έγγραφο με κείμενο το οποίο είχε γραφεί πάνω σε μια σβησμένη παλιά εργασία. Τα πα­

λίμψηστα(πάλιν ψάω=πάλι ξύνω) δημιουργούνταν ξαίνοντας το μελάνι από τα αρχικά έργα των περγα­ μηνών και τα επαναχρησιμοποιούσαν. Αυτή ήταν μια συνηθισμένη πρακτική στον Μεσαίωνα, καθώς η

περγαμηνή ήταν αρκετά ακριβή. Τα έργα στο παλίμψηστο διαβάστηκαν με ειδικές μεθόδους. Οι πραγματείες του Αρχιμήδη στο παλίμψηστο είναι: Περί επιπέδων

ισορροmών, περί ελίκων, κύκλου μέτρησις, περί σφαίρας και κυλίνδρου, περί των εmπλεόντων σωμάτων, περί μηχανικών θεωρημάτων προς Ερατοσθένη, η έφοδος

και το Στομάχιον, στην αττική και δωρική διάλεκτο.Η περγαμηνή, πριν να πωλη­ θεί σε ιδιώτη συλλέκτη το 1 920, ήταν για εκατοντάδες χρόνια σε μια μοναστηρια­

κή βιβλιοθήκη στην Κωνσταντινούπολη. Στις 29 Οκτωβρίου 1998, πωλήθηκε σε δημοπρασία σε έναν ανώνυμο αγοραστή για 2 εκατομμύρια δολάρια από τον οίκο Κρίστις, στη Νέα

Υόρκη . Το παλίμψηστο φυλάσσεται στο Walters Art Museum της Βαλτιμόρης και είναι στη διάθεση της επιστήμης. Το 75 π. Χ., 1 37 χρόνια μετά το θάνατο του Αρχιμήδη, ο

Ρ ωμαίος ρήτορας Κικέρων αφού κανένας από τους ντόπιους δεν ήξερε τη θέση του τάφου του Αρχιμήδη έψαξε τον βρήκε ανάμεσα σε θά­

μνους διέταξε να τον καθαρίσουν και αναφέρει ότι επιστεφόταν από μια σφαίρα εγγεγραμμένη στο εσωτερικό ενός κυλίνδρου. Ο Αρχιμή­

1----·---+------·�·-···-··;.--�---·---_, . . . . ..

δης είχε αποδείξει ότι ο όγκος της σφαίρας είναι τα 2/3 του όγκου του κυλίνδρου και η επιφάνειά της τα 2/3 της επιφάνειας του κυλίνδρου,

αυτό θεωρείται ως το μεγαλύτερο των μαθηματικών εmτευγμάτων του.

ΕΥΚΛΕΙΔΗΣ

Β' 93 τ.l/2


-------

Το Βοεικό Πρόβλημα. Ο Αρχιμήδης και ο Νεύτωνας -------

Οι αρχαίοι Έλληνες είχαν κατορθώσει το ακατόρθωτο ήταν: φtλόσοφοι, ρήτορες, πολιτικοί, τεχνί­ τες, επιστήμονες υψηλού επιπέδου, αστρονόμοι, μαθηματικοί, φυσικοί, γιατροί, αρχιτέκτονες, έμποροι,

ναυπηγοί, γνώριζαν τις ιδιότητες υλικών, φυτών, ζώων και ανθρώπων. Έσκαψαν σήραγγες, έφτιαξαν μη­

χανές, aυτοματισμούς, πολύπλοκους μηχανισμούς με γρανάζια, βρήκαν τρόπους για τις μεταφορές, ταξί­ δεψαν μακριά από τα εθνικά τους σύνορα για να μάθουν τις γνώσεις άλλων λαών και πολιτισμών. Εκείνο

που θαυμάζει κανείς περισσότερο είναι με πόση εmδεξιότητα χρησιμοποιούσαν αυτή τη γνώση. �

Τα βόδια στους κάμπους των Συρακουσών . . . ο Αρχιμήδης και ο Νεύτωνας. �

Ας θαυμάσουμε τον Αρχιμήδη σε αυτό το πρόβλημα. Πόσα βόδια βόσκουν στα λιβά­ δια της Σικελίας; «Αν είσαι ώ ξένε μου σοφός κι ο νους σου κατεβάζει, σκέψου κα­ λά και μέτρησε πόσα είναι τα βοοειδή του αφέντη της γης του Ήλιου, που βό­ σκουνε σε τέσσερα κοπάδια μοιρασμένα στους κάμπους του όμορφου νησιού της Θρινακρίας (Σικελίας) και που έχει το καθένα τους αλλιώτικο το χρώμα. Το πρώτο λάμπει και είναι λευκό σα γάλα, το δεύτερο είναι μαύρο, το άλλο είναι όμορφο ξανθό ενώ το τέταρτο κοπάδι είναι παρδαλό.»

Αυτά γράφει ο Αρχιμήδης προς τους Αλεξανδρινούς μαθηματικούς, ύστερα συνεχίζει με τη διατύ­ πωση του προβλήματος και στο τέλος καταλήγει «Και τώρα, ξένε μου, αν βρεις πόσα είναι τα βόδια του Ήλιου, θα σε δεχτώ για μάστορη στων α­ ριθμών την τέχνη». Ο Αρχιμήδης συνήθιζε να στέλνει εmστολές από

τις Συρακούσες όπου ζούσε στον Ερατοσθένη και τον Κόνωνα για τους μα­

θηματικούς της Αλεξάνδρειας. Οι εmστολές περιείχαν τις εκφωνήσεις δικών

του θεωρημάτων χωρίς τις αποδείξεις τους. Ήθελε, πριν στείλει το νέο του κάθε φορά βιβλίο με τις αποδείξεις, να τους δίνει την ευκαιρία ώστε να τα δι­

ερευνούν. Σε μερικά θεωρήματα ο Αρχιμήδης δεν δίνει την σωστή απάντηση, γιατί κάποιοι μαθηματικοί στην Αλεξάνδρεια είχαν την κακή συνήθεια να τα παρουσιάζουν ως δικά τους. Έτσι φτιάχνει προβλήματα στα οποία βάζει διάφορες παγίδες και πονηριές. Ένα τέτοιο πρόβλημα που έφτιαξε με μεγάλη πονηριά και μαεστρία είναι το βοεικό πρόβλημα που με εmστολή του έστειλε στον Ερατοσθένη. Το 1769 ο Γερμανός Gotthold Ephraim Lessing (1729-1781) έφερε στο φως το πρό­ βλημα αυτό όταν ανακάλυψε ένα βυζαντινό χειρόγραφο του 14°uαιώνα το οποίο περιείχε 4 άγνωστα ποιήματα της Παλατινής Ανθολοyίας. Το χειρόγραφο αυτό βρίσκεται στη βιβλιοθήκη του Wolfenbuttel της βόρειας Γερμανίας. Σημαντικότερο από τα 4 ποιήματα είναι αυτό του Αρχιμήδη. Πρόκειται για πρό­ βλημα απροσδιορίστου αναλύσεως. Είναι γραμμένο στην Ιωνική διάλεκτο, με ελεγειακό μέτρο και απο­ τελείται από 22 δίστιχα. Σύμφωνα με την παράδοση, τα βόδια αυτά, τα βόδια του θεού Ήλιου, έβοσκαν στη Σικελική πόλη Ταορμίνα βόρεια των Συρακουσών. Το ποίημα αυτό έμεινε γνωστό στην Ιστορία ως «βοεικό πρόβλημα». Το πρόβλημα δεν έχει καμία δυσκολία για να καταστρώσει κανείς τις 7 εξισώσεις από τα δεδομένα που είναι: Να υπολογίσεις πόσοι είναι οι ταύροι και πόσες οι αγελάδες, σε κάθε ένα από τα τέσσερα κοπάδια των βοοειδών. I. 2. 3. 4.

Ο αριθμός τισν λευκών ταύρων ήταν, όσο το άθροισμα του 1/2 και του 1/3 των μαύρων ταύρων, συν το σύνολο των

ξανθών ταύρισν.

Ο αριθμός των μαύρων ταύρων ήταν όσο το άθροισμα του 1/4 και του 1/5 των ποικιλόχρωμων ταύρων, συν το σύνο­

λο των ξανθών τα\φων.

Το πλήθος στους ποικιλόχρωμους ταύρους ήταν όσο το άθροισμα του 1/6 και του 1/7 των λευκών ταύρων, συν το σύνολο των ξανθών ταύρων .

Ο αριθμός των λευκών αγελάδων ήταν τόσες, όσο το άθροισμα του 1/3 και του'!. του πλήθους του μαύρου κοπαδιού.

ΕΥΚΛΕΙΔΗΣ Β'

93 τ.l/3


-------

5.

Το Βοεικό Πρόβλημα. Ο Αρχιμήδης και ο Νεύτωνας -------

Οι μαύρες αγελάδες ήταν τόσες, όσο το άθροισμα του ι;. και του 1/5 του πλήθους του ποικιλόχρωμου κοπαδιού.

6.

Οι ποικιλόχρωμες αγελάδες ήταν τόσες, όσο το άθροισμα του 1/5 και του 1/6 του πλήθους του ξανθού κοπαδιού.

8.

Όταν συναθροίζαμε στους κάμπους της Θρινακίας ξανθούς και ποικιλόχρωμους ταύρους και τους στοιχίζαμε με τέ­

7.

Οι ξανθές αγελάδες, ήταν τόσες, όσο το άθροισμα του 1/6 και του 1/7 του πλήθους του λευκού κοπαδιού.

τοιο τρόπο ώστε, στην κορυφή να στέκι ένας, πιο πίσω βάζαμε σε σειρές τη μια δυο ταύρους, την άλλη τρεις, την ε­ πόμενη τέσσερεις και πάει λέγοντας, στο τέλος σχηματίζαμε ένα ισόπλευρο τρίγωνο. Όσο γιά τους λευκούς και για_ τους μαύρους ταύρους, όταν τους συναθροίζαμε, μπορούσαμε να τους διατάξουμε σε σειρές ίσες στον αριθμό στο μήκος και στο πλάτος.

Πού όμως είναι η δυσκολία; Αυτό που δεν αντιλαμβάνεται

ο aνυποψίαστος λύτης, είναι ότι ο Αρχιμήδης επέλεξε με τέτοια μαεστρία τα δεδομένα του προβλήματος ώστε η απάντηση να είναι πολύ μεγάλος αριθμός. Το πρόβλημα λύθηκε 23 αιώνες μετά, το 1965 που είχαμε τους πρώτους ισχυρούς ηλεκτρονι­ κούς υπολογιστές και έγινε εφικτή η καταγραφή των αριθμών. Πρόκειται για οκτώ αριθμούς με 200.000 ψηφία ο καθένας.

Η λύση: Το πρόβλημα ζητά να υπολογιστούν οι ταύροι και

οι αγελάδες από κάθε χρώμα: λευκό, μαύρο, ξανθό, ποικιλό-

χρωμο.

Αρχαίο θέατρο στην Ταορμίνα

Ας συμβολίσουμε το πλήθος με τα γράμματα:

Λ=(λευκό), Μ=(μαύρο), Ξ={ξανθό), Π=(ποικιλόχρωμο) για τους ταύρους και αντίστοιχα λ, μ, ξ, π για τις αγελάδες. Στην αρχή προκύπτουν γραμμικές σχέσεις μεταξύ των αγνώστων που οδηγούν σε ένα σύστη­ μα 7 εξισώσεων με 8 αγνώστους, αλλά στο δεύτερο μέρος του προβλήματος ζητά να προσδιοριστεί εκεί­

νη η λύση η οποία ικανοποιεί δύο ακόμα συνθήκες, να είναι το (Λ+Μ) τέλειο τετράγωνο και το (Ξ+Π)

να είναι τρίγωνος αριθμός:

Λ=(l/2 + 1 /3 )Μ+Ξ,

Μ=(1 /4+1 /5)Π+Ξ,

Π=(1 /6+1/7)Λ+Ξ,

λ=(1/3+1/4 )(Μ+ μ),

μ=(1 /4+1 /5)(Π+π), π=(1 /5+1 /6)(Ξ+ξ), ξ=(1 /6+1/7)(Λ+λ)

Η γενική λύ ση είναι:

Λ=2*7*53*4.657ν=10.366.482ν,

Μ=2*32*89*4.657ν=7.460.514ν,

Ξ=22*5*79*4.657ν=7.358.060ν, 0=34*1 1 *4.657ν=4.149.387ν, λ=23*3 *5*7*23 *373ν=7.206.360ν, μ=2*32*17*1 5.991 ν=4.893.246ν, ξ=22*3*5*7*1 1 *761ν=3.51 5.820ν, π=32*13*46.489ν=5.439.213ν

με

ν

συμβολίζεται η απροσδιόριστη παράμετρος που με την τιμή ν=80 ικανοποιούνται οι 7 γραμμικές ε­

ξισώσεις όχι όμως και το δεύτερο μέρος του προβλήματος που απαιτεί να είναι το άθροισμα (Λ+Μ) τέ­

λειο τετράγωνο και το άθροισμα (Ξ+Π) να είναι τρίγωνος αριθμός. Η τιμή της παραμέτρου ν που ικανο­ ποιεί την πρώτη από τις δυο συνθήκες είναι: ν=3 *11*29*4.657*λ2 όπου λ ακέραιος αριθμός. 2 Η δεύτερη συνθήκη μας οδηγεί σε μια εξίσωση Pell της μορφής χ - Dy =1, συγκεκριμένα την χ2- 410.286.423.278.424y =1 η οποία δεν είναι καθόλου εύκολο να λυθεί.

Το πλήθος των ταύρων και των αγελάδων κάθε χρώματος είναι πάρα πολύ μεyάλο και για λόγο αυτό εί­

ναι αδύνατο να τις προσδιορίσει κανείς χωρίς την χρήση ηλεκτρονικού υπολογιστή.

Η

λύση με τη μικρότερη

τιμή στο πλήθος βοδιών εκφράζεται με τον αριθμό 7766 ο οποίος ακολουθείται από 206541 μηδενικά. �

Το πρόβλημα με τα βόδια του Νεύτωνα Ισαάκ Νεύτων (1643-1727). Ο Νεύτωνας και ο Αρχιμήδης μπορούμε να πού­

με υπήρξαν οι μεγαλύτεροι επιστήμονες που από την αρχαιότητα μέχρι σήμερα, εί­ χε η ανθρωπότητα. Ο Νεύτωνας είχε διατυπώσει διάφορα προβλήματα και μάλιστα

κρατούσε σημειώσεις στα Ελληνικά. Ο Νεύτωνας μεταξύ των άλλων έθεσε και το

πρόβλημα σχετικά με την κίνηση των βλημάτων στον αέρα. Το 2012 ένας 16χρονος Ο Souήgia Rey, που γεννήθηκε στην Ινδία κατάφερε να λύσει το πρό-

ΕΥΚΛΕΙΔΗΣ Β' 93 τ.l/4


-------

Το Βοεικό Πρόβλημα. Αρχιμήδης και ο ΝεUτωνας Ο

--------­

βλημα και να υπολογίσει ακριβώς την πορεία ενός βλήματος κάτω από την επίδραση της βαρύτητας και της αντίστασης του αέρα, σε μια εpyασία του σε σχολείο της Γερμανίας όπου φοιτούσε. (έλυσε και ένα δεύτερο πρόβλημα που έχει να κάνει με την κρούση ενός σώματος σε τοίχο). Χιλιάδες σελίδες από τα ιστορικά χειρόγραφα του Νεύτωνα δια­ τηρεί το Πανεπιστήμιο του Κέιμπριτζ όπου δίδασκε και είναι προσβάσι­ μα στο Διαδίκτυο. Μεταξύ αυτών βρίσκεται η πρωτότυπη τυπωμένη έκ­ δοση του αριστουpyήματός του «Pήncipia Mathematica» (Μαθηματικές Αρχές της Φυσικής Φιλοσοφίας), μαζί με τις εμβόλιμες σχετικές χειρό­ γραφες σημειώσεις και απαντητικά σχόλια στους επικριτές του, που ο ί­ διος είχε κάνει πάνω στο δικό του αντίτυπο. Στο σημειωματάριο του Νεύτωνα και συγκεκριμένα στις σημειώσεις που κρατούσε όταν σπού­

Σημειωματάριο του Νεύτω­ να που κρατούσε σημειώ­

δαζε στο Tήnity College από το 1661 έως το 1665 βλέπει κανείς ότι ο

σεις στα Ελληνικά.

Ισαάκ Νεύτων έγραφε Ελληνικά. Ας δ ού μ ε όμω ς τ ο π ρόβλη μα

Σε 12 ημέρες 75 βόδια κατανάλωσαν όλο το χορτάρι που υπήρχε σε ένα λιβάδι 60 στρεμμάτων καθώς και το χορτάρι που φύτρωσε κατά τη διάρκεια αυτών των 12 ημερών. Σε 15 ημέρες 81 βόδια κατανάλωσαν όλο το χορτάρι που υπήρχε σε ένα λιβάδι 72 στρεμμάτων καθώς και το χορτάρι που φύτρωσε κατά τη διάρκεια αυτών των 15 ημερών. Πόσα βόδια θα καταναλώσουν σε 18 ημέρες το

χορτάρι που υπάρχει σε ένα λιβάδι 96 στρεμμάτων, καθώς και αυτό που θα φυτρώσει στη διάρκεια

των 18 ημερών; (το χορτάρι σε κάθε στρέμμα στα λιβάδια είναι το ίδιο, επίσης φυτρώνει ομοιό­ μορφα και όλα τα βόδια τρώνε την ίδια ποσότητα).

Λύ ση : Μια λύση στο πρόβλημα είναι: Έστω λ η ποσότητα του χορταριού που φυτρώνει κάθε ημέρα

σε ένα στρέμμα. Έχουμε ότι τα 75 βόδια τρώνε σε 12 ημέρες το χορτάρι 60 στρεμμάτων και το χορτάρι που φυτρώνει σε 12 ημέρες δηλαδή ποσότητα ίση με 12χ60λ=720λ. Άρα τα 75 βόδια σε 15 ημέρες (121

� ) τρώνε χορτάρι 75 (60 ι6:) στρεμμάτων και ακόμη αυτό που φυτρώνει 720λ ι :

12 λ

=900λ. Δηλαδή

το ένα βόδι τρώει σε 15 ημέρες χορτάρι ενός στρέμματος και 12λ ή 81 βόδια σε 15 ημέρες τρώνε χορτάρι 81 στρεμμάτων και 972λ(=81 χ 12λ) (1) Από την εκφώνηση του προβλήματος ξέρουμε ότι τα 81 βό­ δια τρώνε σε 15 ημέρες το χορτάρι 72 στρεμμάtων και το χορτάρι που φυτρώνει σε 15 ημέρες δηλαδή ποσότητα 15 χ72λ=1080λ (2) Από τις πιο πάνω σχέσεις (1) και (2) προκύπτει: ποσότητα χορταριού 81 στρεμμάτων και 972λ ισούται με ποσότητα χορτα­ ριού 72 στρεμμάτων και 1080λ, ή η ποσότητα χορταριού 9 στρεμμάτων ισούται με 108λ. Δηλαδή η αρχική ποσότητα Όταν θυσίαζαν στην Αρχαία Ολυμπία το αί­ μα των ζώων ανάβλυζε σε αυτή τη λίμνη

χορταριού σε ένα στρέμμα είναι ίση με 12λ (=το χορτάρι που φυτρώνει σε ένα στρέμμα σε 12 ημέρες). Επομένως τα 75 βόδια σε 12 ημέρες έφαγαν το χορτάρι 60+ 720λ= 60+ 60χ12λ=

60+ 60χ1=120 στρεμμάτων, σε 18 ημέρες έφαγαν

1

2° 1 2

χ18=180 στρεμμάτων.

Έτσι για να καταναλωθεί σε 18 ημέρες το χορτάρι 96 στρεμμάτων και (18 χ 96λ=) 1728λ δηλαδή 96+ 144χ12λ=96+ 144χ1=240 στρεμμάτων θα βοσκήσουν

::ο χ240=100 βόδια.

ΕΥΚΛΕΙΔΗΣ Β' 93

τ.l/5


-------

Το Βοεικό Πρόβλημα. Ο Αρχιμήδης και ο Νεύτωνας

-------

2'1 λύ ση : Έστω Χ τα βόδια με ΧεΖ. Έστω Α η αρχική ποσότητα χόρτου ανά στρέμμα. Έστω Β η

αύξηση χόρτου ανά στρέμμα την ημέρα. Έστω Κ η ποσότητα χόρτου που τρώει ένα βόδι την ημέρα. Τότε 60.Α+ 60.Β.12=75.12.Κ ή Κ= Α+ 28 (1). Και 72Α+ 72Β.15=81.15Κ ή Κ= S(A

1 ' (1) και (2) εχουμε Α+ 128 = 8(Α+158) δη λαδη ' Α=12Β Απο 1S 7_5 2 96.Α+96.Β.18=Χ.18.Κ ή 96.12Β+96.Β.18=Χ.18. 128 128 οπότε X=lOO βόδια.

2��:8)

(2)

'

Σχόλιο: Μπορεί να λέμε το πρόβλημα του Νεύτωνα αλλά ίσως να μην κατασκευάστηκε από τον ί­ διο τον Νεύτωνα και να είναι της μαθηματικής παράδοσης. Μια άλλη εκδοχή του προβλήματος

'Εχουμε τρία λιβάδια, στο πρώτο που είναι 1 ° στρέμματα 3

βόσκουν 12 βόδια για 4 εβδομάδες, ενώ στο δεύτερο που είναι 1Ο στρέμματα βόσκουν 21 βόδια για 9 εβδομάδες. Πόσα βόδια είναι δυνατό να βόσκουν στο τρίτο λιβάδι που είναι 24 στρέμ­ ματα για 18 εβδομάδες; (τα τρία λιβάδια έχουν ίδιο χορτάρι σε κάθε στρέμμα, επίσης φυτρώνει με τον ίδιο ρυθμό κάθε μέρα και όλα τα βόδια τρώνε την

ίδια ποσότητα).

Ο Ηρακλής και τα βόδια του Γηρυόνη

:\Ιια ί.ύση στο π ρό βλημα είναι: Έστω Α η αρχική ποσότητα χόρτου ανά στρέμμα. Έστω Β η αύξη­

ση χόρτου ανά στρέμμα την εβδομάδα. Έστω Κ η ποσότητα χόρτου που τρώει ένα βόδι την εβδομάδα. Σύμφωνα με το πρόβλημα έχουμε το σύστημα: 0

10

1 3.Α+ 4.3.Β=4.12.Κ

10.Α+ 40.Β=144.Κ (1)

10.Α+9.10.Β=9.21.Κ

ή

24.Α+ 18.24.Β=18. Χ.Κ

54Κ

,

,

10.Α+90.Β=189.Κ (2) 4.Α+ 72.Β=3. Χ.Κ 9Κ

Απο (1) και (2) εχουμε Α=- και Β=ϊΟ 5 );;>

,

(3)

16Κ 648Κ ι 3ΚΧ ή 108Κ=3ΚΧ Άρα Χ=36 απο (3) 2 5

10

Ένα ακόμα πρόβλημα με βόδια από την Αρχαία Ελλάδα

Τ ρεί ς έμπ οροι

Τρείς έμποροι πούλησαν βόδια ο Α 20, ο Β 30, ο Γ 40 όλα της iδιας αξίας.

Μέχρι το μεσημέρι και οι τρείς πούλησαν στην iδια τιμή. Το απόγευμα το iδιο αλλά σε διαφο­ ρετική τιμή από την πρωινή. Οι τιμές είναι ακέραιοι θετικοί αριθμοί. Στο τέλος της ημέρας είχαν πουλήσει όλα τα βόδια και είχαν εισπράξει το iδιο ποσό. Πως t:yινε αυτό; Λύση:

Αν α, β, γ τα βόδια αντίστοιχα που πούλησαν οι έμποροι το πρωί με Χ Ευρώ το ένα, θα είναι 20-α,

30-β, 40-γ αυτά που πούλησαν το απόγευμα με Ψ Ευρώ το ένα. Εισέπραξαν αΧ+ (20-α)Ψ=βΧ+ (30β)Ψ=γΧ+ (40-γ)Ψ, 2 εξισώσεις με 5 αγνώστους.

Οι εξισώσεις γίνονται (Ψ-Χ)(β-α)=lΟΨ και (Χ-Ψ)(γ-β)=10Ψ από τις δύο αυτές εξισώσεις προκύπτει β-α=γ-β ή α+ γ=2β που επαληθεύεται από άπειρες τριάδες ακέραιων αριθμών. Αν πάρουμε α=1 τότε β­ <e11 δηλαδή έχουν διαφορά με γαλύτερη του 10. Αν πάρουμε β=12 έχουμε γ=23 με αντικατάσταση των τιμών στην εξίσωση έχουμε Ψ=11Χ και αν Χ=1 τότε Ψ=11. Με τις ίδιες τιμές για τα α, β, γ, και με ίδια πολλαπλάσια για τα Χ, Ψ έχουμε πάντα μια λύση. π.χ. α=1, β=12, γ=23, Χ=5, Ψ=55. Όμως για να είναι κοντά στην πραγματικότητα οι τιμές των Χ, Ψ και ακέραιοι αριθμοί, θα πάρουμε α=2, β=17, γ=32 οπότε Ψ=3Χ και για X=lOOO€, Ψ=3000€ και ο κάθε έμπορος εισέπραξε 56000€.

ΕΥΚΛΕΙΔΗΣ Β' 93 τ.l/6


Το μετάλλιο Fields 2014 Στο Intemational Congress of Mathematicians 2014, στη Σεούλ της Νότιας Κορέας, δόθηκε το «βραβείο Nobel» Μαθηματικών το Fields Medal 2014, για πρώτη φορά σε γυναίκα. Η 37χρονη Ιρα­ νή μαθηματικός Maryam Mirzakhani, είναι η πρώτη γυναίκα που κέρδισε το μετάλλιο μαζί με τρεις ακόμη επι­ στήμονες τους Artur Aνila Γαλ­ λο-Βραζιλιάνο ε­ ρευνητή στο ευρω­ παϊκό ινστιτούτου CERN του πανεπιστημίου Ντενί Ντιντερό του Παρισιού, Manjul Bargaνa καθηγη­ τή του Πανεπιστημίου Πρίνστον και τον Martin Hairer καθηγητή του Πανεπιστημίου Warwick της Βρετανίας. Η Ιρανή μαθηματικός είναι καθηγήτρια στο Πανεπιστήμιο Στάνφορντ της Καλιφόρνιας και βραβεύτηκε γα εργασία στη. γεωμετρία στη δομή των γεωδαιτικών γραμμών των παραμετρικών χώ­ ρων (moduli spaces). Το φως θεωρείται πως κινείται πάντα σε γεωδαιτικές γραμμές καθώς αποτελούν το πιο σύντομο δρόμο για να φτάσει από το σημείο Α στο σημείο Β. Περιέγραψε στην εργασία της, τους τρόπους με τους οποίους μπορεί να διαδοθεί το φως σε μία κλειστή διαδρομή εντός ενός Σύμπα­ ντος με δύο διαστάσεις, επινοώντας τρόπους πλοή­ γησης σε πολυδιάστατα σύμπαντα. Η εργασία του Α νila είναι στα χαοτικά συστήματα, του Bargaνa στη θεωρία των αριθμών και του Hairer στις δια­ φορικές εξισώσεις. Γεννήθηκε και μεγάλωσε στην Τεχεράνη, σπούδασε στο Πανεπιστήμιο Σαρίφ της ιρανικής πρωτεύουσας και στις Διεθνείς Μαθη­ ματικές Ολυμπιάδες του 1994 και 1995 τιμήθηκε με χρυσά μετάλλια. Για την επιτυχία της είπε πολλά για τα σχολεία, τα βιβλία, τους καθηγητές και ότι: «Δεν έχω κάποια ιδιαίτερη συνταγή για την πρωτότυπη μαθη­ ματική σκέψη. Η έ­ ρευνα στα Μαθηματι�ά φέρνει στο μυαλό έναν άνθρωπο χαμένο στη ζούγκλα. Πρέπει να αξιο­ ποιήσει όλες τις γνώσεις του για να επινοήσει νέα τεχνάσματα και με λίγη τύχη ίσως βρει κάποιον τρόπο να φτάσει στον πολιτισμό».

Το μετάλλιο Fields απονέμεται κάθε τέσσερα χρόνια, σε τέσσερις το πολύ μαθηματικούς ηλικίας μέχρι 40 ετών. Από το 1936 μέχρι σήμερα βραβεύ­ τηκαν 55 άνδρες και η Maryam Mirzakhani. Το 2006 ο Ρώσος μαθηματικός Perelman, που απέ­ δειξε την εικασία Poincare, αρνήθηκε να παραλά­ βει το βραβείο που συνοδευόταν από μεγάλο οικο­ νομικό ποσό. Το 2011 το αντίστοιχο μετάλλιο Sbaw πήρε ο Έλληνας μαθηματικός-φυσικός Δη­ μήτρης Χριστοδούλου. Η

Ο

κ. Δ. Χριστοδούλου

με μαθητές

Ελληνογαλλική σχολή Ουρσουλινών προ­ σκάλεσε το Νομπελίστα καθηγητή Φυσικής και Μαθηματικών στο Πολυτεχνείο της Ζυρίχης, κ. Δημήτρη

Χρι­

Ο κ. Χριστοδούλου, όταν δεν διδάσκει σε κάποιο ξένο πανεπιστήμιο και είναι στην Ελλάδα, ανταπο­ κρίνεται στις προσκλήσεις σχολείων και με το χάρισμα που έχει να κάνει απλά, τα δύ­ σκολα θέματα της επιστήμης, μιλά στα παιδιά. Την Πέμπτη 6 Φεβρουαρίου 2014 στο συγκε­ κριμένο σχολείο έγινε ομιλία για το έργο του. και την προσφορά του στην επιστήμη. Οι μα­ θητές ήταν προετοιμασμένοι και είχαν διακο­ σμήσει την αίθουσα διαλέξεων με φωτογρα­ φίες του και άλλα έντυπα σχετικά με το έργο του. Το κύριο επιστημονικό έργο του κ. Χρι­ στοδούλου είναι η Γενική Θεωρία της Σχε­ τικότητας και η κλασσική Φυσική του συ­ νεχούς (Μηχανική των ρευστών, ελαστικότη­ τα των στερεών και ηλεκτρομαγνητισμό συ­ νεχών μέσων). Από τα Μαθηματικά τα γραμ­ μικά συστήματα διαφορικών εξισώσεων με μερικές παραγώγους. Ο κ. Χριστοδούλου εξή­ γησε στα παιδιά, με τον χαρισματικό και ήρε­ μο λόγο που έχει, τα δύσκολα θέματα της επι­ στήμης. Ξεκινώντας από το Νεύτωνα και τον Αϊνστάιν κατέληξε στην καμπυλότητα του χωροχρόνου. Ο λόγος του συνεπήρε τους μα­ θητές και καθηγητές οι οποίοι, με εύστοχες ερωτήσεις. ζήτησαν να μάθουν περισσότερα από τα μυστήρια της φύσης. Ο κ. Χ. Μελισ­ σάρης από τη Διεύθυνση του σχολείου τον ευ­ χαρίστησε και τον συνεχάρη για αυτή την εκ­ δήλωση. στοδούλου.


Τα Μαθηματικά μέσα από τα μάτια της Εθνικής Βιβλιοθήκης Αργυρουπόλεως του Πόντου «0 Κυριακίδης»

Πουλτίδης Σταύρος

Η

Αργuρούπολη ήταν ένα από τα σημαντικότερα πνευματικά κέντρα του Πόντου και το Φροντιστή ριό της διατηρούσε μια μεyάλη και εξαιρετικά αξιόλογη βιβλιοθήκη. Δεν γνωρίζουμε ποιοι πνευματικοί άνθρωποι μετέφεραν εκεί τα πρώτα χειρόγραφα, αλλά φωτισμένοι Πόντιοι ασχολήθηκαν με τον εμπλουτισμό της, τη μόρφωση και την καλλιέργεια των μαθητών του Φροντιστηρίου. Ο Βησσαρίωνας από την Τραπεζούντα του Πόντου, ήθελε να μαζέψει ένα μεyάλο αριθμό βιβλίων γιατί πίστευε πως όταν θα έρθουν καλύτερες μέρες για τους Έλληνες, πρέπει να βρουν κάπου συγκεντρωμένη τη γλώσσα και τον πολιτισμό τους. Το ίδιο ακριβώς όραμα είχαν και οι Έλληνες της Αργuρούπολης και γι' αυτό τον λόγο, ιδρύεται το 1723 από τον Μητροπολίτη Χαλδίας Ιγνάτιο Α' Σκρίβα το Φροντιστήριο της Αργuρούπολης. Σύμφωνα με την καταγραφή του Γεώρyιου Κανδιλάπτη, ο αριθμός της συλλογής της Βιβλιοθήκης το 1903 ανήλθε σε 5.000 τόμους, εκ των οποίων ξεχώριζαν τα 166 χειρόγραφα. Ο ξεριζωμός όμως των Ελλήνων του Πόντου (1922) και η περιπετειώδης εγκατάστασή τους στη βιβλιοθήκη της Ευξείνου Λέσχης Ποντίων Νάουσας (1927), κόστισε στην ανθρωπότητα 4.028 βιβλία. Έτσι, το 1939 κωδικοποιήθηκαν από τον καθηγητή Παπυρολογίας και Παλαιογραφίας Αντώνιο Σιγάλα 972 βιβλία, ανάμεσά τους και 76 χειρόγραφα. Για τα ζωντανά αυτά υπολείμματα της ελληνικής παιδείας και του πολιτισμού της ανατολής, έχει επισημανθεί από ειδικούς. διαπρεπείς επιστήμονες ότι μερικά από αυτά είναι μοναδικά στον κόσμο και η επιστημονικά τους αξία είναι m'ΕΙC'ήμητη. Ο Πρόεδρος του Μαθηματικού Τμήματος του Αριστοτελείου Πανεπιστημίου θεσσαλονίκης κ. Αντωνίου ανέφερε χαραηηριστικά: «Αυτά που ξέρουμε σήμερα για την επιστήμη των Μαθηματικών κατά την διάρκεια της τουρκοκρατίας, φαi\τται ότι ιφέχει να διορθωθούν και να ξαναγραφτούν. Βλέπουμε δηλαδή τη Γεωμετρία του Legendre, Α.Μ. να είναι μεταφρασμb.η η 111 έκδοση το 1840 και η 2η το 1860. Αυτό σημαίνει ότι παρακολουθούσαν οι Πόντιοι αδελφοί μας κανονικά τις εuρωιωϊιcές εξελίξεις της επιστήμης. Αυτά πρέπει να διαβαστούν και να γίνουν οι απαραίτητες διορθώσεις>>. Ανάμεσα στα μοναδικά αυτά κειμήλια βρίσκονται Στοιχεία Μαθηματικών από το 1767, βιβλία δηλαδή τα οποία διδάσκονταν στο Γυμνάσιο και το Πανεπιστήμιο από το Νικηφόρο θεοτόκη το 1798 και το 1828., ενώ ο Ευγένιος ΒούλΎαρης το 1805 είχε γράψει Αστρονο μία την οποία και ονόμασε Σύστημα του Παντός. Σήμερα, οι πνευματικοί αυτοί φάροι της Ανατολής έχουν ψηφιοποιηθεί και το υλικό έχει κοινοποιηθεί στη διάθεση της aνθρωπότητας. Παρακάτω παρατίθεται ο κατάλογος των βιβλίων της Εθνικής Βιβλιοθήκης Αργuρουπόλεως που σχετίζονται με την μαθηματική επιστήμη. Συ' α έα�

. iiiiiiiiiiiBίΓίτλoι: i Βι 1\(J\;\i 1.

2.

3. 4.

5.

6.

7.

8.

9.

10. 11. 12.

13.

14. 15.

16.

17.

18. 19.

20.

21. 22.

23.

Στοιχεία Αριθμητικής Στοιχεία Γεωμετρίας Στοιχειώδης Αριθμητική Πρακτική Αριθμητική Συμπλήρωμα Αριθμητικής Μαθήματα Άλγεβρας Των μαθηματικών στοιχείων Σειρά στοιχειώδους μαθηματικής Στοιχείων μαθηματικών εκ παλαιών κ νεωτέρων Στοιχειώδης Αριθμητική Γεωμετρίας στοιχειώδη μαθήματα Σειράς στοιχειωδών Μαθηματικών Άλγεβρας μαθήματα Έκθεσις συνοπτική Αριθμητικής, Άλγεβρας και Χρονολογίας Στοιχειώδης Αριθμητική Στοιχειώδης Γεωμετρία & Τριγωνομετρία Στοιχεία Γεωμετρίας Οδού Μαθηματικής Μέρος Πρώτον Στοιχειώδους των Μαθηματικών και Φυσικών Πραγματειών Εγχειρίδιον της Προκαταρκτικής Διδασκαλίας προς χρήσιν των δημοδιδασκάλων

Tables de Logaήthmes a cinq decima\es Σύνοψις των κωνικών τόμων Των κωνικών τόμων αναλυτική πραγματεία

Κόνδης, Γ. Legendre, Α.Μ. Γεράκης, Γ. Αθ. Βάφας,Χ. Βάφας,Χ. Βάφας,Χ. Σέγνερος,Ι., Ανδρ. Bourdaum

Νικηφόρος Τερζάκης , Η.Α. Βάφας, Χ. Βάφας,Χ. Βάφας,Χ. Μπαλάνος, Κοσμάς Βασιλόπουλος. Γεράκης, Γ. Αθ.

Legendre, Α.Μ. Δημ. Π, Γοβδελάς Κ. Μ. Κούμα Λαρισσαίου Κωνσταντινίδης Γ. J.

Dupuis

Grandi, Guido

Γαζής, Άνθιμος

ΕΥΚΛΕΙΔΗΣ Β' 93

τ.l/8

Χ ονοί.ο ία Έκδο σ ι:

Αντώνιος Λαμπρινίδης, 1866 Καραμπίνος & Βάφας, 1860 Ανδρέας Κορομηλάς, 1845

1847

Καραμπίνος & Βάφας, 1848 Καραμπίνος & Βάφας, 1851

Vreitls2PL 1767

Anton ν. Hayku1,

1828 Ρηδηγέρος & Κλαύδιος, 1798 Π.Δ. Σακελλάριος, 1868 Καραμπίνος & Βάφας,1861 Κωνσταντίνος Βάφας,1848 Σηλυβριώτης & Βάφας,1866 Μαρκ. Πούλιος, 1798 Λαζαρίδης, 1847 Απελίδης, 1849 Γκαρπόλας, 1840,_ 1812

_

__

1807 1873

1916 1802 1803


Μαθηματικοί Δtαyωvισμοi Μαθηματικές Ολυμπιάδες

Ε.Μ.Ε.

Προκριματικός διαγωνισμός 2014 12 Απριλίου 2014 Πρόβλημα 1

Έστω ( xn ) , n � 1 , μια ακολουθία πραγματικών αριθμών με χ1 αναδρομική σχέση

2xn+l

=3 xn + �sx; - 4 .

=1, που ικανοποιεί την

(α) Να αποδειχθεί ότι όλοι οι όροι της ακολουθίας είναι φυσικοί αριθμοί. (β) Να εξετασθεί αν υπάρχει όρος της ακολουθίας που να διαιρείται με το 2011. (Σ. Μπραζιτίκος) Λύση

Από τη δοθείσα αναδρομική σχέση παίρνουμε ότι

(2χn+ι - 3xn)2 =5 χ; - 4�4χ;+ι -12xn+ιxn + 4χ; =-4 � χ;+ι - 3xn+ιxn + χ; =-1 (1 ). Αν τώρα γράψουμε τη τελευταία για n το n + 1 παίρνουμε χ;+2 - 3xn+2xn+Ι + χ;+Ι =-1 (2). Θεωρούμε τώρα τη δευτεροβάθμια εξίσωση: χ 2 - 3χ χn+ι + χ;+ι + 1 =Ο. Λόγω των (1 ) και (2), αυτή η δευτεροβάθμια έχει λύσεις τα xn , xn+2 , οπότε από τους τύπους του Vieta παίρνουμε: και (3) (4). Γράφοντας τη σχέση (3) ως xn+2 =3χn+Ι - xn και αφού οι πρώτοι δύο όροι χ1 =1 και χ2 =2 είναι ·

ακέραιοι, επαγωγικά βλέπουμε ότι όλοι θα πρέπει να είναι ακέραιοι. Για το δεύτερο σκέλος του ερωτήματος, ας υποθέσουμε ότι υπάρχει κάποιος όρος της ακολουθίας , έστω ο xs , έτσι ώστε: 20111 xs. Τότε η σχέση (4) για n=s, δίνει xsxs+2 =χ;+ι + 1 . Αφού οι όροι είναι ακέραιοι και 20111 xs , θα έχουμε ότι:

20lllx;+l +1�χ;+Ι Ξ-1(mod20ll)� ( χ;+1)1005 Ξ (-1)1005 Ξ -1(mod2011) � χ;��Ο Ξ-1(mod2011) (5) Όμως ο 2011 είναι πρώτος και (xs+I'2011)=1 . Πράγματι, αν (xs+l'2011) =d >1' τότε

dlxs+l'dl2011 � dlxs+l' dlxs, αφού 2011lxs' οπότε από τη σχέση xsxs+2 =x;+l + 1 προκύπτει

ότι dl1 , άτοπο. Επομένως, από το θεώρημα Fennat θα έχουμε ότι: χ;��ο 1(mod2011) , (6) η οποία έρχεται σε αντίφαση με τη σχέση (5) Επομένως, δεν υπάρχει όρος της ακολουθίας που να διαιρείται με 2011 . Ξ

Σ η μ είωση . Πρέπει να παρατηρήσει κανείς ότι δε μπορεί να μιλήσει για διαιρετότητα στη σχέdη (2), γιατί δε νω γ ρίζουμε αν όλοι οι όροι είναι ακέραιοι ή όχι. Επομένως για το 2° σκέλος της απάντησης απαιτείται το ]0 Πρόβλη μ α 2 •

Βρείτε όλα τα μη μηδενικά πολυώνυμα με πραγματικούς συντελεστές που ικανοποιούν την ισότητα Λύση

(Ρ( χ ) Υ+ 3(Ρ( χ) )2 =Ρ( χ3 } - 3Ρ( -χ) ,

(1) για κάθε χ ε JR.

(Α. Φελλούρης)

Έστω ότι deg Ρ(χ)=Ο, οπότε Ρ (χ)=α * Ο. Τότε από τη σχέση ( 1) προκύπτει ότι: α3 + 3α2 =α- 3α <=> α3 + 3α2 + 2α=Ο<=> α=-1 ή α=-ο--2. Άρα τα σταθερά πολυώνυ μα Ρ(χ)=-1 και Ρ(χ)=-2 είναι λύσεις του προβλήματος. Έστω degP(x)=n > Ο. Τότε το πολυώνυμο Ρ(χ) μπορεί να γραφεί στη μορφή ΕΥΚΛΕΙΔΗΣ Β' 93 τ.l/9


------ Μαθηματικοί Διαγωνισμοί - Μαθηματικές Ολυμπιάδες

------

Ρ(χ)=axn +Q(χ), με α:;e Ο και degQ(χ)=k � n-1 ή Q (χ)=Ο . Με αντικατάσταση στη σχέση (1) λαμβάνουμε: 2 (axn +Q(x))3 +3(axn +Q(x)) =ax3 n +Q(x3 )-3Q(-x)-3α·(-1 )n xn , (2) για κάθε χΕ IR , οπότε εξισώνοντας του συντελεστές του x3 n των πολυωνύμων των δύο μελών, a;<O

λαμβάνουμε: α3 =α<=> α= 1 ή α=-1 . Επομένως διακρίνουμε τις περιπτώσεις: Ι. α= 1 . Τότε η σχέση (2 ) γίνεται: (xn +Q(x))3 +3{ xn +Q(x))2 =x3 n +Q(χ3 )-3Q(-χ)-3·(-1Υ Xn <=>Α( Χ)=Β(χ), (3) όπου έχουμε θέσει Α(χ)=3x2nQ(χ)+3xn (Q(x))2 + (Q(χ))3 + 3x2 n +6xnQ(χ)+3(Q(χ))2 , Β(χ)=Q( Χ3 )-3Q(-Χ)-3 ·(-I)n Xn. Αν υποθέσουμε ότι Q(χ)=Ο , τότε λαμβάνουμε: 3x2 n =-3 ·(-1 )n xn, αδύνατη. Αν υποθέσουμε ότι degQ(χ)=k > Ο , τότε, αφού Ο< k < n, θα έχ ουμε: 2n+k =degA(x)=degB(χ)=max {3k , n};?: 3k => n;?: k , άτοπο . Άρα είναι degQ(x)=O, οπότε Q(x)=c;eO . Τότε η ισότητα Α(χ)=Β(χ) γίνεται: 3(c+ 1 )x2 n +3 c2 + 2c+(-1Υ xn + c3 +3c2 + 2c=Ο, για κάθε χΕ IR

)

(

<=> c+Ι =Ο, c2 + 2c+(-1 )n =Ο, c3 +3c2 + 2c=Ο <=> c=-1, n= 2m , m Ε Ν*. Επομέν ως προκύπτει το πολυώνυ μο: Ρ(χ)=x2 m-1, χΕ IR, m Ε w· . 11. α=-1. Τότε η σχέση (2) γίνεται: 3 (-Xn +Q( Χ)) +3(-Xn +Q ( Χ )) 2 =-x3 n +Q{ χ3 )-3Q( -Χ ) +3 ·(-1 )n Xn <=> Α( Χ)=Β ( Χ ) , όπου έχουμε θέσει Α(χ)=3x2 nQ(x)-3xn (Q(x))2 +(Q(χ)γ +3x2n -6xnQ(x)+3 (Q(x))2 Β( Χ ) =Q{ Χ3 ) - 3Q(-Χ)+3 ·(-1 )n Xn. Εργαζόμενοι όπως και στην προηγού μενη περίπτωση, αν υποθέσουμε ότι Q(χ)=Ο , τότε

·

λαμβάνουμε: 3x2n =3 (-1 )n xn, αδύνατη . Αν υποθέσουμε ότι degQ(χ)=k > Ο , τότε, αφού Ο< lf< n, θα έχουμε: 2n+k =degA(x) =degB(x)=max {3k,n};?: 3k => n;?: k , άτοπο. Άρα είναι degQ(x)=O, οπότε Q(x)=c:;CO . Τότε η ισότητα Α(χ)=Β(χ) γίνεται: 3(c+1)x2n -3 c2 +2c+(-1 )n xn +c3 +3c2 + 2c=Ο, για κάθε χΕ IR

(

)

+ 1 =0, c2 +2c+(-1 )n =0, c3 +3c2 + 2c=0 <::::> C =-1, n= 2m , m Ε Ν*. Επομένως προκύπτει το πολυώνυ μο: Ρ(χ)=-x2 m -1,χΕ IR, m Ε w·. Άρα τα πολυώνυμα που ζητάμε είναι τα εξής: Ρ(χ)=-1 , Ρ(χ)=-2, P(x)=x2 m-1, P(x)=-x2 m-1, xElR,m EN*. <::::> C

Πρόβλη μα 3

Δίνεται οξυγώνιο σκαληνό τρίγωνο ΑΒΓ με ΑΒ < ΑΓ

<

ΒΓ. Έστω Δ,Ε,Ζ τα μέσα των

πλευρών ΒΓ,ΑΓ,ΑΒ αντίστοιχα και τα ύψη ΒΚ, ΓΛ . Στη προέκταση του ΔΖ θεωρούμε

σημείο Μ, έτσι ώστε η παράλληλη από το Μ προς την ΚΑ να τέμνει τις προεκτάσεις των Γ Α, ΒΑ και ΔΕ στα σημεία Σ, Τ και Ν αντίστοιχα. (Ολες τις προεκτάσεις, τις θεωρούμε

προς το μέρος του άκρου που εμφανίζεται δεύτερο στη γραφή του αντίστοιχου τμήματος).

Αν ο περιγεγραμμένος κύκλος του τριγώνου ΜΒΔ , έστω

ΕΥΚΛΕΙΔΗΣ Β ' 93 τ.l/10

( c1), τέμνει την ευθεία ΔΝ


------ Μαθηματικοί Διαγωνισμοί - Μαθηματικές Ολυμπιάδες

-------­

στο σημείο Ρ και ο περιγεγ ραμμένος κύκλος του τριγώνου ΝΓΔ , έστω ( c2 ) , τέμνει την (Ε. Ψύχας) ευθεία ΔΜ στο σημείο Π, aποδείξτε ότι ΣΤ 1 1 ΠΡ . Λύ ση Τα σημεία Δ, Ε, Ζ είναι μέσα των πλευρών ΒΓ, ΑΓ, ΑΒ (του τριγώνου ΑΒΓ ) αντίστοιχα.

Άρα τα τετράπλευρα ΑΕΔΖ , ΖΕΔΒ και ΖΕΓΔ είναι παραλληλόγραμμα. Εφόσον ΔΜ 11 ΓΣ , θα ισχύει: Μ = i1 Εφόσον ΜΝ 11 ΚΑ , θα ισχύει: i1 = Κ . Από το εγγράψιμο τετράπλευρο ΒΛΚΓ (διότι ΒΛΓ = Bkr = 90° ) έχουμε: Κ = Β . Από τις τρεις τελευταίες ισότητες γωνιών ισχύει Μ = Β , οπότε τα σημεία Μ, Β, Δ, Τ , ανήκουν στον ίδιο κύκλο. Εφόσον ΔΝ 11 ΒΤ , θα ισχύει: Ν = fΊ . Εφόσον ΜΝ 11 ΚΑ , θα ισχύει: fΊ = Λ . Από το εγγράψιμο τετράπλευρο ΒΛΚΓ (διότι ΒΛΓ = ΒΚΓ = 90° ) έχουμε: Λ = f . Από τις τρεις τελευταίες ισότητες γωνιών ισχύει Ν = f , οπότε τα σημεία Ν, Γ, Δ, Σ , ανήκουν στον ίδιο κύκλο. •

\��:.�.

�--

··

............

-····

Δ ·--.. . .......... ··

Γ

--����/

Σχήμα 1 Τα σημεία Β, Δ , Ρ, Τ, Μ ανήκουν στο κύκλο (c1 ) , οπότε από το εγγεγραμμένο τετράπλευρο ΜΤΡΔ λαμβάνουμε: fΊ + Δ = 180° . Τα σημεία Γ, Δ , Π, Σ, Ν ανήκουν στο κύκλο ( c2 ) , οπότε από το εγγεγραμμένο τετράπλευρο ΔΠΣΝ λαμβάνουμε: i2 + Δ = 180° . Από τις δύο τελευταίες ισότητες έχουμε: fΊ = i2 = 180° - Δ = 180° - Α . Στο τετράπλευρο ΣΤΡΕ έχουμε: fΊ +Ε= 18rf - Α+ Α = 18rf , δηλαδή το τετράπλευρο είναι εγγράψιμο σε κύκλο (έστω ( c) ). Στη συνέχεια θα αποδείξουμε ότι το τετράπλευρο ΤΡΕΖ είναι εγγράψιμο (οπότε και το σημείο Ζ θα ανήκει στο κύκλο ( c) ) . Θα αποδείξουμε ότι: ΤΖΕ + ΤΡΕ = 180° ( ΤΡΕ = ΤΡΔ ). Το τετράπλευρο ΜΤΡΔ είναι εγγεγραμμένο στον κύκλο (c1 ) , οπότε έχουμε ότι: ΊΡΔ = 18rf - JM1 = 18rf - τiJr = 18rf - Β . Ισχύει επίσης ΤΖΕ = ΤΒΓ = Β . Προσθέτοντας τις δύο τελευταίες ισότητες κατά μέρη, έχουμε: ΤΖΕ + ΤΡΕ = 180° - Β + Β = 180° . Τέλος θα αποδείξουμε ότι και το σημείο Π ανήκει στο κύκλο ( c) . Η γωνία i2 = 180° - Α είναι εξωτερική στο τρίγωνο ΣΠΜ , οπότε: ΣiΙΖ = ΣiΙΜ = i2 - Μ = 180° - Α - Β = f = ΣΕΖ . Το τετράπλευρο ΣΤΡΠ είναι εγγεγραμμένο στο κύκλο ( c ) και επειδή fΊ = i2 , αυτό είναι ισοσκελές τραπέζιο. Π ρ ό β λημ α 4

Τετράγωνο ABCD διαιρείται σε n 2 ίσα μικρά (στοιχειώδη) τετράγωνα, σχεδιάζοντας ευθείες παράλληλες στις πλευρές του. Μία αράχνη ξεκινά από το σημείο Α και κινούμενη μόνο προς τα δεξιά και προς τα άνω, ΕΥΚΛΕΙΔΗΣ Β ' 93 τ.l/1 1


-------

Μαθηματικοί Διαγωνισμοί - Μαθηματικές Ολυμπιάδες

--------­

προσπαθεί να φθάσει στο σημείο C. Κάθε "κίνηση" της αράχνης αποτελείται ή από k στοι;r.ει6όη βήματα δεξιά και m στοιχειώδη βήματα άνω ή από m στοιχειώδη βήματα δεξιά και k στοι;r.ει6όη βήματα άνω (τα οποία μπορεί να τα κάνει, με όποιο τρόπο θέλει). Η αράχνη κάνει I "κινήσεις" και στη συνέχεια κινείται (πάντα προς τα δεξιά και άνω) ;r.αι ρ ί.; κανένα περιορισμό. Αν n = m · I , να βρεθεί ο αριθμός όλων των δυνατών τρόπων με τους οποίου.; η αράχνη μπορεί να φθάσει το σημείο C , όπου n, m , k , l είναι θετικοί ακέραιοι με k < m . (Ε. Ψύχα.;) Λύ σ η Υποθέτουμε ότι το τετράγωνο είναι τοποθετημένο σε ορθοκανονικό σύστημα αναφοράς με αρχή το σημείο Α και τους άξονες να ταυτίζονται με τις πλευρές ΑΒ και AD . Η αράχνη ξεκινά από το σημείο Α και κάνει τη πρώτη "κίνηση" κινούμενη m βήματα επάνω και k δεξιά ή m βήματα δεξιά και k επάνω. (Στο σχή μα φαίνεται η περίπτωση για m = 4 και k = 3 ) . . • . Μετά την ολοκλήρωση της πρώτης "κίνησης", η αράχνη μπορεί να • ·

βρίσκεται

στο

M (k, m) ( Ξ Μ(3, 4) )

σημείο

ή

στο

σημείο

N( m, k) (Ξ N (4, 3) ) .

( J( J

Οι τρόποι με τους οποίους μπορεί να προσεγγιστεί το σημείο M(k, m) ή το σημείο N(m, k) είναι όσοι οι συνδυασμοί των

k+m

αντικειμένων ανά

k

, δηλαδη

'

k+m k

_

_

k+m m

_

ν,

Γ; ) 4

s

1s

1

4

10

20

3

6

10

1s

2

3

4

5

1

_

Μ

1

3S =

Ν

35 =

.

( � ) • 3

4

.. .

.

γιατί κάθε σημείο του πλέγματος (μέχρι να φτάσουμε στο σημείο Α Σχήμα 2 M (k, m) ή το σημείο N (m, k) ), μπορεί να προσεγγιστεί με το άθροισμα των τρόπων που προσεγγίζονται το πλησιέστερο σημείο από τα αριστερά και το πλησιέστερο σημείο από κάτω. Μετά την ολοκλήρωση της δεύτερης "κίνησης", η αράχνη μπορεί να βρίσκεται σε τρία διαφορετικά σημεία ( K(2k, 2m) , L(k + m, k + m) ) και T(2m, 2k) με αντίστοιχους , , , , τ , 2 ο σημειο Μ , μπορουμε να το προσεγγισουμε με ν τροπους προσεγγισης ν 2 , ν 2 και ν 2 . διαφορετικούς τρόπους (από το σημείο Α ). Το σημείο Κ , μπορούμε να το προσεγγίσουμε με ν διαφορετικούς τρόπους (από το σημείο Μ ) . Άρα το σημείο ν' =

(�)

· ν3

) • ν' Σ

μπορούμε να το προσεγγίσουμε με ν 2 διαφορετικούς τρόπους (από το σημείο Α ). Με όμοιο τρόπο βρίσκουμε ότι και το σημείο Τ ,

Κ,

(�)

• ν'

3 · ν' =

(�) ν• =

μπορούμε να το προσεγγίσουμε με τρόπους (από το σημείο Α ).

. y>

(�)

. y3

ν2

διαφορετικούς

Το σημείο L μπορούμε να το προσεγγίσουμε με ν 2 διαφορετικούς τρόπους (ακολουθώντας τη διαδρομή

A,M,L ) και με ν 2 διαφορετικούς τρόπους (ακολουθώντας τη διαδρομή A,N ,L ) Δηλαδή το σημείο L μπορούμε να το .

Α

Σχήμα 3

' ' τρόπους (από το προσεγγισουμε με 2 ν 2 δ ιαφορετικους σημείο Α ). Με ανάλογο τρόπο συμπεραίνουμε ότι:

(� J ( � J (� J ( �J

Μετά την ολοκλήρωση της τρίτης κίνησης, η αράχνη μπορεί να βρίσκεται σε τέσσερα διαφορετικά σημεία, τα οποία μπορούμε να τα προσεγyίσουμε με

·

ν'

,

·

ν'

,

·

ν'

και

·

ν' , αντίστοιχα.

Επομένως, μετά την ολοκλήρωση της I κίνησης, η αράχνη μπορεί να βρίσκεται σε I + 1 διαφορετικά σημεία, τα οποία θα βρίσκονται προφανώς επάνω στην ευθεία με εξίσωση : χ + y = I ( k + m) .

r = m - k , τότε τα σημεία αυτά (με σειρά από δεξιά προς αριστερά) θα είναι τα: A0 (/m, lk) , A 1 (1m - r, lk + r) , A 2 (1m - 2r, lk + 2r ) , . . A1 (1m - lr, lk + lr) Ξ A1 (/k, lm) .

Αν τώρα θέσουμε

ΕΥΚΛΕΙΔΗΣ Β ' 93 τ.l/12


------

Μαθηματικοί Διαγωνισμοί - Μαθηματικές Ολυμπιάδες -----... ''!"' . ······ •

.. . . . . !

. . . · •·

· · · · · · • · · · · · ·•

. . .. . . . · • · · ·

· ·

·

i ΓΗΙ�-] - ---

� + :

-- ---·

''

· · · · · · - - - - - - ·• - - · · · · ·

ι

c

+ '' ι

-- -. -- . -- ....

. . . .. . . . . . . �- - -· · · · • · · · · · ·• · · · · · ·•

;

,

·• · · · · · · · • · ·

Α 1(lnι-r,Uι+r) •· · · · · · · • · · · · · · • · · · · · · •

Σχήμα 4 Τα σημsία αυτά μπορούμs να τα προσεyyίσουμs μ>:

(;} ν' , ( � 1} ( � 2} ' ν ,

1

ι

' v .

... . και

) C(ml,ml), ) + (ml-lk)J = (ml-lkJ = (lr J = ( lr J =1 τρόπους (**). Ao (lm,lk) με ((ml-lmml-l k ml-lk lr O·r , k -r)J = ( ml-lkJ = ( lr J τροπους. A1(lm -r,lk +r) με ( (ml-lm +r)+(ml-l r r 1·r k -2r)J = ( ml-lkJ = ( lr J τροπους. , �(lm -2r,lk +2r) με ((ml-lm +2r)+(ml-l 2r 2·r 2r )+(ml-lm)J = (/rJ = τρόπους. A1(lk,lm) με ( (ml-lkml-l k lr Άρα το σημείο C( ml, ml) , μπορεί να προσεγγιστεί από το σημείο Α με: ((;}(�Hz � !}(';Hz � 2} (�]+- - · +(�} (;:JJ

τρόπους αντίστοιχα (από το σημείο Α . Το σημείο μπορεί να προσεγγιστεί από το σημείο:

(�} ν'

1

ν

'

τρόπους.

(**) Η μετακίνηση από το σημείο δεξιά

και

άνω), μπορεί να γίνει με

(ί,j) (i+k,j+m) ( k : m J = ( k :m J στο σημείο

του πλέγματος (κινούμενοι προς τα

τρόπους. Η διαδικασία εύρεσης του πλήθους των

τρόπων, είναι η ίδια με τη διαδικασία εύρεσης των τρόπων μετακίνησης στη πρώτη "κίνηση ".

Σημείωση. Στην περίπτωση που είναι

m-k = r = 1,

τότε ο αριθμός των δυνατών τρόπων απλοποιείται

ΕΥΚΛΕΙΔΗΣ Β ' 93 τ.l/13


------- Μαθηματικοί Διαγωνισμοί - Μαθηματικές Ολυμπιάδες -----17°ς Μ ΕΣΟΓΕΙΑΚΟΣ ΜΑΘ Η ΜΑ1Ι ΚΟΣ ΔΙΑΓΩ Ν ΙΣΜΟΣ 2014 PETE R Ο' HALLORAN M EMO RIAL 13 Απριλίου 2014

Πρόβλη μα 1 Έστω a1 , a 2 , , an , και b1 , b2 , , bn , 2n πραγματικοί αριθμοί. Να αποδείξετε ότι υπάρχει n n ακέραιος k μ ε ι � k � n τέτοιος ώστε a; - ak � ak •••

•••

Ι Σ lh; - ι .

ΣΙ i=l

i=l

Λύση

Έστω A,in = min { α1 , α2 , .. . , an } και Α.ηaχ max { α1 , α2 , ... , an } . Τότε, από την τριγωνική ανισότητα, για κάθε ί = 1 , 2, . .. , n έχουμε: IA.ηax - ai Ι + i ai - Α.ηίη Ι = ( Α.ηaχ - ai ) + ( ai - Α.ηίη ) = Α.ηaχ - A.ηin I A.ηax - Α.ηίη Ι = I A.ηax - bi + bi - Amin I � I A.ηax - bi I + lbi - Α.ηίη ι . Με πρόσθεση κατά μέλη των παραπάνω ανισοτήτων λαμβάνουμε =

=

n n n n (1) Σ1 IΑ.ηaχ - α; Ι + Σ1 l ai - Α.ηίη l � Σ IΑ.ηaχ - b; l + Σ1 lbi - A.ηin l · i= i= i= i=l Επομένως , ο ακέραιος k για τον οποίο ισχύει ότι ak = Α.ηaχ ή ak = A.ηin ικανοποιεί τη

ζητούμενη ανισότητα. Πράγματι, αν ίσχυαν και οι δύο ανισότητες που ακολουθούν

n n n n > > , Σ1 I Α.ηaχ - ai i Σ1 IΑ.ηaχ - bi l Σ1 l ai - Α.ηίη l Σ1 lbi - Α.ηίη l' i= i= i= i=

τότε με πρόσθεση κατά μέλη θα προέκυπτε η ανισότητα

n n n n > Σ1 IΑ.ηaχ - ai i + Σ1 l ai - Α.ηίη l Σ1 IΑ.ηaχ - bi l + Σ1 lbi - Α.ηίη l' i= i= i= i=

η οποία είναι αντίθετη της ( 1 ), άτοπο.

(2)

Πρόβλη μα 2. Θεωρούμε αύξουσες ακολουθίες ακέραιων με στοιχεία από το σύνολο ι, 2, ... , ιΟ 6 . Μία

τέτοια ακολουθία k

tk

=

>

( a1 , a2 ,

•••

, an

)

{

}

λέγεται «Αδριατική» , αν a1 = ι και ak � 2ak -Ι , για κάθε

2,3, , n . Επίσης, μία τέτοια ακολουθία λέγεται «Τυρρηνική», αν tn = ι Ο 6 και ..•

t1 + t2 + ... + tk - Ι , για κάθε k = 2, 3, ... , n . Ν α εξετάσετε, αν ο αριθ μός των «Αδριατικών»

ακολουθιών είναι (i) μικρότερος «Τυρρηνικών» ακολουθιών. Λύση

(ίί)

ίσος

(ίίί) μεγαλύτερος, από τον αριθμό των

Θεωρούμε μία Αδριατική ακολουθία ( α1 , αυ . . . , an ) με α1 = 1 και από αυτή κατασκευάζουμε μία καινούρια ακολουθία α2 - 1 , α3 - α2 , ... , an - an _ 1 , 1 0 6 ) Παρατηρούμε ότι η καινούρια ακολουθία είναι Τυρρηνική, αφού ( α2 - 1 ) + ( α3 - α2 ) + · ·· ( ak-1 - ak- 2 ) � ak- 1 - 1 < ak - αk-Ι ' ισχύει, για κάθε k = 2, 3, ... , n - 1 και επιπλέον ( α2 - 1 ) + ( α3 - α2 ) + ... ( an - an _ 1 ) an _ 1 - 1 < 1 0 6• Στη συνέχεια θεωρούμε μία Τυρρηνική ακολουθία ( t1 , t2 , . . . , tn ) με tn = 1 0 6 και από αυτή κατασκευάζουμε μία καινούρια ακολουθία ( 1 , 1 + t1 , 1 + t1 + t2 , . .. , 1 + 11 + ... + tn _ 1 ) . Παρατηρούμε ότι η νέα ακολουθία είναι Αδριατική, αφού ισχύει ότι

(

=

ΕΥΚΛΕΙΔΗΣ Β ' 93 τ.l/14


-------

Μαθηματικοί Διαγωνισμοί - Μαθηματικές Ολυμπιάδες

------

2 ( 1 + 11 + ... + tk_1 ) � 1 + t1 + ... + tk <::::> 11 + ... + tk-I < tk . Επομένως, από τις δύο παραπάνω κατασκευές έχουμε δύο αμφιμονοσήμαντες αντιστοιχίες μεταξύ των συνόλων των Αδριατικών και Τυρρηνικών ακολουθιών, οπότε τα σύνολα αυτά έχουν το ίδιο πλήθος στοιχείων. Εύκολα διαπιστώνουμε ότι η δεύτερη αντιστοιχία είναι η αντίστροφη της πρώτης.

Πρόβλη μα 3 Να αποδείξετε ότι για κάθε ακέραιο S � 100 υπάρχει ένας ακέραιος Ρ για τον οποίο η ακόλουθη ιστορία μπορεί να επαληθευτεί: Ο μαθηματικός ρωτάει τον ιδιοκτήτη του καταστήματος «Πόσο κοστίζουν τα τραπέζια, τα ντουλάπια και τα ράφια ;» Ο ιδιοκτήτης του καταστήματος απαντάει: «Κάθε αντικείμενο κοστίζει ένα ακέραιο (θετικό) αριθμό ευρώ. Το τραπέζι είναι ακριβότερο από το ντουλάπι και το ντουλάπι είναι ακριβότερο από το ράφι. Το άθροισμα των τριών τιμών είναι S και το γινόμενο τους είναι ίσο με Ρ . Ο μαθηματικός σκέπτεται και διαμαρτύρεται: «Αυτές δεν είναι αρκετές πληροφορίες για να προσδιοριστούν οι τρεις τιμές» Λύση

Γράφουμε τον ακέραιο S στη μορφή S = 6k + r, με k, r ακέραιους τέτοιους ώστε 1 � r � 6 και k > 2r . Ισχυριζόμαστε ότι ο αριθμός Ρ = 6k ( k - r ) ( k + r ) είναι μία κατάλληλη επιλογή. Πράγματι, σημειώνουμε την τιμή που έχει το τραπέζι, το ντουλάπι και το ράφι με x, y, και z, αντίστοιχα. Τότε έχουμε δύο δυνατότητες: 1 . x = 3 (k + r) , y = 2 (k - r) , z = k 2. x' = 3k, y' = 2 ( k + r ) , z' = k - r Οι δύο περιπτώσεις αυτές είναι διαφορετικές, αφού εύκολα διαπιστώνουμε ότι ισχύουν: χ > y > z, χ' > y' > z' και χ' < χ, y < y', z > z' .

Πρόβλη μα 4 Οι διάμεσοι AA', BB', CC' του τριγώνου ABC τέμνουν το κύκλο των εννέα σημείων του, στα σημεία D, E και F , αντίστοιχα. Τα σημεία L, M και Ν των ευθειών BC, CA και ΑΒ, αντίστοιχα, είναι τα ίχνη των υψών του τριγώνου από τις κορυφές Α, Β και C, αντίστοιχα. Οι εφαπτόμενες του κύκλου των εννέα σημείων στα σημεία D, Ε και F , τέμνουν τις ευθείες ΜΝ, LN και LM στα σημεία P, Q και R , αντίστοιχα. Να αποδείξετε ότι τα σημεία Ρ , Q και R είναι συνευθειακά. Λύση

Α Ρ

Β

Ισχύει η ισότητα των γωνιών: D ι = Ν ι , γιατί η και την γωνία Dι σχηματίζεται από τη χορδή εφαπτομένη και η γωνία Νι είναι η αντίστοιχη του κύκλου εγγεγραμμένη που βαίνει στη χορδή του Euler (Σχήμα 1 ). Άρα τα τρίγωνα και είναι όμοια, οπότε:

DP

Σχήμα 1

M D PD = ΡΜ = DM PD DN PD = DM ΡΝ PD ND ΡΝ DN �

ΡΜ

=

}

PDM PND ΡΜ = ΡΝ

ΕΥΚΛΕΙΔΗΣ Β ' 93 τ.1/15

2 M D ( DN ) ( 1 )

DM DM


----- Μαθηματικοί Διαγωνισμοί - Μαθηματικές Ολυμπιάδες Τα ορθογώνια τρίγωνα BCN και BCM , έχουν κοινή υποτι;:ίνουσα την BC (Σχήμα 2) . Άρα οι κορυφές Μ, Ν θα βρίσκονται στον κύκλο με κέντρο το μέσο Α' της BC και ακτίνα BC α = ' δηλαδη' Α'Μ = Α' Ν = α . 2 2 2

-----­

Α

Σχήμα 2

Τα ευθύγραμμα τμήματα Α' Μ, Α' Ν είναι χορδές του κύκλου του Euler στα αντίστοιχα τόξα των οποίων βαίνουν οι γωνίες 61 και 6 2 , δηλαδή 61 = 6 2 και κατά συνέπεια η DA' είναι διχοτόμος της γωνίας Ν6Μ . Εφαρμόζοντας τώρα το θεώρημα των διχοτόμων στο τρίγωνο DM τΜ , -DMN εχουμε: = DN τΝ -

.

Α

Η ΜΝ

Σχήμα 3 ' ' ' 3) , οποτε: ' τΜ = ΑΜ 2 = c 2 ' ομως ειναι αντιπαραλλη λη της BC (Σχημα -- --- τΝ r

ΑΝ 2

b2

ΡΜ c 4 , , , αποτελεσματα, εχουμε: = b4 . Συνδυα, ζοντας τα παραπανω ΡΝ 4 QN α 4 , , ' , , απο δ εικνυουμε b , , RL Με ομοιο τροπο οτι: =4 και -- = 4 . Μ ε πο λλαπλασιασμο κατα μελη RM a QL c ΡΜ · QN · RL l , παραπανω , ισοτητων , , , απο, το αντιστροφο , λαμβ ανουμε του των τριων = , οποτε PN · QL · RM θεωρήματος του Μενελάου στο τρίγωνο MNL , συμπεραίνουμε ότι τα σημεία P, Q, R είναι συνευθειακά. ΕΥΚΛΕΙΔΗΣ Β' 93 τ.l/16


------ :\Ιαθη ματικοί Διαγωνισμοί - Μαθηματικές Ολυμπιάδες

------

Βαλκανική Μαθη ματική Ολυ μπιάδα Βουλγαρία, Πλέβεv� 2 - 7 Maϊou 2014

3 1η

Η 3 ιη Βαλκανική Μαθηματική Ολυμπιάδα έγινε στο Πλέβεν της Βουλγαρίας, από 2 έως 7 Μαίου 2Ο ι4. Έλαβαν μέρος συνολικά 20 χώρες, Σύμφωνα με τον κανονισμό επισήμως συμμετείχαν οι χώρες: Αλβανία, Βουλγαρία, Ελλάδα, Κύπρος, Μαυροβούνιο, Μολδαβία, ΠΓΔΜ (Fyrom), Ρουμανία, Σερβία και Τουρκία, ενώ ανεπισήμως συμμετείχαν οι ομάδες των χωρών: Αζερμπαϊτζάν, Ηνωμένο Βασίλειο, Ιταλία, Καζακστάν, Κιργιστάν, Ουζμπεκιστάν,

Σαουδική Αραβία, Τατζικιστάν, Τουρκμενιστάν και Βουλγαρία (Β ' Ομάδα). Η Ελληνική ομάδα κατέκτησε 2 χρυσά μετάλλια, 3 χάλκινα μετάλλια και 1 εύφημη μνεία,

ως εξής:

Δημάκης Παναγιώτης Σκιαδόπουλος Αθηναγόρας Ντούνης Πέτρος Χαχάμης Νέστορας Ζητρίδης Αντώνιος Σαράντης Μιχαήλ

Σχολή Μωραίτη Ροδίων Παιδεία 1 ο ΓΕΛ Κορωπίου ΓΕΛ Παλαίρου Αιτωλοακαρνανίας Ελ. Σχ. Ουρσουλινών Λεόντειο Λύκειο Πατησίων

Χρυσό μετάλλιο Χρυσό μετάλλιο Χάλκινο μετάλλιο Χάλκινο μετάλλιο Χάλκινο μετάλλιο Εύφημη μνεία

Συνοδοί των μαθητών ήταν ο μαθηματικός Σιλουανός Μπραζιτίκος και ο διδάκτωρ μαθηματικός Ξενοφών Δημητρίου. Οι μαθητές διαγωνίζονται σε τέσσερα προβλήματα που καθένα από αυτά βαθμολογείται με ι Ο μονάδες. Σύμφωνα με την άτυπη βαθμολογία η Ελλάδα με 182 μονάδες κατέλαβε την 5η θέση μεταξύ των επισήμως συμμετεχουσών χωρών, ενώ ήταν επίσης στην 5η θέση μεταξύ των 20 ομάδων που συνολικά έλαβαν μέρος. Τις τέσσερις πρώτες θέσεις κατέλαβαν οι ομάδες της Βουλγαρίας (221), Τουρκίας (214), Ρουμανίας (209) και Σερβίας (187). Τα π ρο βλή ματα

Πρόβλη μα 1 . Θεωρούμε τους θετικούς πραγματικούς αριθμούς x, y και z που είναι τέτοιοι ώστε xy + yz + zx = 3xyz . Να αποδείξετε ότι: x 2 y + y 2 z + z 2 x � 2 x + y + z - 3 και να

(

)

προσδιορίσετε πότε ισχύει η ισότητα. Λύση ( 1°ς τρόπος)

ι ι -ι = 3 . Χρησιμοποιώντας την τελευταία xy + yz +zx =3xyz <=>-+-+ χ Υ z ισότητα έχουμε: x2y+ /z+z2x-2(x+ y+z) +3 = x2y-2x+..!.. + y2 z-2y+ .!. +z2x-2z+ .!. = z χ Υ ' Υ χ-� + z (y- + {z- ;, ο. Η ισότητα ισχύει αν, και μόνον xy= yz=zx=ι <=>χ= y=z =1 . (2°ς τ ρό πος) : Από τη σχέση .!. + ..!.. + .!. = 3 και την ανισότητα Cauchy-S chwarz, λαμβάνουμε: χ Υ z ' ' 3( fy+/z+z'+ � fy+/z+z'+ (χ,}Υ)' ψΓz)' +(zΓz)' ) ψ+y+z)2 , + + Επειδή

x, y,z > Ο ,

έχουμε:

( ) υ H Θ �)

οπότε προκύπτει ότι:

αν,

[(�J' (}.) (l) )

χ2 y + / z + z2 χ � ( χ + Υ3 + z )

2

, οπότε, αν θέσουμε

ΕΥΚΛΕΙΔΗΣ Β ' 93 τ.l/17

χ+y+z

=

t , αρκεί να


-------

Μαθηματικοί Διαγωνισμοί - Μαθηματικές Ολυμπιάδες

------

2 3 χ� = Υ� = z Fx � xy = yz = zx <=:> χ = y = z = 1 , αφου, χ + y + z = 3 . -1- -1- -1� � Fx

αποδείξουμε ότι: !_ � 2t -3 � ( t -3 ) 2 � Ο, που ισχύει. Η ισότητα ισχύει όταν:

Π ρόβλη μα 2. Ένας ειδικός αριθμός είναι ένας θετικός n για τον οποίο υπάρχουν θετικοί + , , a 3 2b 3 Ν , , d ακεραιοι α, b , c και τετοιοι ωστε: n = 3 + d3 • α απο δ είξετε οτι: c 2

(α) υπάρχουν άπειροι ειδικοί αριθμοί, ( β) ο 20 1 4 δεν είναι ειδικός αριθμός. Λύση (α) Παρατηρούμε ότι κάθε τέλειος κύβος ενός θετικού ακέραιου είναι

k3

αφού για

α, b

θετικούς ακέραιους μπορεί να γραφεί ως

k ειδικός αριθμός, 2b3 ( ka ) 3 + 2 ( kb ) 3 k3 k3 α33 ++ 2b3 α α3 + 2b3 _

_

_

_

Παρατηρούμε ότι 2014 = 2 · 19 · 53. Αν ο 2014 ήταν ειδικός αριθμός, τότε για κάποιους θετικούς ακέραιους x, y, u , ν θα είχαμε χ3 + 2/ = 2014( u3 + 2ν3 ) . (1) Υποθέτουμε ότι ο αριθμός χ3 + 2/ είναι ο ελάχιστος με την παραπάνω ιδιότητα. Στη συνέχεια θα χρησιμοποιήσουμε την συνεπαγωγή: 191 ( χ3 + 2 / ) => 19ix και 1 9l y. Πράγματι, αν ο 1 9 δεν διαιρεί το χ , τότε δεν μπορεί να διαιρεί το y . Από τη σχέση ( β)

χ3 -2/ ( mod19) ( χ3 )6 ( -2/ )6 ( mod 19) χ18 2 6 y 18 ( mod19). Τώρα, σύμφωνα με το μικρό θεώρημα του Feπnat, έχουμε: 1 2 6 ( mod 1 9) 1 91 6 3 , που είναι άτοπο. Επομένως, χ = 1 9χ1 , y = 1 9y1 , για κάποιους θετικούς ακέραιους x , y1 , οπότε η 1 σχέση (1) γίνεται: 1 9 2 (χ; + 2y; ) = 2 · 53 · ( u3 + 2ν3 ) (2) 1 91 ( 3 + 2ν3 ). Επομένως, u = 1 9u1 και ν = 1 9ν1 , οπότε η σχέση (2) γίνεται: χ; + 2y; = 2014 ( ; + 2ν; ) . Όμως, είναι φανερό ότι ισχύει χ; + 2 y; < χ3 + 2/ , που είναι αντίθετο με την υπόθεση του ελαχίστου για τον αριθμό χ3 + 2 / . Επομένως ο 2014 δεν είναι ειδικός αριθμός. =>

Ξ

Ξ

<:::::>

Ξ

=>

Ξ

=>

u

u

Π ρόβλημα 3. Έστω ABCD ένα τραπέζιο που είναι εγγεγραμμένο σε κύκλο Γ με διάμετρο ΑΒ Έστω Ε το σημείο τομής των διαγωνίων Α C και BD . Ο κύκλος με κέντρο Β και ακτίνα ΒΕ τέμνει τον κύκλο Γ στα (τημεία Κ και L , όπου το Κ είναι στο ίδιο ημιεπίπεδο της ΑΒ με το C Η κάθετη στην ευθεία BD στο σημείο Ε τέμνει την ευθεία CD στο σημείο Μ . Να αποδείξετε ότι η ΚΜ είναι κάθετη στη DL . •

Λύση

Επειδή ΑΒ 11 CD, έχουμε ότι το τραπέζιο AB CD είναι ισοσκελές. Έστω Ο το κέντρο του κύκλου Γ και έστω ότι η ΕΜ τέμνει την ΑΒ στο σημείο Q . Τότε από το ορθογώνιο τρίγωνο BEQ έχουμε: ΒΕ2 = ΒΟ · BQ . Επειδή ΒΕ = ΒΚ , λαμβάνουμε ΒΚ 2 = ΒΟ · BQ . ( 1) Υποθέτουμε ότι η ΚL τέμνει την ευθεία ΑΒ στο σημείο Ρ . Τότε, από το ορθογώνιο τρίγωνο ΒΑΚ , έχουμε ΒΚ 2 = ΒΡ · ΒΑ. (2) Από τις (1) και (2) λαμβάνουμε:

ΒΡ = ΒΟ = .!. , (3) οπότε το BQ ΒΑ 2

Ρ είναι το μέσο του BQ. Όμως, DM 1 1 A Q και MQ 1 1 AD (και οι

δύο είναι κάθετες στη DB ), οπότε το τετράπλευρο A QMD είναι παραλληλόγραμμο, οπότε MQ = AD = BC. Επομένως το τετράπλευρο QBCM είναι ισοσκελές τραπέζιο. Από την (3) ΕΥΚΛΕΙΔΗΣ Β ' 93 τ.l/18


------

έπεται ότι η συμμετρικό

ΚL

.\Ι αθη ματικοί Διαγωνισμοί - Μαθη ματικές Ολυμπιάδες

-------­

είναι μεσοκάθετη των δύο βάσεων BQ και CM , οπότε το σημείο

Μ

είναι το

.....

Σχήμα 1 του C ως προς την ευθεία ΚL. Τελικά, χρησιμοποιώντας την γνωστή πρόταση, ότι το συμμετρικό του ορθοκέντρου ενός τριγώνου ως προς κάθε πλευρά του βρίσκεται στον περιγεγραμμένο κύκλο του (και αντιστρόφως), διαπιστώνουμε ότι το σημείο Μ είναι το ορθόκεντρο του τριγώνου DLK . Π ρ όβ λη μ α 4 . Έστω n ένας θετικός ακέραιος. Ένα κανονικό εξάγωνο με πλευρά μήκους n διαιρείται σε ισόπλευρα τρίγωνα με πλευρές μήκους 1 με ευθείες παράλληλες προς τις πλευρές του. Να βρείτε τον αριθμό των κανονικών εξαγώνων των οποίων όλες οι κορυφές είναι και κορυφές των ισοπλεύρων τριγώνων. Λύση (Ε. Ψύχας)

Θα συμβολίσουμε με Tn το κανονικό εξάγωνο, του οποίου κάθε πλευρά έχει μήκος n. Το πλήθος των σημείων που δημιουργούνται πάνω και στο εσωτερικό του κανονικού εξαγώνου με πλευρά n από τις τομές των παραλλήλων είναι: Kn = 2((n + 1) + (n + 2) + · · · + 2n) + 2n + 1 = 3n ( n + 1) + 1 . (1) Σε ένα κανονικό εξάγωνο Tn , δημιουργούνται κανονικά εξάγωνα Ι; (πλευράς 1 ) που τα κέντρα τους είναι τα σημεία που δημιουργούνται στο ομόκεντρο Tn_1 κανονικό εξάγωνο πλευράς n - 1 . Δηλαδή, στο κανονικό εξάγωνο Tn ορίζονται m1 = 3(n - 1)n + 1 το πλήθος κανονικά εξάγωνα ΙΊ . Για τον υπολογισμό του m 1 = 3( n - 1 )n + 1 , θέτουμε όπου n το n - 1 στη σχέση (1) .

Σχήμα 2

Ανάλογα σκεπτόμενοι καταλήγουμε: Το πλήθος των κανονικών εξαγώνων τ; είναι: Το πλήθος των κανονικών εξαγώνων τ; είναι:

m2

3(n - 2 )( n - 1) + 1 . m3 = 3(n - 3)(n - 2) + 1 . =

ΕΥΚΛΕΙΔΗΣ Β ' 93 τ.l/19


------- Μαθηματικοί Διαγωνισμοί - Μαθηματικές Ολυμπιάδες

--------­

Το πλήθος των κανονικών εξαγώνων Tn_2 είναι: mn -2 = 3(n - n + 2)(n - n + 3) + 1 = 3 . 2 · 3 + 1 . Το πλήθος των κανονικών εξαγώνων 1',_ 1 είναι: mn-ι = 3(n - n + 1 )(n - n + 2) + 1 = 3 · 1 · 2 + 1 = 7 (Σχήμα 3). Το πλήθος των κανονικών εξαγώνων Tn είναι: mn = 3( n - n )( n - n + 1) + 1 = 3 · Ο · 1 + 1 = 1 . Στη συνέχεια, με τον όρο "εγγράφεται", εννοούμε ότι: υπάρχει κανονικό εξάγωνο, που οι κορυφές του βρίσκονται στις πλευρές του κανονικού εξαγώνου Υ: και είναι κορυφές ισοπλεύρων τριγώνων. Στο κανονικό εξάγωνο 1Ί δεν "εγγράφεται" κανονικό εξάγωνο (Σχήμα 2). Στο κανονικό εξάγωνο τ; "εγγράφεται" 1 κανονικό εξάγωνο (Σχήμα 2). Στο κανονικό εξάγωνο τ; "εγγράφονται" ί - 1 κανονικά εξάγωνα.

.

Τελικά ο αριθμός των κανονικών εξαγώνων θα είναι: Ν = 1 · mι + 2 · m2 + 3 · m + . + (n - 1) · mn - ! + n · mn . 3

·

Σχήμα 3 Ο τυχόν όρος

Άρα έχουμε

ί · m; , του παραπάνω αθροίσματος, γράφεται: ί · mi = ί(3(n - ί)(n - ί + 1) + 1 ) = 3ί(n - ί)(n - ί + 1 ) + ί n

Ν = Σ (3ί(n - ί)(n - ί + 1) + ί) i=!

και χρησιμοποιώντας τα γνωστά αθροίσματα: + 1)(2n + 1) και + 1) ί = n(n , ί 2 = n(n 6 2 i=! i=! καταλήγουμε στη σχέση:

Ι

Σ

ΕΥΚΛΕΙΔΗΣ Β' 93 τ.l/20

Σί i=l

3

(

)

1) 2 = n(n + ' 2


-------

Μαθη ματικοί Διαγωνισμοί - Μαθη ματικές Ολυμπιάδες

------

S S η Διεθνής Μαθη ματική Ολυ μπιάδα Νότια Αφρική, Κέιπ Τάουν, 3-13 Ιουλίου 2014

Η 55η Διεθνής Μαθηματική Ολυμπιάδα έγινε από 3 έως 13 Ιουλίου 2014 στην πόλη του Ακρωτηρίου (Κέιπ Τάουν) της Νότιας Αφρικής με συμμετοχή 101 χωρών και 560 διαγωνιζόμενων (504 μαθητές και 56 μαθήτριες). Η Ελληνική ομάδα κατέκτησε 2 αργυρά μετάλλια, 2 χάλκινα μετάλλια και 2 εύφημες μνείες, ως εξής: Δημάκης Παναγιώτης Ντούνης Πέτρος Χαχάμης Νέστορας

Αργυρό Μετάλλιο Αργυρό Μετάλλιο Χάλκινο Μετάλλιο Χάλκινο Μετάλλιο Εύφημη μνεία Εύφημη μνεία

Σχολή Μωρα"iτη 1 ο ΓΕΛ Κορωπίου ΓΕΛ Παλαίρου Ροδίων Παιδεία Λύκειο Αθηνά Λεόντειο Λύκειο

Σκιαδόπουλος Αθηναγόρας

Αναγνώστου Νικολέττα Ζητρίδης Αντώνιος

Επίσης η Ελληνική ομάδα κατέλαβε την 41 η θέση στην κατάταξη των χωρών. Τις τρεις πρώτες θέσεις στη κατάταξη κατέλαβαν οι ομάδες της Κίνας, Ηνωμένων Πολιτειών Αμερικής και Ταϊβάν. Αρχηγός της Ελληνικής αποστολής ήταν ο Αναπληρωτής καθηγητής του Ε. Μ. Πολυτεχνείου Ανάργυρος Φελλούρης και υπαρχηγός ο μαθηματικός Ευάγγελος Ζώτος. Εκτός αυτών, στο τελικό στάδιο της προετοιμασίας των μαθητών, έλαβαν μέρος οι συνάδελφοι μαθηματικοί Σιλουανός Μπραζιτίκος, Γιάννης Τυρλής και Ευάγγελος Ψύχας. Πρόβλη μα 1 . Έστω

Τα προβλή ματα

α0 < α1 < α 2 <

· · · μία άπειρη ακολουθία θετικών ακεραίων.

, , , , απο δ ει'ξετε οτι υπαρχει μονα δ ικος ακεραιος

n�1

,

,

τετοιος ωστε

an

<

αο

+ αι + ... + αn

Να

� an+ι

n Λύση :Για n 1, 2, ... ορίζουμε: dn ( α0 + α1 + · · · + αn ) - nαn . Είναι φανερό ότι η πρώτη στη σειρά από τις δύο ανισότητες αληθεύει, αν, και μόνον αν, dn > Ο. Σχετικά με τη δεύτερη ανισότητα παρατηρούμε ότι nαn+\ - ( αο + α\ + . . . + αn ) ( n + 1 ) αn +\ - ( αο + α\ + . . . + αn + αn +! ) -dn +\ οπότε η δεύτερη ανισότητα αληθεύει, αν, και μόνον αν, dn +ι � Ο . Επομένως, για την επίλυση του προβλήματος αρκεί να αποδείξουμε ότι υπάρχει μοναδικός δείκτης n � 1 τέτοιος ώστε: dn > Ο � dn +ι . Από τον ορισμό της η ακολουθία dn έχει όρους ακέραιους και ισχύουν: d1 ( α0 + α1 ) - 1 · α1 α0 > Ο , =

=

=

=

=

'

=

dn+! - dn (( αο + αι + . . . + αn + αn+! ) - ( n + 1 ) αn+! ) - (( αο + αι + · · · + αn ) - nαn ) n ( αn - αn+! ) < ο. Επομένως η ακολουθία dn έχει πρώτο όρο d1 > Ο και για κάθε ισχύει ότι dn +ι < dn , δηλαδή =

=

n

είναι γνησίως φθίνουσα. Αυτό σημαίνει ότι υπάρχει κάποιος δείκτης n τέτοιος ώστε dn > Ο � dn+ι και αυτός είναι ο δείκτης που ζητάμε. Διαφορετικά θα είχαμε άπειρους διαφορετικούς ακέραιους όρους της ακολουθίας dn στο διάστημα [O , d1 ] , που είναι άτοπο. Επειδή η ακολουθία dn είναι γνησίως μονότονη ο δείκτης αυτός είναι μοναδικός. Πρόβλη μ α 2. Έστω n � 2 ακέραιος. Θεωρούμε έναν n χ n σκακιστικό πίνακα που αποτελείται από n 2 μοναδιαία τετράγωνα. Ένας σχηματισμός από n πύργους σε αυτόν τον πίνακα είναι ειρηνικός, αν κάθε γραμμή και κάθε στήλη του πίνακα περιέχει ακριβώς ένα πύργο. Βρείτε το μέγιστο ακέραιο k που είναι τέτοιος ώστε, για κάθε ειρηνικό σχηματισμό από n πύργους, υπάρχει ένα k χ k τετράγωνο το οποίο δεν περιέχει πύργο σε κανένα από τα k 2 μοναδιαία τετράγωνά του. ΕΥΚΛΕΙΔΗΣ Β' 93 τ.l/21


-------

Μαθη ματικοί Διαγωνισμοί - Μαθη ματικές Ολυμπιάδες

-------­

Λύση : Έστω f ένας θετικός ακέραιος. Θα αποδείξουμε ότι: 1 . Αν n > f 2 , τότε κάθε ειρηνικός σχηματισμός περιέχει ένα κενό f χ f τετράγωνο. 2. Αν n 5, f 2 , τότε υπάρχει ένας ειρηνικός σχηματισ ός που δεν περιέχει ένα κενό

μ

τετράγωνο. Με την απόδειξη των δύο παραπάνω προτάσεων προκύπτει ότι ο μέγιστος ακέραιος k που είναι τέτοιος ώστε, για κάθε ειρηνικό σχηματισμό από n πύργους, υπάρχει ένα k χ k τετράγωνο το οποίο δεν περιέχει πύργο σε κανένα από τα k 2 μοναδιαία τετράγωνά του είναι μικρότερος του δηλαδή ισούται με .Jn -1 όπου με .Jn -1 σημειώνουμε τον μεγαλύτερο

ι

J'

ι

J

Fn ,

i- · · · · i- · ·

fχ f

+ - + - · - · · · · -f -·� - -

· - f·f-- -+•-;- ----++··- �-+-·- .+-+··· ··!:-·-+· : .:-·- :···· Σχήμα 1 · ·

-

··

+ - + - - · - ·· - -

--

ακέραιο που είναι μικρότερος ή ίσος με .Jn - 1 . (1) Ας υποθέσουμε ότι n > f 2 • Θεωρούμε έναν ειρηνικό σχηματισμό πύργων. Τότε υπάρχει γραμμή R που περιέχει ένα πύργο στο τελευταίο προς αριστερά τετράγωνό της. Θεωρούμε f διαδοχικές γραμμές στις οποίες περιέχεται και η γραμμή R . Το σύνολο υ αυτών των γραμμών περιέχει ακριβώς f πύργους. Μετακινούμε από το σύνολο υ τις n - f 2 � 1 προς τα αριστερά στήλες, οπότε μετακινείται και ένας τουλάχιστον πύργος. Το σχήμα που απομένει είναι ορθογώνιο τύπου f 2 χ f , οπότε μπορεί να διαμεριστεί σε f τετράγωνα τύπου f χ f , στα οποία περιέχονται το πολύ f - 1 πύργοι. Επομένως ένα από αυτά τα τετράγωνα θα είναι κενό. (2) Ας υποθέσουμε ότι n 5, f 2 • Πρώτα θα κατασκευάσουμε έναν ειρηνικό σχηματισμό που δεν περιέχει κάποιο κενό f χ f τετράγωνο, για την περίπτωση με n = f 2 • Στη συνέχεια θα τροποποιήσουμε την κατασκευή αυτή έτσι ώστε να ισχύει και για μικρότερες τιμές του n . Αριθμούμε τις γραμμές από κάτω προς τα πάνω και τις στήλες από αριστερά προς τα δεξιά με τους αριθμούς Ο, 1, ... , f 2 - 1 . Κάθε τετράγωνο προσδιορίζεται από το διατεταγμένο ζεύγος ( r, c ) του αριθμού της γραμμής και της στήλης του. Στη συνέχεια τοποθετούμε τους πύργους σε όλα της τετράγωνα της μορφής { if + j, jf + ί ) , με ί, j = 0, 1, ... , f - 1 . Στο παρακάτω σχήμα φαίνεται η τοποθέτηση αυτή για n 3 . Επειδή κάθε αριθμός από Ο μέχρι f 2 - 1 έχει μοναδική αναπαράσταση της μορφής ί.e + j με Ο 5, ί, j 5, f - 1 , κάθε γραμμή και κάθε στήλη περιέχει ακριβώς ένα πύργο. Στη συνέχεια θα αποδείξουμε ότι καθένα f χ f τετράγωνο Α του πίνακα περιέχει ένα πύργο. Θεωρούμε ένα τέτοιο τετράγωνο Α και f διαδοχικές γραμμές των οποίων η ένωση περιέχει το τετράγωνο Α. Έστω ότι η τελευταία προς τα κάτω από αυτές τις γραμμές έχει αριθμό pf + q με 05,p, q5,f-1 (είναι pf + q 5, f2 - f ). Τότε οι πύργοι σε αυτή την ένωση τοποθετούνται σε στήλες με αριθμούς qf+p, ( q+1) f+p, ..., ( f-1) f+p,p+ι,e+( p+1) , ..., ( q-1).e+p+1 ή γράφοντας αυτούς τους αριθμούς σε αύξουσα σειρά p + 1, f + ( p + 1 ) , ... , ( q - 1 ) f + p + 1, qf + p, ( q + 1 ) f + p, ... , ( R - 1 ) R + p. Εύκολα διαπιστώνουμε ότι ο πρώτος αριθμός της παραπάνω λίστας είναι το πολύ ίσος με f - 1 . (Αν p = f - 1, τότε q = Ο και ο πρώτος αριθμός της λίστας είναι τότε q f + p = f - 1 ), ο τελευταίος αριθμός της λίστας είναι τουλάχιστον ( f - 1 ) R και η διαφορά μεταξύ δύο διαδοχικών αριθμών είναι το πολύ f . Έτσι μία από τις f διαδοχικές στήλες που τέμνουν το τετράγωνο Α θα έχει αριθμό κάποιον από την παραπάνω λίστα και ο πύργος σε αυτή τη στήλη θα είναι μέσα στο Α. Απομένει η κατασκευή ενός ειρηνικού σχηματισμού πύργων που δεν περιέχει ένα κενό f χ f τετράγωνο, για n < f 2 • Για το σκοπό αυτό θεωρούμε την κατασκευή που κάναμε προηγουμένως για το f 2 χ R 2 τετράγωνο από το οποίο μετακινούμε τις f 2 - n προς τα κάτω γραμμές και τις f 2 n στήλες που βρίσκονται δεξιότερα. Έτσι θα έχουμε ένα σχηματισμό πύργων χωρίς κάποιο f χ f τετράγωνο κενό, αλλά ενδέχεται να υπάρχουν μερικές γραμμές και =

-

ΕΥΚΛΕΙΔΗΣ Β' 93 τ.l/22


------ Μαθη ματικοί Διαγωνισμοί - Μαθη ματικές Ολυμπιάδες

------

στήλες κενές, δηλαδή χωρίς να περιέχουν κάποιο πύργο. Ο αριθμός των κενών γραμμών πρέπει να ισούται με τον αριθμό των κενών στηλών, οπότε μπορούμε να βρούμε μια αμφιμονοσήμαντη αντιστοιχία μεταξύ αυτών και να τοποθετούμε ένα πύργο στο τετράγωνο της τομής κάθε ζεύγους γραμμής και στήλης που είναι αντίστοιχες.

Πρόβλη μα 3. Κυρτό τετράπλευρο ABCD έχει LABC = LCDA = 90° . Το σημείο Η είναι το ίχνος της κάθετης από το Α προς τη διαγώνιο BD . Τα σημεία S και Τ βρίσκονται πάνω στις πλευρές ΑΒ και AD , αντίστοιχα, έτσι ώστε το σημείο να βρίσκεται στο εσωτερικό του τριγώνου SCT και LCHS - LCSB = 90° και LTHC - LDTC = 90° Να αποδείξετε ότι η ευθεία BD είναι εφαπτομένη του περιγεγραμμένου κύκλου του τριγώνου TSH Λύση (1 ος τ ρόπος) •.

Έστω ότι η ευθεία που περνάει από το C και είναι κάθετη προς την SC τέμνει την ΑΒ στο Q (σχήμα 2). Τότε LSg:7=9σ -LCSB=Wo - ( LCHS -w·) = 1 sσ - LCHS, οπότε προκύπτει ότι τα σημεία C, H, S και Q ανήκουν στον ίδιο κύκλο. Επειδή επιπλέον η SQ είναι διάμετρος αυτού του κύκλου, το περίκεντρο Κ του τριγώνου SHC βρίσκεται πάνω στην ευθεία ΑΒ. Ομοίως το περίκεντρο L του τριγώνου CHT Σχήμα 2 βρίσκεται πάνω στην ευθεία AD . Για να αποδείξουμε ότι ο περιγεγραμμένος κύκλος του τριγώνου SHT εφάπτεται της ευθείας BD , αρκεί να αποδείξουμε ότι οι μεσοκάθετες των τμημάτων HS και ΗΤ τέμνονται πάνω στην ευθεία ΑΗ . Όμως οι δυο αυτές μεσοκάθετες είναι και διχοτόμοι των γωνιών ΑΚΗ και t: ALH , αντίστοιχα. Επομένως από το θεώρημα της διχοτόμου στα τρίγωνα ΑΚΗ και ALH , αρκεί να αποδείξουμε ότι Σχήμα 3 ΑΚ AL ( 1 ) . 'Εστω ότι οι ευθείες ΚL και HC τέμνονται ΚΗ LH στο σημείο Μ (σχήμα 3). Επειδή είναι ΚΗ = KC και LH = LC , τα σημεία Η και C είναι συμμετρικά μεταξύ τους ως προς την ευθεία ΚL . Επομένως το σημείο Μ είναι μέσο του HC . Αν Ο είναι το περίκεντρο του τετραπλεύρου ABCD , τότε το Ο είναι το μέσο του AC , οπότε έχουμε ΟΜ 1 1 ΑΗ και επομένως ΟΜ l_ BD . Λαμβάνοντας υπόψη και την ισότητα ΟΒ = OD συμπεραίνουμε ότι η ΟΜ είναι μεσοκάθετη του BD , οπότε ΒΜ = DM. Επειδή CM l_ ΚL , τα σημεία B, C, M και Κ ανήκουν σε κύκλο διαμέτρου KC . Επομένως, με χρήση του νόμου των η μιτόνων στο τρίγωνο MDL και από το ορθογώνιο τρίγωνο CML , λαμβάνουμε DM DM DM ML DM ΒΜ = . Ομοίως, sin L.AΚL = sin LALΚ = · sin L.LDM = , · sin L.LCM = · ML ML ML CL CL CL , ΑΚ sin LALΚ DM CK CK ΚΗ οποτε - = = -- · - = - = ΚL sin L.AΚL CL ΒΜ CL LH (2°ς τρόπος) : Όπως και στον πρώτο τρόπο καταλήγουμε στην ( 1 ). Στη συνέχεια θεωρούμε δύο περιπτώσεις: (α) Αν τα σημεία Α, Η και C είναι συνευθειακά, τότε ΑΚ = AL και ΚΗ = Uf, οπότε η ( 1 ) ισχύει. ( β ) Ας υποθέσουμε ότι τα σημεία Α, Η και C δεν είναι συνευθειακά και έστω ω ο κύκλος που ορίζουν (σχήμα 4). Από το εγγράψιμο τετράπλευρο ABCD έχουμε Α

Α

'· , ·

... · -

·-·

' ' ' '

=

ΕΥΚΛΕΙΔΗΣ Β' 93 τ.l/23

·

Σχήμα 4


-------

Μαθηματικοί Διαγωνισμοί - Μαθηματικές Ολυμmάδες

------

LBA C = LBDC = 90° - LADH = LHAD. Έστω Ν * Α το σημείο τομής του κύκλου ω και της διχοτόμου της γωνίας L.CAH . Τότε η ΑΝ είναι επίσης διχοτόμος της γωνίας LBAD . Επειδή τα σημεία Η και C είναι συμμετρικά μεταξύ τους ως προς την ευθεία ΚL και ΗΝ = NC , έπεται ότι και τα δύο, το σημείο Η και το κέντρο του κύκλου ω βρίσκονται πάνω στην ευθεία KL . Επομένως ο κύκλος ω είναι ένας Απολλώνιος κύκλος των σημείων Κ και L , οπότε η ισότητα ( 1 ) προκύπτει άμεσα. Π ρόβλημ α 4 . Τα σημεία Ρ και Q βρίσκονται πάνω στην πλευρά BC του οξυγωνίου τριγώνου ABC , έτσι ώστε LPAB LBCA και LCAQ LABC . Τα σημεία Μ και Ν βρίσκονται πάνω στις ευθείες ΑΡ και AQ , αντίστοιχα, έτσι ώστε το Ρ να είναι το μέσο του ΑΜ και το Q να είναι το μέσο του ΑΝ . Να αποδείξετε ότι οι ευθείες ΒΜ και CN =

=

τέμνονται πάνω στον περιγεγραμμένο κύκλο του τριγώνου ABC Λύση (1 ° ς τ ρόπ ος) : Έστω S το σημείο τομής των ευθειών ΒΜ •

και CN . Έστω επίσης β = LQA C = LCBA και y = LPAB = LACB . Από τις παραπάνω ισότητες προκύπτει ότι τα τρίγωνα ΑΒΡ και CAQ είναι όμοια, ΒΡ ΒΡ AQ NQ = = = οπότε έχουμε: . ( 1 ). ΡΜ ΡΑ QC QC β ��'*""..;,;, � Επίσης LΒΡΜ=β+y = LCQΝ. (2). Άρα, από ( 1 ) και (2), τα τρίγωνα ΒΡΜ και NQC είναι όμοια, οπότε LBMP = LNCQ . Άρα και τα τρίγωνα ΒΜΡ και BSC είναι όμοια, αφού έχουν και τη γωνία L.A1BC κοινή. Άρα έχουμε LCSB= LBPM=β+y=1&1 -LBAC, Σχήμα 5 οπότε τα σημεία A, B, C και S είναι ομοκυκλικά. ° ς {2 τ ρόπος) : Θεωρούμε το σημείο S = ΒΜ n CN Έστω ακόμη ότι ο περιγεγραμμένος κύκλος του τριγώνου ABC τέμνει ξανά τις ευθείες ΑΡ και AQ στα σημεία Κ και L , αντίστοιχα. (σχήμα 6) Έχουμε ότι: LIBC= LLAC= LCBA και LΚCB = LΚAB = LBCA . Επομένως οι ευθείες BL και CK τέμνονται σε σημείο Χ συμμετρικό του Α ως προς την ευθεία BC . Επειδή ΑΡ = ΡΜ και Α Q = QN , έπεται ότι το Χ βρίσκεται πάνω στην ευθεία ΜΝ. Επομένως, από το θεώρημα του Pas cal στο εξάγωνο ALBSCK προκύπτει ότι το S βρίσκεται πάνω στον περιγεγραμμένο κύκλο Σχήμα 6 του τριγώνου ABC. Π ρόβλημα 5. Για κάθε θετικό ακέραw n , η Τράπεζα του Κέιπ Τάουν εκδίδει κέρματα αξίας ! . Α

__...:.,

_

c

Ν

.-------..

Ν

n

Αν δοθεί μια πεπερασμένη συλλογή τέτοιων κερμάτων (όχι απαραίτητα με διαφορετικές αξίες) με συνολική αξία το πολύ 99 + ! , να αποδείξετε ότι είναι δυνατόν να χωρίσουμε αυτή τη συλλογή σε 2

100 ή λιγότερες ομάδες, έτσι ώστε κάθε ομάδα να tι.,ει συνολική αξία το πολύ 1 . Λύση: Θα αποδείξουμε τη γενικότερη πρόταση: Για κάθε θετικό ακέραω Ν οποιαδήποτε

συλλογή

κερμάτων της Τράπεζας του Κέιπ Τάουν συνολικής αξίας το πολύ Ν _ ..!_ μπορείνα διαχωριστεί σε Ν 2 ομάδες που η καθεμία ομάδα έχει συνολική αξία κερμάτων το πολύ 1 . Η λύση του προβλήματος προκύπτει για Ν = 1 00 . Αρχίζουμε με κάποιες προπαρασκευαστικές κινήσεις. Αν κάποια από τα

κέρματα της συλλογής έχουν συνολική αξία τη μορφής ..!.. , όπου k θετικός ακέραιος, τότε θεωρούμε k όλα αυτά μαζί ως ένα καινούρω κέρμα. Αν η συλλογή που προκύπτει μετά τη διαδικασία συνένωσης κερμάτων μπορεί να διαχωριστεί κατά το ζητούμενο τρόπο, τότε το ίδιο ισχύει και για την αρχική ΕΥΚΛΕΙΔΗΣ Β' 93 τ.l/24


------- Μαθηματικοί Διαγωνισμοί - Μαθηματικές Ολυμπιάδες

-------­

συλλσγή. Μετά τη διαδικασία συνένωσης κερμάτων ο συνολικός αριθμός κερμάτων μειώνεται, οπότε σε κάποια στιγμή θα φθάσουμε σε μία κατάσταση που δεν θα είναι δυνατές άλλες συνενώσεις. Τότε για κάθε άρτιο k θα υπάρχει ακριβώς ένα κέρμα αξίας ..!.. , ενώ για κάθε περιττό ακέραιο k > 1 θα υπάρχουν k το πολύ k - 1 κέρματα αξίας ..!.. . Στη συνέχεια είναι φανερό ότι κάθε κέρμα αξίας 1 , θα πρέπει να ορίζει k μόνο του μία ομάδα. Αν υπάρχουν συνολικά d τέτοια κέρματα, τότε τα αφαφούμε από τη συλλσγή και λύνουμε το πρόβλημα για το Ν - d αντί του Ν . Έτσι μπορούμε να υποθέσουμε ότι δεν θα έχουμε κέρματα αξίας 1 . Τελικά μπορούμε να διαχωρίσουμε όλα τα κέρματα ως εξής: Για κάθε k ι, 2, . . . , Ν ι ι , ' ' ' ' η συνολικη' θετουμε ο' λα τα κερματα με αξια και , αν υπαρχουν, στην ίδια ομάδα G* . Τοτε 2k - ι 2k ι ι _ + _ < ι, k ι, 2, . . . , Ν . Επομένως, αξία της ομάδας G* είναι το πολύ V ( G* ) ( 2k - 2 ) · 2k - ι 2k έχουμε μέχρι τώρα το πολύ Ν ομάδες με κέρματα αξίας για την καθεμία το πολύ ι . Απομένει ο ι χωρισμός σε ομάδες των «μικρών» κερμάτων αξίας μικρότερης από το __ . Σε κάθε βήμα θεωρούμε 2Ν ένα μόνο τέτοιο κέρμα Επειδή η συνολική αξία των κερμάτων των μέχρι τώρα ομάδων είναι το πολύ ι ι ι ι ' ' θα υπαρχει ' ' ' ομάδα με συνολικη' αξια ' κερματων 1 Ν Ν - - , επεται οτι μια το πολ:υ' Ν 2 2 2Ν οπότε είναι δυνατόν να εισάγουμε κέρματα «μικρήφ αξίας σε αυτή την ομάδα. Με αυτό τον τρόπο μπορούμε να διαχωρίσουμε τελικά όλα τα κέρματα. Παρ ατήρηση. Ο αλγόριθμος που περιγράψαμε παραπάνω μπορεί να τροποποιηθεί στο βήμα της δημιουργίας των ομάδων G* , k ι , 2, ... , Ν. Θα μπορούσαμε σε κάθε ομάδα G* , k ι, 2, . . . , Ν να =

_

=

=

-

--

=

--,

=

=

τοποθετήσουμε όλα τα κέρματα με αξίες

( )

1

( 2k - ι ) · 2

s

,

για κάθε s Ο . �

Πρόβλημα 6. Ένα σύνολο ευθειών στο επίπεδο είναι σε γενική θέση, αν δεν υπάρχουν δύο ευθείες του παράλληλες, ούτε τρε-ις ευθείες του που περνούν από το ίδιο σημείο. Ένα σύνολο ευθειών σε γενική θέση χωρίζει το επίπεδο σε χωρία, μερικά από τα οποία tχ,ουν πεπερασμένο εμβαδό, που τα ονομάζουμε πεπερασμένα χωρία του συνόλου. Να αποδείξετε ότι για όλα τα αρκετά μεyάλα n, σε

.[;;

κάθε σύνολο n ευθειών σε γενική θέση είναι δυνατόν να χρωματίσουμε μπλε τουλάχιστον από τ-.ς ευθείες, κατά τέτοιο τρόπο ώστε κανένα από τα πεπερασμένα χωρία του να μην έχ,ει όλες τ-.ς πλευρές του χρωματισμένες μπλε. Σημείωση : Αποτελέσματα στα οποία το tχ,ει αντικατασταθεί από το c θα βαθμολογηθούν με μονάδες που θα εξαρτηθούν από την τιμή της σταθεράς c Λύση: Έστω L το δεδομένο σύνολο ε:υθειών. Θεωρούμε ένα μέγιστο ως προς τη σχέση εγκλεισμού υποσύνολο Β c L , τέτοιο ώστε όταν χρωματίσουμε όλες τις ε:υθείες του μπλε, τότε δεν υπάρχει

.[;;

.[;;

πεπερασμένο χωρίο που να έχει όλες τις πλε:υρές του μπλε. Έστω ότι I B I k . Θα αποδείξουμε πρώτα ότι =

k

rEl. όπου με k r El σημειώνουμε τον ελάχιστο ακέραιο που είναι μεγαλύτερος ή ίσος με �

m . Χρωματίζουμε όλες είναι σημείο τομής

δύο

τις

ε:υθείες του συνόλου L - Β κόκκινες. Ονομάζουμε ένα σημείο μπλε, αν

μπλε ει&ών,

οπότε

υπάρχουν συνολΙΚά

( �)

μπλε σημεία. Στη

συνέχεuχ

θεωρούμε μία κόκκινη ε:υθεία λ. Επειδή το Β είναι μέγιστο υποσύνολο του L με τη ζητούμενη ιδιότητα, υπάρχει ένα τουλάχιστον πεπερασμένο χωρίο Α του οποίου μόνο κόκκινη πλε:υρά θα ανήκει στην ΕΥΚΛΕΙΔΗΣ Β' 93 τ.l/25


------ Μαθη ματικοί Διαγωνισμοί - Μαθημαnκές Ολυμπιάδες

------

ευθεiα λ. Επειδή το χωρίο Α έχει τρεις τουλάχιστον πλευρές, αυτό πρέπει να έχει μία τουλάχιστον μπλε κορυφή. Μία τέτοια κορυφή την αντιστοιχούμε στην ευθεία λ. Επεtδή κάθε μπλε σημείο ανήκει σε τέσσερα χωρία (μερικά mθανώς μη φραγμένα), μπορεί να αντιστοιχηθεί σε τέσσερις το πολύ κόκκινες

n -k ,

οι ευθείες αυτές είναι συνολικά ·

=> k �

rFzl·

θα αποδείξουμε

Η σταθερά ότι

,

.

=

=>

οπότε θα έχουμε

ffz στη σχέση k Fz

(�)

2k ( k - I) 'Ομως n -k :::;; 2k ( k -1) n :::;; 2k2 -k < 2k2

ευθεfες. Επομένως ο σuνολuώς αριθμός των κόιcκινων ευθεΙών είναι το 100λύ 4

μπορεί να βελτιωθεί με δύο τρόπους: Πρώτα

στην προηγούμενη απόδεtξη έχουμε

ότι :

k

=

I BI �

r�2� l·

Θα κάνουμε

αντιστοιχήσεις με συγκεκριμένα βάρη ως εξής: Έστω ένα χωρίο Α του οποίου μόνο κόκκινη πλευρά ανήκει στην ευθεiα λ , έχει κορυφές, οπότε από αυτές είναι μπλε. Αντιστοιχούμε κάθε μία από αυτές τις μπλε κορυφές στην ευθεiα λ, με βάρος αντιστοίχησης Έτσι το βάρος όλων αυτών των αντιστοιχήσεων είναι συνολικά Παρατηρούμε τώρα ότι μεταξύ των τεσσάρων χωρίων που είναι γειτονικά σε μία μπλε πλευρά υ, το πολύ δύο είναι τρίγωνα. Επομένως, το άθροισμα των βαρών όλων των αντιστοιχίσεων που περtλαμβάνουν την πλευρά υ είναι το πολύ + + + 3, οπότε θα έχουμε

k

n- k ,; 3

n - k.

(�)<ο> 2n Ωk' -k 3k2 <

k-2

=>

1/(k- 2).

1 1 112 1/2

k I�2% l- Στη σονέχεια θα ωrοδεiξοuμε <:

=

όn:

k

=

Ι Β!<= I Fn l

Για το σκοπό αυτό θα αντιστοιχήσουμε σημεία στις κόκκινες ευθείες με διαφορετικό τρόπο. Ονομάζουμε ένα σημείο κόκκινο, αν αυτό βρίσκεται πάνω σε μία κόκκινη ευθεiα καθώς επίσης και πάνω σε μία μπλε ευθεiα. Θεωρούμε οποιαδήποτε κόκκινη ευθεία ε και παίρνουμε ένα πεπερασμένο χωρίο Α του οποίου μόνο κόκκινη πλευρά ανήκει στην ευθεiα ε. Έστω r', r, b1 , ••• , bk οι κορυφές του με φορά όμοια με την κίνηση των δεικτών του ωρολσyίου με r', r ε ε . Τότε τα σημεία r', r είναι κόκκινα, ενώ όλα τα σημεία b1 , b , ... ,bk είναι μπλε. Αντιστοιχούμε στην ευθεiα ε το κόκκινο σημείο r και το μπλε σημείο b1 • Σημειώνουμε ότι σε κάθε ζεύγος που αποτελείται από ένα κόκκινο σημείο r και ένα μπλε σημείο b , το πολύ μία κόκκινη ευθεiα μπορεί να αντιστοιχηθεί, αφού υπάρχει το πολύ ένα χωρίο Α που έχει τα σημεία r και b ως δύο διαδοχικές κορυφές με τη φορά των δεικτών του ωρολογίου. Ισχυριζόμαστε, ότι το πολύ δύο ευθείες αντιστοιχίζονται σε κάθε μπλε ση μείο b , το οποίο τελικά οδηγεί στο

ζητούμενο φράyμα, αφού

n- k

Ω(�) <ο> n ,; k' <ο> k

=

Ι Β!<= I Fn l

Πράγματι, αν υποθέσουμε ότι υπάρχουν τρεις κόκκινες ευθείες ε1 , ε2 και ε3 που αντιστοιχίζονται στο ίδιο μπλε σημείο b . Έστω 'i , r2 και r3 , αντίστοιχα, τα κόκκινα σημεία, διαφορετικά μεταξύ τους, που αντιστοιχίζονται σε αυτές τις ευθείες. Το ση μείο b ορίζει τέσσερις διαφορετικές μπλε ημιευθείες και κάθε σημείο 'ί είναι το πλησιέστερο κόκκινο σημείο προς το σημείο b πάνω σε μία από τις αυτές τις μπλε ημιευθείες. Επομένως μπορούμε να υποθέσουμε ότι Σχή μα 7 τα σημεία r2 και '3 στη μία μπλε ευθεiα που περνάει από το μπλε, ενώ το σημείο 'i ανήκει στην άλλη . Θεωρούμε τώρα το χωρίο Α που χρησιμοποιήσαμε για την αντιστοίχηση των σημείων 'i και b με την ευθεiα ε1 • Με τη φορά των δεικτών του ωρολογίου τρεις διαδοχικές κορυφές είναι οι 'i , b και μία από τις r2 ή r3 , έστω η r2 • Επειδή το χωρίο Α έχει μόνο μία κόκκινη πλευρά , αυτό μπορεί να είναι μόνο το τρίγωνο fίbr2 • Τότε όμως και οι δύο ευθείες ε1 και ε2 περνούν από το σημείο r2 , όπως και κάποια μπλε ευθεiα. Αυτό όμως είναι αδύνατο από τις υποθέσεις του προβλήματος. ΕΥΚΛΕΙΔΗΣ Β ' 93 τ.Ι/26


------

Μαθη ματικοί Διαγωνισμοί - Μαθη ματικές Ολυμπιάδες

-------

Λύσεις προτεινόμενων ασκήσεων τεύχους 92 a + b + c = 1 , να αποδείξετε 3 2 2 2 2 2 2 ( 2 a b 2 ) + ( 2 b c 2 ) + ( 2 c a 2 ) � ab + bc + ca c3 a - ab + b b3 c - ca + a a3 b - bc + c

Α3 1 . Για όλους τους θετικούς πραγματικούς αριθμούς a, b, c με

,

οτι:

[Τουρκία 2008, ΤΜΟ]

( -α2 J2 ;;:: (Σ (b2 -αbbc2c+2 c2 ) J ( αb + bc3 + cα ) 2 Όμως έχουμε ότι : (Σ_!_J ;;:: Σ(! (� + � - _!_) · (Σ_!_J = "( b2 + c: �bc · (Σ!J cyc d cyc α b c bc J cyc α � ab c J cyc α Πράγματι, από την ανισότητα του Σ χ ' (χ - y) (χ - z) ;;:: Ο , για = προκύπτουν οι 2 ανισότητες : Σ χ4 ;;:: Σ χ3 ( y + z ) + xyz Σ χ , ( Σχ2 J ;;:: Σ ( χ(/ + z3 ) + x2 ( y 2 + z 2 - yz )), 2 ;;:: Σ( x (y2 + z2 -;-z))· Σχ , οπότε, αν θέσουμε χ= .!. , y= .!. , z =.!. στην τελευταία ανισότητα (Σr J cyc cyc cyc α b c λαμβάνουμε την ανισότητα Επομένως για την απόδειξη της ζητούμενης ανισότητας, αρκεί να αποδε�ουμΕ ότι : (?;�J(?;ab} 3 � (?;�J (?;ab} {?;+� ?;( ab - oc)' � Ο, που ισχύει. 1

οπότε, αρκεί να αποδείξουμε ότι : Σ cyc

(1)

cyc

.

S chur

cyc

cyc

cyc

r

cyc

cyc

(2)

2,

cyc

(2).

f : JR JR που ικανοποιούν την ισότητα f(x + y) + y � !(f(f(x))), για κάθε x,y E lR . [Λευκορωσία 2013] �

Α32. Βρείτε όλες τις συναρτήσεις

'

'

Για y = Ο, η δεδομένη σχέση δίνει: f ( x) � f(f(f ( x) )) , για κάθε χ Ε IR . ( 1 ) . Επίσης για Υ = f(f ( x ) ) - x λαμβάνουμε f(x + f(f ( x)) -x) + f(f(x)) -x � f(f(f ( x))) � f(f(x)) �x, (2) για κάθε χ Ε IR . Θέτοντας, όπου χ το f (χ) στη σχέση (2), λαμβάνουμε f { f ( f (χ) )) � f (χ) , για κάθε χ Ε IR, (3) οπότε από τις σχέσεις ( 1 ) και (3) έπεται η ισότητα f(f (f ( x))) = f ( x ) , για κάθε χ Ε JR . (4). Λόγω της (4) η δεδομένη σχέση γίνεται: f ( χ + y ) + y � f ( x ) , για κάθε x, y Ε IR , (5) από την οποία για χ = Ο προκύπτει η σχέση f ( y) � α - y, για κάθε y E IR , ( 6 ) όπου α = f ( Ο) . Επίσης από τη σχέση (5) για y = -x , προκύπτει ότι : α - χ � f (χ) , για κάθ ε χ Ε JR . (7) Από τις σχέσεις ( 6 ) και ( 7 ) λαμβάνουμε f (χ) = α - χ, χ Ε JR Εύκολα επαλη θ εύουμε ότι η συνάρτηση f (χ) = α - χ, χ Ε JR , ικανοποιεί τη συνθή κη του προβλή ματος, για κάθε τιμή της σταθεράς α Ε JR . Λύ ση :

ΕΥΚΛΕΙΔΗΣ Β' 93 τ.l/27


------

Α33.

Μαθη ματικοί Διαγωνισμοί - Μαθηματικές Ολυμπιάδες

-------­

Σε κανονικό δεκατετράγωνο Α 1 Α 2 Α 1 4 εγγεγραμμένο σε κύκλο κέντρου Ο, έστω Κ, Λ και Μ τα σημεία τομής τη ς ευθείας ΑιΟΑ8 με τις ευθείες Α Α4 , Α4Α και Α 6 Α 0 αντίστοιχα.

2

Να αποδειχθεί ότι, αν

{οκ = κ · ΟΑ1, 0Λ = λ ·ΟΑ1, 0Μ = μ ·ΟΑ1},

κ, λ και μ είναι ρίζες της εξίσωσης: 8χ3 - 4χ2

Λί>ση :

1,

•••

-

4χ + 1 = Ο.

2

( 1 ) τότε οι αριθμοί

(2)

[Γιάννης Περδικάρης]

Οι αριθμοί κ, λ και μ είναι ρίζες της εξίσωσης: 8χ3 -4χ2 - 4χ + 1 = Ο, αν, και μόνον αν,

180' �7ω=180', ' κ+ λ+ μ =-,1 (3) κλ+λμ+μκ = --,1 (4) κλμ = - -1 , (5)} . Αν θεσουμε ω=7 2 2 8 { τότε έχουμε Α10Α = ω, Α10Α4 = 3ω, Α10Α6 = 5ω, οπότε τα δεδομένα στη σχέση (1) γίνονται : 2 μ = συv5ω , ενώ οι aποδεικτέες σχέσεις γράφονται ισοδύναμα στη κ = συvω, λ = συv3ω, μορφή συνω+ συν3ω + συv5ω = _!_ (3') , συvωσυv3ω+ συν3ωσυv5ω+συν5ωσυνω = - _!_ (4'), 2 2 συvωσυv3ωσυv5ω = _.!_(5') 8 . Επειδή 7ω = 1 80° , έπεται ότι : συv6ω = -συvω, συv5ω = -συv2ω, συv4ω = -συv3ω, ημ6ω = ημω, ημ5ω = ημ2ω, ημ4ω = ημ3ω, οπότε, αφού ημω * Ο , έχουμε: (3') � 2ημωσυvω + 2ημωσυv3ω + 2ημωσυv5ω = ημω � ημ2ω + ( ημ4ω -ημ2ω ) + ( ημ6ω - ημ4ω) = ημω � ημ6ω = ημω, που ισχύει. (4') � 2συvωσυv3ω+ 2συv3ωσυv5ω+ 2συv5ωσυvω = -1 �( συv4ω+συν2ω)+( συv6ω+συν4ω) +( συv8ω+συν2ω) =-1 �2(συv4ω+συν2ω) +συν6ω+συν8ω= -1 �-2 ( συv3ω+συν5ω) -συνω+συν8ω= -1 �2 (συvω- �) -συvω+συν8ω=-1� συv8ω= -συνω, ισχύει γιατί 8ω= 7ω+ω= 180" +ω. (5') � 8ημωσυvωσυv3ωσυv5ω = -ημω � 4ημ2ωσυv3ω( -συ v2ω) = -ημω � 2ημ4ωσυv3ω= ημω � 2ημ3ωσυv3ω = ημω � ημ6ω= ημω, ισχύει. που

που

,

Ν2 1 . (α) Βρείτε όλους τους πρώτους αριθμούς p, για τους οποιους ,

, , ο' λους τους πρωτους τετραγωνο ακεραιου. αριθ μους , , ( β ) Β ρειτε

Ρ

,

,

ειναι τελειο

, p, για τους οποιους

είναι τέλειο τετράγωνο ακεραίου. Λύση :

7p- l - 1

ι ιp- ι - ι Ρ

---

[Τουρκία 2008, ΤΜΟ]

q pΓια τον τυχόντα περιττό θετικό ακέραιο q έχουμε : 1 - 1 = χ 2 , χ Ε Ζ � Ρ

� ( q (p-Ι)ι2 - 1 )( q (p- ι)ι2 + 1 ) = px2 , χ Ε Ζ . (1) Επειδή οι αριθμοί q(p-Ι)ι2 - 1, q (p-Ι)ι2 + 1 είναι διαδοχικοί άρτιοι ακέραιοι, έπεται ότι: ( q (p-Ι )ι2 - 1, q (p- ι )ι2 + 1 ) = 2 , οπότε από την ( 1 ), δεδομένου ότι οι δύο αυτοί παράγοντες δεν έχουν κοινό διαιρέτη μεγαλύτερο του 2 προκύπτουν δύο δυνατές περιπτώσεις: Ι. q (p-ι )ι2 - 1 = 2 py2 και q (p- Ι)ι2 + 1 = 2z2, y, z Ε Ζ . Π . q (p-Ι)ι2 - 1 = 2y 2 και q (p-Ι)ι2 + 1 = 2 pz2, y, z Ε Ζ . (α) Έστω q = 7 . Επειδή το 2 είναι ένα τετραγωνικό υπόλοιπο mod7, ενώ το -1 δεν είναι, η περίπτωση 11 είναι αδύνατη. Από την περίπτωση Ι έχουμε ότι: 6= (7 -1) 1{ 7>-ι)ι2 -1) =2w2 => 3 j PI (2) 72 - 1 = 16 = 42 , οποτε ' ο πρωτος ' αριθ μος ' p = 3 ειναι λυση. ' ' Αν p = 3 , τοτε 3 •

'

ΕΥΚΛΕΙΔΗΣ Β' 93 τ.l/28


------ Μαθη ματικοί Διαγωνισμοί - Μαθη ματικές Ολυμπιάδ ες •

1

1

·,

------

Αν p :;ι: 3 , τότε από τη (2) προκύπτει ότι: 3 / => 3 j y � 9 2 p/ = 7 (p- Ι )ι2 - 1 (3)

Από τη σχέση (3) προκύπτει ότι: 3

(Ρ; 1 ) . Αν υποθέσουμε ότι Ρ; 1 = 3k, τότε

2z2 = 73k + 1 = ( 7k + 1 )( 72k - 7k + 1) έπεται ότι ο 72k - 7k + 1 είναι τέλειο τετράγωνο, που είναι άτοπο, αφού ( 7k - 1 ) 2 < 72k - 7k + 1 < ( 7k ) 2 • ( β ) Έστω q = 1 1 . Επειδή το 2 δεν είναι τετραγωνικό υπόλοιπο mod1 1, η περίπτωση Ι είναι αδύνατη. Από την περίπτωση 11 έχουμε: 1 1 (p-Ι)ι2 + 1 = 2pz 2 => 1 1 (p-Ι)ι2 -1( mod p) => =>2/ =ll(p-l)/2 -1 -2 ( ιmdp) =>/ -1(ιmdp) => p = 1 ( ιmd4) . Επομένως, από τη σχέση 1 1 (p- Ι )ι2 - 1 = 2 y 2 , έπεται ότι ( 1 1 (p-Ι)ι4 - 1 ) ( 1 1 (p-Ι)ι4 + 1 ) = 2/ , από την από τη σχέση

=

=

=

οποία προκύπτουν δύο δυνατές περιπτώσεις: 1 . 1 1 (p-Ι)ι4 + 1 = u 2 � 1 1 (p-Ι)ι 4 = ( u -1 )( u + 1) , αδύνατη αφού οι u - 1, u + 1 δεν είναι δυνατόν να είναι και οι δύο δυνάμεις του 1 1 . 2. 1 1 (p-Ι)ι4 + 1 = 2 u 2 , αδύνατη, αφού το 2 δεν είναι τετραγωνικό υπόλοιπο mod1 1 . Επομένως, για q = 1 1 , δεν υπάρχει λύση. Ν22. Να προσδιορίσετε τους πρώτους αριθμούς p,q που ικανοποιούν την εξίσωση : q3

=

p2 - p + 1 .

[Λευκορωσία 2013]

q 3 = p 2 - p + 1 � q3 - 1 = p 2 - p � ( q - 1)( q2 + q + 1) = p(p - 1) (1) 2 Άρα ο πρώτος αριθμός p διαιρεί το γινόμενο ( q-1)(q + q+1) , οπότε: p l (q -1) ή pl(q2 + q +1) . Αν p j( q - 1), τότε q � p + 1 => q3 � (p + 1γ > p2 - p + 1, άτοπο. q2 + q + 1 = kp, k ε Ν* . Αν p l ( q2 + q + 1) , τότε: (2) Επομένως, από τη σχέση ( 1 ) προκύπτει ότι: ( q-1) /φ = p ( p-1) � ( q-1)k = p-1 �p = kq -k+1, q2 + (1 - k 2 ) q + (k 2 - k + 1) = 0 (3) Επομένως η διακρίνουσα του οπότε η σχέση (2 ) γίνεται: τριωνύμου ως προς q που δίνεται στη σχέση (3) : Δ = (1-k2 )2 -4(k2 -k+1) = k4 -6k2 +4k-3 πρέπει να είναι τέλειο τετράγωνο. 'Ομως ισχύει ότι: ( k 2 - 3 ) 2 < Δ = k4 - 6k2 + 4k - 3 < ( k 2 - 1 ) 2 , για κάθε k > 3 , ενώ η εξίσωση k4 - 6k 2 + 4k - 3 = (k 2 - 2 ) 2 � 2k 2 -4k + 7 = Ο , δεν έχει λύση στο IR. . Επομένως πρέπει k :::; 3. Για k = 1, είναι Δ = -4 < Ο, ενώ για k = 2 , είναι Δ = -3 < Ο . Για k = 3 , έχουμε Δ = 36 = 62 , οπότε προκύπτουν οι λύσεις q = 7 ή q = 1, από τις οποίες δεκτή είναι η λύση q = 7 , οπότε p = kq - k + 1 = 1 9 . Εύκολα διαπιστώνουμε ότι το ζευγάρι (p, q) = (1 9, 7 ) είναι λύση του προβλήματος. Λύση : Έχουμε

Ν23. Βρείτε όλες τις τριάδες

εξίσωσης: 7χ = 3z - 2Υ. Λύση : Η δεδομένη εξίσωση

(x, y , z )

μη αρνητικών ακέραιων που είναι λύσεις της [Λευκορωσία 2013]

γράφεται: Τ + 2Υ = 3Ό (1) Για y Ο, το αριστερό μέλος της εξίσωσης δίνει άρτιο ακέραιο, ενώ το δεύτερο μέλος δίνει περιττό. Άρα πρέπει y > Ο . Στη συνέχεια θα θεωρήσουμε τις περιπτώσεις: y 1, y = 2 και y � 3 . Για y = 1 , έχουμε την εξίσωση: Τ + 2 = 3z ('2) Για χ = Ο , προκύπτει z Ι και η λύση ( x, y, z) = (0, 1, 1) . =

=

=

ΕΥΚΛΕΙΔΗΣ Β' 93 τ.l/29


------- Μαθη ματικοί Διαγωνισμο ί - Μαθη ματικές Ολυμπιάδ ες -----• •

Για χ = 1, προκύπτει z = 2 και η λύση ( � , y, z) = (1, I, 2) . 'Εστω χ > 2 και z > 2. Τότε 33 = 27 1( γ + 2) . Θεωρώντας τα υπόλοιπα της διαίρεσης του

γ moduio 27 έχουμε τη διαδοχή υπολοίπων: 7 � -5 �-8 �-2 �13 �10 �-1 1 �4 �1 , από την οποία προκύπτει ότι χ 4( mod9) . Επιπλέον παρατηρούμε ότι 76 + 73 + 1 = 1 17993 = 3 37 · 1 063, οπότε 371( 76 + 73 + 1) 37179 - 1 79 1( mod37) ::::::> γ 74 492 12 2 144 33( mod37) ::::::> ::::::> γ + 2 35( mod37). Θεωρώντας και τα υπόλοιπα της διαίρεσης του 3z modulo 37, έχουμε: 3 � 9 � 27 � 7 � 21 � 26 � 4 � 12 � 36 � 34� 28 � 10 � 30 � 16 � 1 1 � 33 � 25 � 1, οπότε παρατηρούμε ότι: 3z ;1. 35 ( mod37) . Επομένως η εξίσωση (2) δεν έχει λύσεις για χ > 2 και z > 2 . 'Εστω y = 2 . Τότε 3z = γ + 4 2 ( mod3) , άτοπο. Έστω y 2:: 3 . Τότε από την εξίσωση γ - 3z = 2Υ προκύπτει ότι γ 3z ( mod 8) , οπότε πρέπει οι χ και z να είναι πολλαπλάσια του 2, έστω χ = 2χ1 και z = 2z1 • Τότε η δεδομένη εξίσωση 3zι _ γι = 2a (3) b > α 2:: 1 (αφού 2 1 ( 3zι - 7 χι )) . γίνεται : (3zι - 7 χι )(3zι + γι ) = 2 Υ 3zι + γι = 2b (4) Με πρόσθεση κατά μέλη των (3) και (4) λαμβάνουμε: 2a + 2 b = 2 3zι ::::::> α = 1, αφού διαφορετικά θα είχαμε ότι 412 3zι , άτοπο. Τότε η (3) γράφεται: 3zι - γι = 2 3zι = γι + 2, η οποία είναι ισοδύναμη με την εξίσωση (2). Άρα έχουμε { ( χ1 , z1 )} { (Ο, 1) , ( 1, 2)} , οπότε λαμβάνουμε b = 2 και b = 4, αντίστοιχα. Έτσι έχουμε τις λύσεις: { (χ, y, z)} = { (Ο, 3, 2) , ( 2, 5, 4) } . Ξ

·

::::::>

::::::>

Ξ

Ξ

Ξ

Ξ

Ξ

Ξ

Ξ

Ξ

<::::>

{

}

Ξ

,

·

<::::>

·

=

Ν24. Ένα θετικό ακέραιο Ν τον ονομάζουμε «ισορροπημένο 2ης τάξης», αν ο αριθμός των ψηφίων του είναι διπλάσιος του αριθμού των διαφορετικών πρώτων παραγόντων του στην 2 ανάλυσή του. Για παράδειγμα, οι αριθμοί 16 = 2 4 , 49 = 7 , κανονική

1 225 = 5 2 7 2 , 810000 = 2 4 3 4 5 4 , κάθε διψήφιος πρώτος, κλπ, είναι «ισορροπημένοι «θετικοί ακέραιοι, ενώ οι αριθμοί 21 = 3 7, 1024 = 2 10 , κάθε μη διψήφιος πρώτος, κάθε ακέραιος με •

·

·

περιττό πλήθος ψηφίων κλπ., δεν είναι. Να αποδειχθεί ότι υπάρχει πεπερασμένο πλήθος [Γιάννης Περ δικάρης] «ισορροπημένων 2ης τάξης» θετικών ακέραιων. Λύση : Έστω αριθμός των διαφορετικών πρώτων παραγόντων του θετικού ακέραιου Ν στην

ko

κανονική ανάλυσή του. Τότε: Ν 2:: Γ k = p1p2 pk (1) και 2k είναι ο αριθμός των ψηφίων του 2k Ν , οπότε 1 0 > Ν <::::> 100 k > Ν . (2). Οι μικρότεροι του 100 πρώτοι είναι συνολικά 25: { 2,3, 5, 7, ... , 89,97 } , ενώ ο αμέσως μεγαλύτερος του 100 πρώτος είναι ο 101, οπότε για k > 27 , Γ 25Γ 25 1 Ο Ι k <=> -Γ 25 Ρ k - Γk k > z5 . ( Ι + Ο ' 01) k (3) εχουμε.. Γ k - Γ 26 Ρ27 Pk > ---τs26 1012 5 100 101 Ρ26 Στη συνέχεια θα χρησιμοποιήσουμε το παρακάτω λήμμα: Λή μμα. Για οποιουσδήποτε θετικούς πραγματικούς αριθμούς α, β, μ, ( μ > α) , υπάρχει Μ > Ο τέτοιος ώστε να ισχύει α ( 1 + β Υ > μ , για κάθε χ > Μ . ln μ - ln α ) . , ( ια την απο' δειξη, αρκει, να παρουμε Μ= · · ·

·

· · ·

r 1

Αν θέσουμε α = ..!1L > Ο και 1 0 1 25

·

·

ln ( l + β )

β = Ο, Ο 1 > Ο, από

θετικός Μ τέτοιος ώστε, για κάθε k > Μ , να ισχύει

το παραπάνω λήμμα έπεται ότι υπάρχει Ι

���5 ( Ι + Ο, Ο 1 γ > 1 , •

ΕΥΚΛΕΙΔΗΣ Β' 93 τ.l/30


-------

�Ιαθηματικοί Διαγωνισμοί - Μαθη ματικές Ολυμπιάδες

από την οποία προκύπτει όn:

Γk k

-------­

k k > 1 � Γk > 1 OO � Ν > 1 OO , (4) που αντίκειται στη σχέση (2).

1 00 Επομένως, δεν υπάρχουν ισορροπημένοι αριθμοί δεύτερης τάξης με περισσότερα από 2k>2M ψηφία, οπότε υπάρχει πεπερασμένο πλήθος τέτοιων αριθμών.

Ν25. Ένα θετικό ακέραιο Ν τον ονομάζουμε «ισορροπημένο v - τάξης», όπου v δεδομένος θετικός ακέραιος, αν ο αριθμός των ψηφίων του είναι v - πλάσιος του αριθμού των διαφορετικών πρώτων παραγόντων του στην κανονική ανάλυσή του. Για παράδειγμα, οι αριθμοί 49 = 7 2 και 67 (πρώτος) είναι «ισορροπημένοι 2 ης τάξης», ο αριθμός 1 024 = 2 10 είναι ισορροπημένος 4ης τάξης, ενώ οι αριθμοί 30 = 2 · 3 · 5 και 143 = 1 1 · 13 δεν είναι ισορροπημένοι καμιάς τάξης. Να αποδειχθεί ότι υπάρχει πεπερασμένο πλήθος «ισορροπημένων v - τάξης» [Γιάννης Περ δικάρης] θετικών ακέραιων, για κάθε θετικό ακέραιο v. Λύση : Η απόδειξη είναι όμοια με αυτή της προηγούμενης άσκησης, θεωρώντας όπου 2k το vk . Γ2 1 . Έστω Μ, Ν τα μέσα των πλευρών K, L

AB,BC ,

ABC . Έστω ακόμη ABC με τις πλευρές

αντίστοιχα, τριγώνου

τα σημεία επαφής του εγγεγραμμένου κύκλου του τριγώνου AB, AC αντίστοιχα. Αν οι ευθείες ΜΝ και ΚL τέμνονται στο σημείο Τ, να αποδείξετε ότι

αυτό ανήκει στη- διχοτόμο της γωνίας

ACB .

[Λευκορωσία 2013]

Λύση : Έστω Ρ το σημείο επαφής του εγγεγραμμένου κύκλου του τριγώνου ABC με την πλευρά BC και έστω χ = Β Κ = ΒΡ, y = ΑΚ = AL, z = CP = CL. Α Αν BC = AC , τότε το ζητούμενο ισχύει, αφού τα σημεία Μ, Κ, Τ ταυτίζονται και η CT είναι διχοτόμος της γωνίας ACB. Έστω AC > BC (σχήμα 1 ). Επειδή Μ, Ν είναι μέσα των πλευρών ΑΒ, BC , αντίστοιχα, έπεται ότι: ΜΝ 1 1 AC, οπότε ΚΤΜ = κiΑ. . Όμως ΑΚL = κiΑ , οπότε ΑΚ = AL, και ΜΚΤ = AKL = ΚΤΜ . Επομένως έχουμε: ·, ._ ·� - - ����-�c Β 1 1 1 ΜΤ = ΜΚ = 2 · ΑΒ - Β Κ = ( χ + y) - x = 2 ( y - x ) , 2 1 1 1 Α ΤΝ = ΜΝ - ΜΤ = 2 ( y + z) - 2 ( y - x ) = 2 (z + x ) = CN. Άρα το τρίγωνο TCN είναι ισοσκελές με ΝΓC=ΝCΓ. Επειδή ΜΝ Ι Ι ΑC, έχουμε ότι Nfc=ΊtA . Από τις δύο τελευταίες ισότητες γωνιών έπεται ότι NCT = TCA, οπότε η CT είναι διχοτόμος της γωνίας ACB. χ

Λ

\

Λ

Λ

'

Ανάλογα εργαζόμαστε και στην περίπτωση που είναι AC < BC (σχήμα 2).

B �--

�.. Ρ � Ν

·,· ... . ... .

χ

---�c z

--

---

Σχήμα

5

Γ22. Έστω τρίγωνο ABC με BAC = 60° Να αποδείξετε ότι το κέντρο Ε του κύκλου Euler του τριγώνου βρίσκεται στην εσωτερική διχοτόμο της γωνίας BAC. [Γιά ννης Περδικάρης] Λύση

Έστω Ο το περίκεντρο, Η το ορθόκεντρο και Μ το μέσο της πλευράς BC του τριγώνου ABC. Επειδή BAC = 60° έπεται ότι ΒΟΜ = 60° και αφού ΒΜΟ = 90° έπεται ότι ΟΒ = 2 · 0Μ�ΟΑ = 2 · 0Μ . ΕΥΚΛΕΙΔΗΣ Β' 93 τ.Ι/31


------- Μαθη ματικοί Διαγωνισμοί - Μαθη ματικές Ολυμπιάδες

Όμως είναι γνωστό ότι και ΑΗ 2 ΟΜ, οπότε ΑΗ = ΑΟ , οπότε το σημείο Ε μέσο του ευθύγραμμου τμήματος ΟΗ βρίσκεται στην εσωτερική διχοτόμο της γωνίας ΗΑΟ . Όμως είναι επίσης γνωστό ότι σε οξυγώνιο τρίγωνο οι γωνίες ΗΑΟ και BAC έχουν κοινή εσωτερική διχοτόμο, οπότε το σημείο Ε βρίσκεται στην εσωτερική διχοτόμο της γωνίας BAC. =

·

-------­

Α

Σχήμα 3 Γ23. Έστω τρίγωνο ABC με BAC = 120· Να αποδείξετε ότι το κέντρο Ε του κύκλου Euler του τριγώνου βρίσκεται στην εξωτερική διχοτόμο της γωνίας BAC. [Γιάννης Περδικάρης]

Έστω Ο το περίκεντρο, Η το ορθόκεντρο και Μ το μέσο της πλευράς BC του τριγώνου ABC. Επειδή BAC 120" έπεται ότι Β όΜ = 180 - 120" = 60. και αφού ΒΜΟ = 90· έπεται ότι ΟΒ = 2 ΟΜ => ΟΑ = 2 ΟΜ . Όμως είναι γνωστό ότι και ΑΗ 2 ΟΜ, οπότε ΑΗ = ΑΟ , οπότε το σημείο Ε μέσο του ευθύγραμμου τμήματος ΟΗ βρίσκεται στην εσωτερική διχοτόμο της γωνίας ΗΑΟ . Όμως είναι επίσης γνωστό ότι σε αμβλυγώνιο τρίγωνο στο Α, η εσωτερική διχοτόμος της γωνίας ΗΑΟ είναι εξωτερική διχοτόμος της γωνίας BAC , οπότε το σημείο Ε βρίσκεται στην εξωτερική διχοτόμο της γωνίας BAC. Λύση :

=

Η

·

=

·

·

Σχήμα 4

Ση μείωση. Λόγω έλλειψης χώρου, οι λύσεις των ασκήσεων Γ24, Γ25, Γ26, Δ12, Δ13 και Δ14 του προηγούμενου τεύχους θα δοθούν στο επόμενο τεύχος. ΑΣΚΗΣΕΙΣ ΓΙΑ ΛΥΣΗ

Α33. Έστω x, y και

z

, . χ Υ z Υ z Π' , , οπου m = mιn - + - + - , - + - + -x . οτε ισχυει η ισοτητα; Υ z χ χ Υ z

(

(

J

θετικοί πραγματικοί αριθμοί. Να αποδείξετε ότι: ( x +y+z) � +_! +� � ,J , χ Υ z

J

Έστω m � 3 και n θετικοί ακέραιοι τέτοιοι ώστε n > m ( m - 2 ) . Βρείτε το μεγαλύτερο δυνατό ακέραιο d που είναι τέτοιος ώστε d Jn ! και k jd , για κάθε k ε {m, m + l , ... , n} . Ν26.

Έστω .e ( n) είναι ο μεγαλύτερος περιττός διαιρέτης του φυσικού αριθμού n . Να υπολογίσετε το άθροισμα: .e (1) + .e ( 2 ) + .e (3) + . . . + .e( 2 2ο ι 3 ) . Γ27. Έστω τρίγωνο ABC και Ι το έκκεντρό του. Δίνεται ότι υπάρχει κύκλος k που περνάει από το Β και εφάπτεται της ΑΙ στο Ι . Ο κύκλος k τέμνει την ευθεία ΑΒ και στο σημείο Ρ , και επίσης την ευθεία BC και στο σημείο Q . Η ευθεία QI τέμνει την AC στο σημείο R . Να αποδείξετε ότι: AR BQ ΡΙ 2 • Ν27.

·

=

ΕΥΚΛΕΙΔΗΣ Β' 93 τ.l/32


ΗΟΜΟ MA THEMATICUS Η Homo Mathematicus είναι μια στήλη στο περιοδικό μας, με σκοπό την ανταλλαγή απόψεων και την ανάπτυξη προβληματισμού πάνω στα εξής θέματα: 1 ) Τι είναι τα Μαθηματικά, 2) Πρέπει ή όχι να διδάσκονται, 3) Ποιοι είναι

οι κλάδοι των Μαθηματικών και ποιο το αντικείμενο του καθενός, 4) Ποιες είναι οι εφαρμογές τους, 5) Ποιες επιστήμες ή κλάδοι επιστημών απαιτούν καλή γνώση των Μαθηματικών για να μπορέσει κάποιος να τους σπουδάσει.

επιμέλεια: Γιάννης Κεpασαρίδης Αμαλία Μανιατοπούλου

Ι. "Τι είναι τα Μαθηματικά; "

Συνεχίζουμε τα περί συστήματος συντεταγμένων του μεγάλου Albert Einstein. Μ' αυτόν τον γίγαντα της επιστήμης, ασχοληθήκαμε και στα τεύχη 85,86,9 1 . Στο παρόν τεύχος ολοκληρώνουμε αυτή την αναφορά μας. «Από τα παραπάνω προκύπτει ότι θα είναι προτιμότερο, όταν τούτο είναι μπορετό, να μεταχειριζόμαστε "αριθμούς μέτρησης" για να προσδιορίσουμε κάποιο τόπο. Θα μπορούσαμε έτσι να προσδιορίζουμε αυτόν ανεξάρτητα από ονοματισμένα σημεία που ενδέχεται να βρίσκονται πάνω στο στερεό, το οποίο χρησιμοποιούμε σαν σύστημα αναγωγής. Αυτό επιτυγχάνει η Φυσική με τις καρτεσιανές συντεταγμένες. Αυτό το σύστημα συντεταγμένων αποτελείται από τρία επίπεδα, ανά δύο κάθετα μεταξύ τους και είναι συνδεδεμένα με κάποιο στερεό. Για να προσδιορίσουμε τον τόπο οποιουδήποτε γεγονότος, σε σχέση προς αυτό το σύστημα συντεταγμένων, θα πρέπει να προσδιορίσουμε τα μήκη των τριών καθέτων ή συντεταγμένων (x,y,z), οι οποίες μπορούν να aχθούν Τις τρεις αυτές συντεταγμένες μπορούμε

να μετρήσουμε με ραβδίσκους ακρίβειας, σύμ φ ωνα με τους νόμους τους οποίους διδάσκει η Ευκλείδεια Γεωμετρία. Στην πράξη, γενικά, δεν πραγματοποιούνται τα τρία στερεά επίπεδα που αποτελούν το σύστημα συντεταγμένων, πολύ λιγότερο μετριούνται οι συντεταγμένες με ραβδίσκους, αλλά καθορίζονται εμμέσως. Δεν πρέπει εντούτοις ποτέ να λησμονούμε αυτή τη φ υσική σημασία του ορισμού του τόπου "επί ποινή " συσκότισης των εξαγομένων της Αστρονομίας και της Φυσικής. Αυτό συνεπάγεται ότι: Κάθε προσδιορισμός της

Προλεγόμενα

θέσης κάποιου γεγονότος στο χώρο απαιτεί τη χρήση κάποιου στερεού σε σχέση με το οποίο

καθορίζεται η θέση του γεγονότος. Ο καθορισμός

αυτός προϋποθέτει ότι οι νόμοι της Ευκλείδειας Γεωμετρίας είναι εφαρμόσιμοι για τις "ευθείες ", δηλ. ότι η "ευθεία '' μπορεί να παρασταθεί φυσικά με δύο σημεία, τα οποία σημειώθηκαν» [ τα

πλάγια και bold πρωτοβουλία]

//. "Ευκλείδεια Γεωμετρία, αγάπη μου ,, ισογώνιο κέντρο τριγώνου Δ

' α1

περιβάλλουσα καμπύλη

γράμματα

είναι

δική

μας

Ψευδοτετράγωνο Α

ι

Γ

σ

α3

4ΒΓ του τυχαίου τριγώνου (σχήμα 1). Το σημείο εξωτερικώς τομής των περιγεγραμμένων περι φερειών, εις τα κατασκευασμένα ισόπλευρα τρίγωνα, διέρχονται από •

ισογώνιο κέντρο τριγώνου

ΕΥΚΛΕΙΔΗΣ Β' 93 τ.l/33


-------

HOMO MATHEMAτiCUS

--------­

έδωσε πολλές λύσεις του προβλήματος (Mathesis, 1 859) Το σημείο Μ έχει την ιδιότητα: το άθροισμα των Στο τεύχος 83, σημειώσαμε δύο σχόλια για την aποστάσεών του από τις κορυφές του ΑΒΓ, να ύπαρξη σημείου Μ . είναι ελάχιστο . Η �νεύρεση αυτού του σημείου είχε προταθεί από τον Fermat στον Toπicelli, ο οποίος το ίδιο σημείο Μ το οποίο ονομάζουμε ισογώνιο σημείο(ή κέντρο) του ΑΒΓ.

(σχήμα 2). ονομάζουμε περ ιβάλλουσα καμπύλη ή απλά περιβάλλουσα μιας κινητής ευθείας, την καμπύλη η οποία εφ άπτεται στην κινητή ευθεία, σε κάθε θέση που είναι δυνατό να καταλάβει η κινητή ευθεία. •

περιβάλλουσα καμπύλη

ψευδοτετράγωνο

Η περιβάλλουσα των ευθειών που απέχουν δοσμένη απόσταση d από δοσμένο σημείο Ο είναι η περι φέρεια του κύκλου που έχει σαν κέντρο το δοσμένο σημείο και σαν ακτίνα τη δοσμένη απόσταση. παράδειγμα

(σχήμα 3). ονομάζεται το τετράπλευρο του οποίου οι διαγώνιες είναι κάθετες και ίσες

ΠΙ. «Αυτό το ξέρατε;>>

α) τι έγραφε, το 1 825, από το Βερολίνο ο Niels Abel προς τον καθηγητή του Κρίστοφερ Χάντσιν; β) ποιος είπε την περίφημη φράση: «Hic ego barbarus sum quia non intelligor illis>> [«Είμαι βάρβαρος γι' αυτούς, επειδή δεν με καταλαβαίνουν>>]

IV. "Α νοιχτά προβλήματα στα Μαθηματικά "

Τη στιγμή που ετοιμάζαμε την ύλη της ενότητας αυτής, ξαφνικά (στις 1 4/4/201 4) λάβαμε ένα τηλεφώνημα, από τον παλιό φίλο της στήλης Ιπποκράτη Παναγ. Δάκογλου, το οποίο μας ξάφνιασε και ταυτόχρονα μας γέμισε από ευγενική περιέργεια, σχετικά με τη βασιμότητα ενός ισχυρισμού: ο I. Δάκογλου μας πληροφορούσε πως την ίδια μέρα κατέθεσε στην Ακαδημία των Αθηνών τα απαιτούμενα αντίτυπα μιας δίγλωσσης (Ελληνικά Αγγλικά), εργασίας του με τίτλο «Μαθηματική Απόδειξη και Γενίκευση των Εικασιών του Christian Goldbach>>, ενώ ταυτόχρονα μας διαβίβαζε ένα αντίτυπο

της εργασίας του. Εμείς γνωρίζαμε από παλιά πως ο I . Δάκογλου είναι ειδικός σε θέματα Πυθαγόρειας Φιλοσοφίας, ότι έχει γράψει τετράτομο έργο για τα έργα και τις απόψεις της Πυθαγόρειας Σχολής, ότι, τέλος, ασχολείται από χρόνια, με την απάντηση στις εικασίες του C. Goldbach, από την οπτική των Πυθαγορείων. Εμείς θα τη μελετήσουμε αυτή την εργασία. Όμως, όντας aμοιροι των βαθύτερων απόψεων της Πυθαγ. Σχολής, δεν έχουμε πολλά περιθώρια να την κατανοήσουμε. Όσοι "πιστοί" προσέλθετε.

V. "Οι συνεργάτες της στήλης γράφουν-ερωτούν "

]0 θέμα ((δύο ερωτήματα πάνω στην έλλειψη» Παναγ. Οικονομάκος

ο φίλος της στήλης Παναγιώτης Οικονομάκος μας έστειλε, όπως το συνηθίζει, ένα θέμα που απαντά στο ερώτημα προσδιορισμού σημείου έλλειψης και της εφαπτομένης σ' αυτό, πριν κατασκευάσουμε την έλλειψη . π ρολεγ όμενα

εκφ ώνηση

α) να κατασκευαστεί σημείο έλλειψης και η α) κατασκευή σημείου και εφαπτοJίένης εφαπτομένη ευθεία της στο σημείο αυτό, με κανόνα [σχήμα (α)) Με κέντρο το σημείο F2 και ακτίνμ ίση με 2α και �ιαβήτη . ' β) ν αποδειχθεί η βασική ιδιότητα των εστιών γράφουμε κύκλο K(F2 , 2α) και έσ�ω Μ τυχαίο σημείο αυτού. Αν (ε) η μεσοκάθετη του τμήματος έλλειψης, χωρίς τη χρήση αναλυτικών μεθόδων F1M και Ρ σημείο τομής αυτής με την ακτίνα F2M, ο ρισμός γματικός τότε το Ρ είναι σημείο της έλλειψης και η (ε) είναι Δίνόνται δύο σημεία F i ,F2 και ένας πρα αριθμός α>Ο. Το σύνολο όλων των σημείων Ρ ενός εφαπτομένη αυτής στο σημείο Ρ . επιπέδου (που περιέχει την ευθεία F ι F2) των (i) απόδειξη (για το σημείο) οποίων το άθροισμα των αποστάσεων από τα Επειδή F2M είναι ακτίνα του κύκλου K(F2 , 2α) θα (1) Ρ F2+MP= F2Μ=2α σημεία F1 και F2 είναι σταθερό και ίσο με 2α, έχουμε ονομάζεται έλλειψη με εστίες F1,F2 και μήκος και επειδή η (ε) είναι μεσοκάθετη στο τμήμα F1M, ΜΡ= F 1Ρ (2) θα έχουμε: μεγάλου άξονα 2α. I·

ΕΥΚΛΕΙΔΗΣ Β' 93 τ.l/34


-------

ΗΟΜΟ MATHEMAτiCUS

Από (1 ) και (2) συνάγεται η ισότητα Ρ F ι +Ρ F2= F2M =2α δηλ. το Ρ είναι σημείο της έλλειψης (ii) από δειξη (για την εφαπτο μένη) Αν Ν είναι ένα οποιοδήποτε άλλο (εκτός του Ρ) σημείο της (ε) τότε θα έχουμε σχήμα (α) Ν F ι +Ν F2=NM+ Ν F2> F2Μ =2α, οπότε το Ν δεν ανήκει στην έλλειψη. άρα η έλλειψη και η (ε) έχουν μοναδικό κοινό σημείο το Ρ, δηλ. η (ε) είναι εφαπτομένη της έλλειψης στο σημείο Ρ. β) εκφώνηση /

/

2° θέμα

-------..­

Τέλος αν φέρουμε (ε ' ) .l (ε) στο σημείο Ρ τότε η (ε ' ) είναι διχοτόμος της γωνίας F 1PF2 . (iii) απόδειξη επειδή οι ευθείες (ε ' ), F1M είναι κάθετες στην (ε) θα είναι (ε ' ) 1 1 F ι Μ, άρα <rβ <ry (εντός-εκτός και επί τα αυτά) (3) και (4) <rα <rδ (εντός εναλλάξ) και <ry ..rδ (παρά την βάση ισοσκελούς) (5) Από (3) και (4) με βάση την (5) συμπεραίνουμε ότι <rα <rβ (6) Η (6) μας πληροφορεί ότι μια ακτίνα (φωτός) �ου εκπέμπεται από το σημείο F 1, ανακλώμενη στηy έλλειψη, στο οποιοδήποτε σημείο της, διέρχεται από το άλλο σημείο F2 (κι αντίστροφα), γι' αυτό τα σημεία F 1 ,F2 ονομάστηκαν εστίες της έλλειψης. =

=

=

=

Στο προηγούμενο τεύχος, η φίλη της στήλης Ιωάννα Τριανταφύλλου (Αιγάλεω), μας έστειλε τον ορισμό, την εκφώνηση και απόδειξη ενός θέματος. Λ όγω έλλειψης χώρου (και σ' αυτό το τεύχος), την απόδειξη θα παρουσιάσουμε στο επόμενο τεύχος. γ θέμα

Ένα "μαθηματικό" ποίημα, γραμμένο από τον φίλο της στήλης Νίκο Λυγερό, είναι μεγάλος πειρασμός. Το καθυστερήσαμε άλλωστε πολύ, γι αυτό και το δημοσιεύουμε Όταν ο Αρχιμήδης αποφάσισε ότι δεν ήξερε πώς λύνεται το μη επιλύσιμο έγινε ένα μεγάλο βήμα γιατί ήταν για τότε

Όταν ο Αρχιμήδης αποφάσισε

τεράστια πράξη κι ήταν ο μόνος που δεν έκανε κανένα λάθος στις αναλύσεις του για τον αριθμό π κι έτσι ανακάλυψε

έναν ισομορφισμό μεταξύ προβλημάτων που τελικά αποδείχθηκε ότι δεν λύνονται και προσέγγισε το απίστευτο.

πηγή : από την ηλεκτρονική μας αλληλσyραφία με τον Νίκο Λυγερό (πανεπιστ. Λυ<Qν)

VI. Ειδησούλες - Ειδήσεις

lη. 6η Μαθηματική Εβδομάδα από 26-30 Μάρτη 20 14 πραγματοποιήθηκε, με μεγάλη επιτυχία, η 6η Μαθηματική Εβδομάδα του Παραρτήματος Κεντρικής Μακεδονίας, με ευρεία διεθνή συμμετοχή. Μια διοργάνωση άρτια από κάθε άποψη. Εκείνο που μας προξένησε ιδιαίτερη εντύπωση ήταν ότι η πλειοψηφία των θεμάτων που αναπτύχθηκαν κατατάσσονται στο ουσιαστικό θεματολόγιο.

Από τις κεντρικές ομιλίες διακρίναμε τρεις: i) των Δόρτσιου Κώστα και Τσίντσιφα Γιώργ. με θέμα «Εισαγωγή στη Γεωμετρία Mίnkowski», ίί) του Σταύρου Παπασταυρίδη με · θέμα «Ευκλείδης: κεντρική επιρροή στον δυτικό πολιτισμό)) και ίίί) του Γιάννη Αντωνίου με θέμα «Η διαχείριση του πεπερασμένου με θέα το άπειρω)

ΕΥΚΛΕΙΔΗΣ Β' 93 τ.l/35


-------

ΗΟΜΟ MATHEMAllCUS

-------

2η. βλογώντας τα . . . γένια μας:

το γνωρίζαμε από παλιότερα , ότι μέρη, του Homo mathematicus διδάσκονται σε μια σειρά Σχολείων . Φέτος ένας φίλος της στήλης ο Γρηγόρης Τσιτσίνιας (Θεσσαλονίκη), μας δήλωσε πως δίδαξε H.m. και στο Θερινό Μαθηματικό Σχολείο του Νομού

Ημαθίας «με αφορμή τα θέματα «ευθεία Housel» και «σημείο Franke» (τεύχος 8 6), κάνοντας την απόδειξη των σχετικών θεωρημάτων . . . . συμπληρώσαμε τις γνώσεις πάνω στην ομοιότητα τριγώνων και δώσαμε στοιχεία για τον μετασχηματισμό της ΟΜΟΙΟΘΕΣΙΑΣ . . . »

3η.

Ο <<Σκόρπιες Σταγόνες Γεωμετρίας>': σημαντικός μαθηματικός Γιώργος Μπαλόγλου (ομότιμος καθηγητής του Πολιτειακού Πανεπιστημίου της Νέας Υόρκης, SUNY Oswego), μας δώρισε ένα βιβλίο του πατέρα του Χρήστου (επίσης μαθηματικού, ο οποίος στο διάστημα 1964-

196 7 δίδαξε Παραστατική-Προβολική στην Πολυτεχνική Σχολή του Α.Π.Θ.), με τίτλο «Σκόρπιες Σταγόνες Γεωμετρίας». Εμείς διαβάσαμε μεγάλο μέρος του βιβλίου και ενθουσιαστήκαμε.

4η. βραβείο A bel 2014:

και στην Μαθηματική Φυσική". Ο πρόεδρος της Νορβηγικής Ακαδημίας Επιστημών και Γραμμάτων, Nils Chr. Stenseth, ανακοίνωσε το νικητή του Abel Prize 2 014 στην Ακαδημία Επιστημών του Όσλο, στις 2 6/3/2 014. Yakoν G. Sinai έλαβε το βραβείο Abe1 στις 2 0 Μαίου 2 014.

Η Νορβηγική Ακαδημία Επιστημών και Γραμμάτων αποφάσισε να απονείμει το βραβείο Abel για το 2 014 στο Ρώσο μαθηματικό Yakoν G. Sinai, του Princeton Uniνersity, USA, και τυυ Ινστιτούτου Landau Θεωρητικής Φυσικής, της Ρωσικής Ακαδημίας Επιστημών, "για τις θεμελιώδεις συνεισφορές του στα Δυναμικά Συστήματα, στην Εργοδική Θ εωρία, VI.

,

Απάντη ση στο «Α υτό το ξέρατε;" ,,

α) Το 1 825 Niels Abel βρισκόμενος στο Βερολίνο, καθ' όπως φ άνηκε, πραγματικά ευτυχής. οδόν προς το Παρίσι, γνώρισε έναν Γερμανό εργολάβο κατασκι;υών, τον Αουγκούστ Λέοπολντ Κρέλε, ο οποίος αγαπούσε με πάθος τα Μαθηματικά, απ' τον οποίο γνώρισε πατρική φιλία και ευεργεσία. Σημειωτέον ότι ο Κρέλε του εξέδωσε ένα μαθηματικό περιοδικό χωρίς να λάβει υπ' όψη του πως ο Abel θα έφευγε για το Παρίσι. Αυτό ακριβώς το περιστατικό περιγράφει σε μια επιστολή του προς τον καθηγητή του Κρίστοφερ Χάνστιν (πανεπιστήμιο Χριστιανίας, δηλ. το σημερινό Όσλο). Σ' αυτή την επιστολή μεταξύ άλλων έγραφε:

Ά ρχισε μια εκτεταμένη συζήτηση τότε δύσκολα διάφορα σχετικά με προβλήματα, τα οποία δεν έχουν λυθεί ακόμη. Όταν έφ τασε στη λύση της εξίσωσης 5°" βαθμού, και του είπα ότι είχα αποδείξει ότι ήταν αδύνατο να δοθεί μια γενική αλγεβρική λύση δεν το πίστε ψε και είπε ότι θα μπορούσε να το αναιρέσει. Έτσι του έδωσα ένα αντίγραφο, αλλά είπε πως δεν μπορούσε να δει το σκεπτικό πίσω από αρκετά από τα συμπεράσματά μου. Και άλλοι είχαν πει το ίδιο πράγμα, γι αυτό έκανα μια γενική περιγραφή της . . . >> [πηγή «Η Εξίσωση Που Δεν Μπορούσε να λυθεί>>, ΜΑΡΙΟ ΛΙΒΙΟ, εδόσεις ΕΝΑΛΙΟΣ, Αθήνα 201 3 ] β) ο :ένariste Galois, ("πατέρας" της θεωρίας των

« . . . Πέρασε αρκετός καιρός μέχρι να μπορέσω να του καταστήσω σαφ ή το σκοπό της επίσκεψής μου, και φαινόταν ότι το θέμα όδευε σ ' ένα μελαγχολικό τέλος, τι είχα μελετήσει ήδη στα Μαθηματικά. Πήρα το θάρρος και του ανέφερα τα έργα μερικών από τους πιο ομάδων) διακεκριμένους μαθηματικούς. ότε έγινε πολύ φιλικός και,

VII. Α νακοινώσεις

α) νέα συνεργάτιδα: βρίσκόμαστε στην ευχάριστη θέση να σας ανακοινώσουμε πως η στήλη μας απόκτησε (στην επιμέλεια της ύλης), μια νέα σπουδαία συνεργάτιδα, τη μαθηματικό Αμαλία Μανιατοπούλου . β) επέτειος: με το τεύχος που κρατάτε στα χέρια σας, η στήλη μας συμπληρώνει ακριβώς 1 5 χρόνια ζωής, κι αυτό οφείλεται αποκλειστικά σε σας και μόνο σε σας. Αν στενοχωρήσαμε κάποιους συναδέλφους, ας μας κατανοήσουν, αλλά ο διαθέσιμος χώρος της στήλης (μόνο 4 σελίδες) δεν επαρκεί για εργασίες πέραν της μιας σελίδας. ΕΥΚΛΕΙΔΗΣ Β' 93 τ.l/36


Μαθη ματικά για την Α ' Λυκείου

Α ' ΛΥΚΕΙΟΥ

Π Ι ΘΑΝΟΤ ΗΤ ΕΣ : Ο κλασι κός ορ ισμός Αλκιβιάδης Γ. Τζελέπης, Ευαγγελική

Το άρθρο αποτελεί μία πρόταση διδασκαλίας διάρκειας δύο ωρών - για την εισαγωγή της έννοι­ ας του κλασικού ορισμού της πιθανότητας στην Α· Λυκείου. Διδακτικοί στόχοι:

Η αναγνώριση ενός πειράματος - φαινομένου ως πείραμα τύχης (στοχαστικό) Ο προσδιορισμός του δειγματικού χώρου και ενδεχομένων αυτού, με τη βοήθεια δενδροδια­ γράμματος, πίνακα διπλής εισόδου, διαγράμ­ ματος Venn και η εύρεση του πληθικού αριθ­ μού τους Η κατανόηση του κλασικού ορισμού της πιθα­ νότητας και η επίλυση προβλημάτων με τη βοήθειά του

4.

Προαπαιτούμενες γνώσεις:

Βασικές γνώσεις για τα σύνολα: ο πληθικός αριθμός, οι σχέσεις και οι πράξεις μεταξύ συνό ­ λων, το διάγραμμα Venn ΦΥΛΛΟ ΕΡΓΑΣΙΑΣ Μια τάξη 30 μαθητών (16 κορίτσια και 14 αγόρια) πηγαίνει σε εκπαιδευτική εκδρομή στη βιβλιοθήκη της πόλης. Στο τέλος της ξε­ νάγησης ο υπεύθυνος της βιβλιοθήκης κάνει δώρο στους μαθητές, μετά από κλήρωση, ένα βιβλίο. α) Από τις έως τώρα γνώσεις και την εμπειρία σας, ποια είναι η πιθανότητα ένας μαθητής της τάξης να αποκτήσει το βιβλίο; β) Αν στην κλήρωση μπαίνουν δύο βιβλία, ένα για τα κορίτσια και ένα για τα αγόρια, ποια η πιθανότητα ένα κορίτσι ή ένα αγόρι να αποκτήσει το βιβλίο; γ) τι είναι απαραίτητο να γνωρίζετε για να βρείτε τις προηγούμενες πιθανότητες; 2 . Τι είδους φαινόμενα - πειράματα μπορούν να μελετήσουν οι πιθανότητες; Πειράματα τύχης (στοχαστικά φαινόμενα) - Αιτι­ οκρατικά πειράματα 3. Ποια από τα παρακάτω πειράματα είναι πει­ ράματα τύχης; Να αιτιολογήσετε τις απαντήσεις σας α) Ο χρόνος μεταξύ δύο διαδοχικών εκλείψε­ ων του ήλιου β) Το πλήθος των παιδιών που έχει μία οικο­ γένεια γ) Το πλήθος των πελατών ενός εμπορικού καταστήματος μία συγκεκριμένη ημέρα

l.

5.

Σχολή Ν . Σμύρνης

δ) Ο αριθμός των αεροπλάνων που φθάνουν σε ένα αεροδρόμιο εντός καθορισμένου χρο­ νικού διαστήματος ε) Ο χρόνος που απαιτείται για να διανύσει ένα κινητό γνωστή απόσταση s με σταθερή ταχύτητα ν ζ) Ο τόκος που θα λάβουμε για καταθέσεις ύψους α με προκαθορισμένο επιτόκιο β Το πρώτο ζητούμενο σε ένα πείραμα τύχης είναι η εύρεση του συνόλου των δυνατών α­ ποτελεσμάτων. Στα παρακάτω πειράματα τύ­ χης, να βρεθούν τα δυνατά αποτελέσματα α) Η ρίψη ενός νομίσματος μία φορά β) Η ρίψη ενός ζαριού και η παρατήρηση της ένδειξης της άνω έδρας του γ) Η διαδοχική ρίψη ενός νομίσματος μέχρι να εμφανισθεί η ένδειξη κεφαλή (κ) δ) Η ταυτόχρονη ρίψη δύο ζαριών ε) Η επιλογή v αντικειμένων από μία παρα­ γωγική διαδικασία και ο προσδιορισμός του αριθμού των ελαττωματικών αντικειμένων στ) Ο αριθμός των εκπεμπόμενων σωματιδίων από μία τυχαία επιλεγόμενη ραδιενεργό πηγή σε συγκεκριμένο χρονικό διάστημα ζ) Ο χρόνος λειτουργίας ενός λαμπτήρα φω­ τισμού που επιλέγεται τυχαία από ένα σύνολο λαμπτήρων Να βρεθεί ο πληθικός αριθμός, στις περιπτώ­ σεις που αυτό είναι εφικτό Ας ρίξουμε ένα ζάρι μία φορά και ας ορίσου­ με ότι κάποιος κερδίζει αν φέρει αποτέλεσμα μεγαλύτερο του 4 α) Να βρείτε το σύνολο των .δυνατών αποτε­ λεσμάτων Ω που μπορούν να προκύψουν από τη ρίψη του ζαριού. Να το aπεικονίσετε με ένα διάγραμμα Venn. Ποιος είναι ο πλη­ θικός αριθμός του συνόλου; β) Να βρείτε το σύνολο Α των αποτελεσμά­ των που δίνουν νίκη και το σύνολο Β των α­ ποτελεσμάτων που οδηγούν στην ήττα γ) Ποια είναι η πιθανότητα ένας άνθρωπος να κερδίσει; δ) Ποια είναι η πιθανότητα ένας άνθρωπος να χάσει; ε) Τι παρατηρείτε σχετικά με τις δύο πιθανό­ τητες; Τι παρατηρείτε σχετικά με τους πληθι­ κούς αριθμούς των συνόλων Α και Β; Ποια σχέση υπάρχει μεταξύ των συνόλων Α, Β και Ω; Να γράψετε το σύνολο που περιλαμβάνει τα κοινά στοιχεία των Α και Β

ΕΥΚΛΕΙΔΗΣ Β ' 93 τ.l/37


Μαθη ματικά για την Α' Λυκείου ζ) Ποια είναι η πιθανότητα να κερδίσει ένας άνθρωπος, δεδομένου ότι έχει φέρει αποτέλε­ σμα μεγαλύτερο από 2; Να δοθούν οι ορισμοί: α) Δειγματικός χώρος β) Ενδεχόμενο (απλό - σύνθετο - αδύνατο βέβαιο). Πραγματοποίηση ενδεχόμενου γ) Ορισμός πιθανότητας ενός ενδεχομένου (κλασικός ορισμός) Τι τιμές μπορεί να πάρει η πιθανότητα οποι­ ουδήποτε ενδεχομένου; Γιατί;

6.

ΑΠΑΝ ΤΗΣΕΙΣ - ΥΠΟΔΕΙΞΕΙΣ

1 1 1 Ρ ιcορ = - , Ρ αr = =) β 30, 14 16 4 . Ω 1 = {κ,γ} , Ω2 = {1,2,3,4, 5 ,6} , Ω3 = {κ.γκ,γγκ, ... } , Ω = { (1,1),( 1, 2 ), ... , ( 6, 6 ) } 4 Ω5 = {0,1,2, ... , ν} , Ω6 = {0,1,2, ... } , Ω, = {t : t � O} = (O,+oo ) ί) Ν α γίνει δενδροδιάγραμμα για το (4y) και πίνα­ κας διπλής εισόδου για το ( 4δ) ii) Ν( Ω, ) = 2, Ν{� ) = 6, Ν(Ω4 ) =36, Ν{Ωs) =ν+1 1 . α)

Ρ1

οι πληθικοί αριθμοί των αντίστοιχων πεπερασμέ­ νων συνόλων. Τα Ω3 και Ω6 είναι αριθμήσιμα α­ πειροσύνολα, ενώ το Ω7 είναι υπεραριθμήσιμο (συνεχές) aπειροσύνολο. 5. α) Ω2 = {1,2,3,4, 5 , 6} , Ν(Ω) = 6,

ΕΦΑΡΜΟΓΕΣ

1 . Ένας πομπός εκπέμπει κωδικοποιημένο ψηφι­

ακό σήμα, το οποίο λαμβάνεται με τη συμβο­ λική μορφή Ο και 1. Αν υποθέσουμε ότι ο δέ­ κτης πρόκειται να λάβει μία «λέξη» τριών ψη­ φίων, τότε να βρείτε το δ.χ. του πειράματος. 2. Ρίχνουμε ένα νόμισμα 2 φορές. Να βρεθεί η πιθανότητα των ενδεχομένων: Α: «Να έρθει και τις δύο φορές Κ » Β: «Να έλθει Κ τη δεύτερη φορά, δεδομένου ότι την πρώτη φορά έχει έλθει Κ » 3. Από μία τράπουλα 5 2 φύλλων επιλέγουμε τυ­ χαία ένα φύλλο. Να βρείτε την πιθανότητα το φύλλο να είναι: α) άσσος β) ρήγας γ) σπαθί δ) άσσος σπαθί 4. Ρίχνουμε ένα ζάρι δύο φορές. Τι είναι πιθανότερο να φέρουμε, άθροισμα 1 1 ή άθροισμα 4; Το πρόβλημα Σε ένα λούνα πάρκ υπάρχει ο τροχός της τύ­ χης που φαίνεται στο σχήμα. Όλα τα τόξα είναι ίσα και ο τροχός είναι αμερόληπτος. Για να παίξουμε πληρώνουμε 2 €. Αν με το γύρισμα του τροχού έρ­ θει το "Ο" χάνουμε, αν έρθει το "1" παίρνουμε 3 €, ενώ αν έρθει το "2" παίρνουμε 4 €. Να εξετάσετε αν το παιχνίδι ευνοεί τον παί­ κτη ή τον ιδιοκτήτη

-

2 β) Α = {5,6} , Β = {1, 2 ,3,4} , γ) Ρ ( Α ) = 6 , 4 Ρ ( Β) = , δ) 6

ε) Ρ ( Α) + Ρ (Β) = 1 , Ρ ( Α ) < Ρ ( Β ) , Ρ ( Β ) = 2Ρ ( Α ) Ν { Α ) + Ν ( Β ) = 6 = Ν ( Ω), Ν (Α) < Ν (Β) , Ν ( Β ) = 2Ν ( Α )

Σχέση συνόλων: Α υ Β = Ω , Κοινά στοιχεία:

A. n B = 0

ζ)

Ρ

(

νίκης δεδομένου ότι έχει φέρει

)= 2 2 4 ) = ����

αποτελέσματα μεγαλύτερο από

6. Κλασικός ορισμός πιθανότητας:

Ρ (Α

Ν (Α ) � Ο , Ν ( Ω) > Ο , επομένως Ρ ( Α ) � Ο , με Ρ ( 0) = Ο (αδύνατο ενδεχόμενο)

Α υποσύνολο του Ω, επομένως

Ν {Α) s; Ν ( Ω) <=> ·

Αν

Ν (Α) -Ν ( Ω)

Ν (Ω)

s; -- <:>

Ν {Ω )

P ( A ) s; 1

Ρ ( Α) = 1 , το ενδεχόμενο καλείται βέβαιο

Λύ ση Επειδή οι κυκλικοί τομείς είναι ίσοι, ο τροχός είναι αμερόληπτος. Ως εκ τούτου, τα ενδεχόμενα να σταματήσει ο τροχός σε οποιονδήποτε από τους 6 τομείς είναι ισοπίθανα. Επομένως ισχύει:

P(O) = l , Ρ(1) = �. Ρ (2) = .!_

6

6

6

6·ν

' ' ο αριθ μος φορες

Αν υποθέσουμε ότι ο παίκτης θα παίξει ν φο­ ρές, τότε αναμένεται να προκύψουν

3

'

' 6 · ν φορες ο αριθ μος

2

Ο,

1 ' ο αριθ μος ' 2 6 · ν φορες

Ο παίκτης θα έχει πληρώσει για να παίξει στο ευρώ, ενώ θα εισπράξει

παιχνίδι

(�· ν } ( � · ν } ( i ·ν } 4 f · ν -35 · ν 2 · ν Ο

Αλλα'

+

3

<

νοεί τον ιδιοκτήτη .

ΕΥΚΛΕΙΔΗΣ Β' 93 τ.l/38

+

=

ευρώ

' , επομενως το παιχνι' δι ευ-


Α ' ΛΥΚΕΙΟΥ Ταυτότητες στους πραγματι κούς αρ ιθμούς

-

JR

Σαραφοπούλου Χαρίκλεια Παράρτημα Ημαθίας

1.

=

iii. χ 4 + χ 2 + 1

Ν α αποδειχθεί η ταυτότητα Euler:

α3 + β3 + y3 - 3αβy ( α + β + y) [ ( α _ β ) z + (β _ y) Z + (y

_

α) 2 ]

Από δειξη : � ( α + β + y) [ (α - β ) 2 + (β - y) z +

(y - α ) 2 ] = � ( α + β + y) ( α2 - Ζ αβ + β2 + β z - Ζ βy + y2 + y z - Ζ αy + αΖ ) = (α + β + γ) ( α2 + β2 + y2 - αβ - αy - βy) = α 3 + β 3 + y 3 - 3 αβy

2. Αν α+ β+γ=Ο τότε α3 + β3 + y3 = 3αβy Από δειξη : Από ( 1 ) έχουμε: α 3 + β 3 + y 3 - 3 αβy

1 = 2 ( α + β + y ) [ ( α - β ) z + (β - y ) 2 + (y - α ) Ζ ] � 3 + β 3 + y 3 _ 3 αβy = 0 � 3 + β 3 + y 3 = 3 αβ 3.

Να μετατραπεί σε γινόμενο παραγόντων η

Κ=rα} (β _ y) 3 + β3 (y _ α) 3 + y3 ( α _ β ) 3

Απάντη ση

Έχουμε: αβ - αy + βy - βα + yα - yβ = Ο Άρα α 3 (β y) 3 + β 3 (y _ α ) 3 + y 3 ( α _ β ) 3 =

( αβ - αy ) 3 + (βy - βα ) 3 + (yα - yβ ) 3 = = 3 αβy (β - y) (y - α ) ( α - β ) _

4.

Αν α + !. = 1 και β + !. = 1 να αποδειχθεί β

ι

ότι: y + - = 1 με α α

*

Από δειξη 1 β

Υ

Ο, β

*

Ο, y

Έχουμε:

α+

Άρα

= 1 , β + .!. = 1 δηλ. β = Υ

α + -1 = 1 � + p

y:1

Υ

=

*

Ο

1 � + L1 = y-

1 �y + 1 = α=> αy + � = � =>Υ + � = 1 5.

α

α

α

α

Να γίνουν γινόμενο:

i. 4 ( αβ + yδ) z - ( α 2 + β z - yz - δz ) ii. Χ4 + 4y4 ίίί. Χ4 + ΧΖ + 1 Απάντη ση i.

2

4( αβ + yδ ) z _ ( α Ζ + β z _ y2 _ δ z ) z = ( Ζ αβ + Ζ yδ + α2 + β 2 - y2 - δ2 ) ( Ζ αβ + Ζ yδ - α2 - β2 + y2 + δ2 ) = β α [ ( + ) z - (y - δ ) z ] [(y + δ ) z - ( α - β ) z ] = ( α + β + y - δ) ( α + β - y + δ) (y + δ + α - β ) (y + δ - α + β ) 4 4 ii. χ + 4y = χ 4 + 4y 4 + 4x z y z 4x z y z = = (χ 2 + 2 y 2 - 2 xy) (x2 + 2 y 2 + Zxy ) _

-'

Απάντη ση

χ 2 + y 2 = z 2 .=:?Χ Ζ = - y z + z 2 και y z = z 2 _ χ 2 Τότε: Α = χ3 + y 3 + z 3 = xx z + y 3 + z 3 = x (z 2 - y 2 ) + (y + z) (y 2 - yz + z 2 ) = ( x z + y) (z - y) + (y + z ) (y 2 - yz + z 2 ) = (y + z) (xz - xy + y 2 - yz + z 2 ) = = (y + z) (xz - xy + z 2 - χ 2 - yz + z 2 ) = (y + z) [z (x + z) - y (x + z) + (z - x) (z + χ) ] = = (y + z) (x + z ) ( Z z - x - y) Αν α, β, y ομόσημοι και (α + β + 7. y) (; + + ) = 9 � α = β = y)

� ;

(

Από δειξη

)

1 1 1 ( α + β + y) � + + Υ = 9 .=:::? β β α βy y αy + ( + )( + + αβ ) = 9αβy => z z z α β + α y + αβ + β z y + αy z + β y z - Ζ αβy - Ζ αβ y - Ζ αβ y = Ο � β ( α2 + y2 - Ζ αy ) + y ( α 2 + β2 - Ζ αβ ) + α (β2 + y2 - Ζ βy) = Ο � β ( α - y ) 2 + y ( α - β ) 2 + α (β - y ) Z = Ο Οι α,β,γ είναι ομόσημοι τότε: β ( α - y) 2 = Ο , y ( α - β ) 2 = Ο, α (β - y) 2 = Ο . Άρα α β γ =

y- 1 Υ

6.

χ 4 + zx 2 + 1 - χ 2 = (χ 2 + 1) 2 - χ 2 = (χ 2 + χ + 1) (χ 2 - χ + 1) Εάν χ2 + y2 = z 2 να γίνει γινόμενο η χ 3 + y3 + z 3

1. 2. 3. 4.

5.

Ασκήσε ις γ ια ε ξά σκ η σ η

=

3 z 3 z 1 Αν x z + y z 1 τότε (3χ - 4x ) + (3 y - 4y ) z z Αν α + {J + y Ο και αΖ + p + y 1 να υπολογιστεί η τιμή της παράστασης: Α=α 4 + p 4 + y 4 1

χ(χ-γ) (χ-z)

+

=

=

=

1

γ(γ -z)(γ-χ)

6.

7.

+

1

z(z-x) (z-γ)

=

χγz

Να απλοποιηθεί η παράσταση χ(χ + 2y) + y (y + 2z) + z(z + 2 χ ) χΖ - y z - z 2 - 2xyz Να βρεθεί ο λ Ε Im., όπου λ =-αν ισχύει η σχέση :

χ+y

=

=

γ

8χ - 9y

x-y Υ Αν 4 (α 2 + p z + y z + δ z λ (α + {J + y + δ) z αποδειχθεί ότι α= β= γ =δ Αν α + {J + y Ο να αποδειχθεί ότι α 4 + p 4 + y 4 2 (α{J + αy + y{J) z Αν ισχύει: .:. !. � να αποδειχθεί ότι z ω γ χ χ3 + y3 + ω3 --

Να

=

--

=

=

=

8.

9.

=

=

y 3 + ω 3 + z3 Αν α + {J + y 2 και α Ζ + p z + y z � 4 Να αποδειχθεί ότι:α 3 + {1 3 + y 3 3α{Jy + 8

ΕΥΚΛΕΙΔΗΣ Β' 93 τ.l/39

=

;

=

=


Μ αθηματικά για την Α ' Λυκείου

Α ' ΛΥΚ ΕΙΟΥ

Ασκήσεις Γεωμετρίας

1. Δίνεται ΑΒΓ με ΑΒ<ΑΓ. Η μεσοκάθετος της πλευ­

ράς ΒΓ τέμνει την εσωτερική διχοτόμο της Α στο Ο και την εξωτερική διχοτόμο της γωνίας Α στο Δ. Φέρνουμε τις προβολές Ε, Ζ του ση μείου Ο στις � θείες ΑΓ, ΑΒ αντίστοιχα και τις 'προβολές Κ, Ν του σημείου Δ στις ευθείες ΑΓ, ΑΒ αντίστοιχα. Να δείξετε: α.

Σαράφης Γιάννης - Παράρτημα Ημαθίας

ΚΓ = ΜΓ β + γ οπότε 2 2 ' β + γ = -(β 1 - γ) ΑΚ = ΑΓ - ΚΓ = β - 2 2 Έχουμε ΑΚ=ΑΝ ( ΑΝΔ = ΑΔΚ ) =

γ.

Ετ= (p-γ) β. ΑΚ= ( p-γ) γ. ΖΝ=β, ΚΕ:::ιy

Δ

Δ

Δ

Δ

και ΑΖ=ΑΕ( ΑΟΖ = ΑΟΕ )

α. Στην ΑΓ θεωρούμε σημείο Θ έτσι, ώ στε ΑΒ=ΑΘ Τότε τα τρίγωνα ΑΘΟ και ΑΒΟ είναι ί, σα. (ΑΘ=ΑΒ,ΟΑ κοινή και ΒΑΟ = ΟΑΘ ). Άρα ΟΒ=ΟΘ . Το Ο είναι σημείο μεσοκαθέτου οπότε ΟΒ=ΟΓ Λύση :

Λ

Λ

Α

1 ΖΝ=ΑΖ+ΑΝ=ΑΕ+ΑΚ=β-ΕΓ+ -(β - γ) = 2 ο

Άρα ΟΘ=ΟΓ δηλ. το ΘΟΓ είναι ισοσκελές και ΟΕ ύψος . Άρα ΟΕ μεσοκάθετος του ΘΓ . ΘΓ 1 ΘΓ=ΑΓ-ΑΘ=ΑΓ-ΑΒ=β-γ, ΕΓ = -τ = '2 (β-γ)

p. Στην προέκταση της ΑΓ θεωρούμε σημείο Μ

τέτοιο ώστε ΑΜ=ΑΒ . Τα ΑΜΔ και ΑΒ Δ είναι ίσα (ΑΜ=ΑΒ , ΑΔ κοινή και ΜΑΔ = ΒΑΔ ) οπότε ΜΔ=ΒΔ . Λ

Λ

1 1 =β- -(β - γ ) + -(β - γ ) =β 2 2 ΚΕ=β-ΑΚ-ΕΓ=β-2 21 (β - γ) =β-β+γ=γ 2 . Δίνεται τρίγωνο ΑΒΓ με ΑΒ<ΑΓ. Έστω Μ

τυχαίο σημείο της διχοτόμου ΑΔ. Θεωρούμε σημείο Ε στην ΑΓ ώστε ΑΒ=ΑΕ. Να αποδείξετε ότι: α. ΒΜ=ΜΕ

p.

Λ

Η παραπληρωματική της ΑΒΜ είναι μεγαλύΛ

τερη της Γ .γ. ΜΓ > ΜΕ δ. ΜΒ < ΜΓ α . Τα ΑΒΜ και ΑΜΕ είναι ίσα

διότι έχουν ΑΜ κοινή, ΑΒ=ΑΕ και ΒΑΜ ΜΑΓ (Π-Γ-Π). Άρα ΒΜ=ΜΕ και ΑΒ Μ = ΑΕΜ . Λύση :

Λ

Λ

=

Λ

Λ

Α

Το Δ είναι σημείο της μεσοκαθέτου του τμήματος ΒΓ , οπότε ΒΔ=ΔΓ . Άρα ΜΔ=ΔΓ το ΜΔΓ ισοσκε­ λές και η ΔΚ ύψος. Άρα Κ μέσο της ΜΓ , ΜΓ=ΜΑ+ΑΓ=ΑΒ+ΑΓ=β+γ

Λ

Λ

p. Η παραπληρωματική της ΑΒΜ είναι η ΜΒχ Λ

Λ

Λ

:Εχουμε ΜΒχ > ΓΒχ . Η ΓΒχ είναι εξωτερική

ΕΥΚΛΕΙΔΗΣ Β' 93 τ.l/40


Λ

Μαθη ματικά για την Α' Λυκείου

Λ

Λ

Λ

Λ

στο ΑΒΓ, οπότε ΓΒχ > Γ . Άρα ΜΒχ > ΓΒχ > Γ Λ

Λ

οπότε ΜΒχ > Γ .

ΑΒΜ = ΑΕΜ

γ. Επειδή

και οι παραπληρωματικές Λ

ίί. Η ΑΔ είναι διχοτόμος της γωνίας ΕΔΖ. Λύση : α. Έστω Θ το σuμμετρικό του Δ ως προς την ευθεία ΑΒ και Λ το σuμμετρικό του Δ ως προς την ευθεία ΑΓ. Ισχύει ΔΕ=ΕΘ και ΖΔ=ΖΛ.

Λ

τους θα είναι ίσες δηλ. ΜΒχ = ΜΕΓ . Στο ΜΕΓ απέναντι από άνισες γωνίες βρίσκονται το ίδιο άνισες πλευρές δηλ. ΜΓ>ΜΕ. δ . Από το ερώτημα (α) έχουμε ΜΒ=ΜΕ και από το προηγούμενο ΜΕ<ΜΓ άρα ΜΒ<ΜΓ. Από το (γ) Λ

Λ

Λ

Λ

έχουμε ΜΒχ > Γ οπότε ΜΕΓ > Γ . 3 . Δίνεται κύκλος (Ο,ρ) και τα aντιδιαμετρικά σημεία Β και Γ. Φέρνουμε τις εφαπτόμενες χ 'Βχ και y Ty με Βχ και Γy προς το αυτό μέρος της ΒΓ. Σε τυχαίο σημείο Ε φέρνουμε εφαπτομένη η οποία τέμνει τις Βχ και Γy στα σημεία Μ,Δ α­ ντίστοιχα. Η ΔΟ τέμνει την χ 'Β στο Λ. Δείξετε Λ

ότι: α. ΜΟΔ = 90° β. ΟΔ=ΟΛ γ. το ΔΜΛ είναι ισοσκελές. δ . ΕΔ=ΒΛ Λύση : α.

Η

Λ

Δ

Για την περίμετρο του τριγώνου ΔΕΖ έχουμε: ΔΕ+ΕΖ+ΖΔ=ΕΘ+ΕΖ+ΖΛ. Το άθροισμα γίνεται ελάχιστο όταν τα σημεία Θ,Ε,Ζ,Λ είναι σuνευθειακά. Άρα οι κορυφές του τριγώνου ΔΕΖ είναι τα σημεία Κ και Ι που είναι τα σημεία τομής της ΘΛ με τις πλευρές ΑΒ,ΑΓ αντί­ στοιχα.

ΟΜ είναι διχοτόμος της ΒΟΕ και η Λ

ΟΔ είναι διχοτόμος της ΕΟΓ . Οι ι.

Λ

Λ

ΒΟΕ και ΕΟΓ είναι εφεξής και παραπληρωματικές και οι ΟΜ, ΟΔ διχοτόμοι αυτών. Άρα Λ

ΜΟΔ = 90° . β. Τα τρίγωνα ΒΟΛ και ΟΓΔ εί­ ναι ίσα διότι έχουν ΟΒ=ΟΓ, Λ

Λ

Λ

Δ

Λ

Β = Γ = 90ο και ΔΟΓ = ΒΟΛ ως κατακορυφήν. Άρα ΟΔ=ΟΛ. γ. Στο τρίγωνο ΛΜΔ η ΟΜ είναι ύψος και διάμεσος, οπότε το τρίγωνο είναι ισοσκε­ λές. δ . Από το (γ) έχουμε ΜΔ=ΜΛ και ΜΕ=ΜΒ εφα­ πτόμενα τμήματα, άρα ΕΔ=ΜΔ-ΜΕ=ΜΛ-ΜΒ=ΒΛ. 4. α . Δίνεται οξυγώνιο τρίγωνο ΑΒΓ και το ύψος του ΑΔ. Ζητούνται σημεία Ε,Ζ των πλευρών ΑΒ,ΑΓ αντίστοιχα ,ώστε η περίμετρος του ΔΕΖ να είναι ελάχιστη. β. Έστω η περίμετρος του τριγώνου ΔΕΖ είναι ελάχιστη. Να αποδείξετε ότι: ί. ΑΘ=ΑΛ (τα Θ και Λ όπως στο σχήμα).

-----

β.ί. Επειδή η περίμετρος του τριγώνου ΔΕΖ είναι ελάχιστη από το προηγούμενο ερώτημα έχουμε ότι η γραμμή ΘΕΖΛ είναι ευθεία. Η ΑΒ είναι μεσοκάθετος της ΘΔ άρα ΑΘ=ΑΔ, όμοια η ΑΓ είναι μεσοκάθετος της ΔΛ οπότε ΑΔ=ΑΛ. Άρα ΑΔ=ΑΛ και ΑΔ=ΑΘ οπότε ΑΘ=ΑΛ δηλ. το τρίγωνο ΑΘΛ είναι ισοσκελές. ίί. Επειδή το τρίγωνο ΑΘΛ είναι ισοσκελές έχουμε Λ

Λ

ΑΘΕ = ΑΛΖ . Τα τρίγωνα ΑΘΕ και ΑΕΔ είναι ίσα Λ

Λ

(ΑΘ=ΑΔ, ΑΕ κοινή, ΕΘ=ΕΔ) οπότε ΑΘΕ = ΕΔΑ . Τα τρίγωνα ΑΖΛ και ΑΖΔ είναι ίσα (ΑΖ κοινή, ΑΛ=ΑΔ, Λ

Λ

ΖΛ=ΖΔ)

οπότε

Λ

Λ

ΑΛΖ

=

ΑΔΖ . Άρα �

ΕΔΑ = ΑΔΖ δηλ. η ΑΔ είναι διχοτόμος της ΕΔΖ .

ΕΥΚΛΕΙΔΗΣ Β' 93 τ.l/41


Μαθη ματικά για την Α ' Λυκείου ΓΡΑΠΤΗ

ΕΞΕΤΑΣΗ ΣΤΗ Ν ΑΛΓΕ ΒΡΑ της Α 'ΛΥΚΕΙΟΥ

Τσαγκάρης Κωνσταντίνος

Εξεταζόμενη ύλη : • ΕΙΣΑΓΩΓΙΚΟ ΚΕΦΑΛΑΙΟ •

ΚΕΦΑΛΑΙΟ ι 0 : Πιθανότητες

I

Χρόνος: δύο διδακτικές ώρες

ΘΕ ΜΑ 1°. Α. Να αποδείξετε ότι: για οποιαδήποτε ασυμβίβαστα μεταξύ τους ενδεχόμενα Α και Β του δειγματικού (Μονάδες 15) χώρου Ω ισχύει: Ρ(ΑυΒ) = Ρ(Α) + Ρ(Β) ( Σχολ Α' Λυκ Σελ 33)

Β. Να χαρακτηριστεί η κάθε πρόταση ως Σωστή Σ Ο ή Λάθος Λ Ο

α) Ο ισχυρισμός « Ρ <:=> Q» λέγεται ισοδυναμία και διαβάζεται « Αν Ρ, τότε Q»

( Σχολ Α ' Λυκ Σελ 1 0)

( Σχολ Α ' Λυκ Σελ 1 1 ) β) Ισχύει η ισοδυναμία : α· β * Ο <:=> (α * Ο ή β * Ο) γ) Δύο σύνολα Α και Β λέγονται ίσα , όταν έχουν τα ίδια ακριβώς στοιχεία. ( Σχολ Α ' Λυκ Σελ 1 5)

ΣΟ

ΣΟ ΣΟ

ΛΟ

ΛΟ ΛΟ

δ) Έστω Ω ένα βασικό σύνολο και δύο υποσύνολα του, Α και Β. Είναι ΑυΒ = {χ ε ΩΙ χ ε Α ή χ ε Β } ΣΟ ΛΟ ( Σχολ. Α ' Λυκ , Σελ 1 7) ε) Δύο ενδεχόμενα Α και Β του δειγματικού χώρου Ω, λέγονται ασυμβίβαστα , όταν AnB = 0 ΣΟ ΛΟ ( Σχολ. Α ' Λυκ., Σελ 24)

ΘΕ Μ Α 2°. ( Α Λυκ. : παρ�δειγμα σελ 1 1 , κατανόησης σελ 1 2 , από το Ε Ι , Το Λεξιλόγιο της Λογικής)

(Μονάδ ες 10)

Δίνονται οι παραστάσεις Α = χ2 - 2χ και Β = χ2 - 4, όπου χ πραγματικός αριθμός. Α. Να παραγοντοποιήσετε τις παραστάσεις Α και Β και να υπολογίσετε τις τιμές του χ για τις οποίες (Μονάδες 6) μηδενίζεται το Α. (Μονάδες 7) Β. Όλες οι τιμές του χ που μηδενίζουν το Β, μηδενίζουν και το Α; Γ. Κάθε ισχυρισμός της l η ς ΟΜΑΔΑΣ , του πίνακα Ι , aντιστοιχίζεται με τον ισοδύναμό του ισχυρισμό της 2 η ς ΟΜΑΔΑΣ . Να συμπληρώσετε τον πίνακα Π, αιτιολογώντας την επιλογή σας για το 2 της 1 ης ομάδας. I

ιη

Π ι'ν ακας Ι

ΟΜΑΔΑ Α

;t Ο

α

β

2

Α·Β = Ο

3

Α·Β = Ο και χ<Ο

4

γ

Β > Ο και χ>Ο

δ ε

(χ= -2 ή χ = ο ή χ= 2)

ζ

:

ι ι ι ι ι

-

-

n

ι

�- - - - - - - - - - - - - - - -·

Πίν ακα Π

I �: ���: 1 1 1 2 1 3 1 4 I

χ ;t Ο και x;t 2 χ=2

' Λυκ. : Ε2, Σύνολα ) ΘΕ Μ Α 3°- (- Α ----- ,

ι - - - - - - - -

ΟΜΑΔΑ

x ;t 0 ή x ;t 2 (χ = - 2 και χ= Ο και χ=2) χ=-2 χ>2

στ

η

χ>Ο

( Μονάδες 2+6+2+2 12 ) =

Στο διάγραμμα Venn δίνονται εποπτικά τα σύνολα Ω, Α, Β και με περιγραφή τα σύνολα Γ , Δ , όπου Γ = { χ ε Ω I 1 < 2χ+3 � 7 } και Δ = { χ ε Ω I (χ-3)(χ-λ2 ) =Ο } .

Α. Να γράψετε με αναγραφή των στοιχείων τους τα σύνολα Ω και Α.

Β. Να αιτιολογήσετε ότι ΓςΑ και να βρείτε τον πραγματικό αριθμό λ , ώστε Δ = AnB

(Μονάδες 6) (Μονάδες 3+7)

Γ. Να γράψετε με αναγραφή των στοιχείων του το σύνολο (ΑυΒ) ' και στη συνέχεια αν (ΑυΒ) ' = { χ ε ΩΙ χ έχει την ιδιότητα Ι } , ποια θα μπορούσε να είναι η ιδιότητα Ι; ( Μονάδες 4+5 ) Θ Ε Μ Α 4° ( Γ ' Γυμν σελ. 180 (Γενικές Ασκ), Α Λυκ. : από το Κεφάλαιο 1°, Πιθανότητες ) Δίνονται τα σύνολα Ω = {χ εΝΙ χ < 1 0 } , Α= {χ ε ΩΙ χ περιττός} και Β= {χ ε ΩΙ χ διαιρέτης του 1 6 } . λ. Να παραστήσετε εποπτικά τα σύνολα Α και Β με διαγράμματα Venn και βαmκό σύνολο το Ω (Μονάδες 5) Β. Τα ενδεχόμενα Α και Β είναι ασυμβίβαστα; (Μ ονάδες 5) Αν επιλέγονται στην τύχη τα στοιχεία του Ω : Γ. Να βρείτε την πιθανότητα i) το στοιχείο να ανήκει στο Α (Μονάδες 10) ii) το στοιχείο να ανήκει στο Α ή στο Β. Δ. Να βρείτε την mθανότητα του ενδεχομένου «να πραγματοποιείται μόνο ένα από τα Α και Β». (Μονάδες 5)

ΕΥΚΛΕΙΔΗΣ Β' 93 τ.1/42


Μαθη ματικά για την Α' Λυκείου

Ο ΕΥΚΛΕ ΙΔΗΣ

Β 'κα ι η ΤΡΑΠ ΕΖΑ ΘΕΜΑΤΩΝ της Α 'ΛΥΚΕ Ι ΟΥ

Την Δευτέρα 23 Ιουνίου 2014 στην συνεδρί­ αση της συντακτικής επιτροπής του περιοδικού «ΕΥΚΛΕΙΔΗΣ Β ' » της Ε.Μ.Ε. έγινε συζήτηση και εκφράστηκαν απόψεις για την Τράπεζα Θεμάτων της Α ' Λυκείου στα μαθήματα της Άλγεβρας και της Γεωμετρίας. Από τη συζήτηση αυτή, προέ­ κυψαν τα παρακάτω σχόλια και παρατηρήσεις καθώς και προτάσεις για την αποτελεσματικότερη (αν γίνεται) λειτουργία της. Η τράπεζα θεμάτων του Υπουργείου Παιδείας περιέχει σε κάθε έναν από τους κλάδους των Μαθηματικών: Άλγεβρας και Γεωμετρίας δύο ομάδες θεμάτων. Θέματα για το 2 ° και το 4 ° ζήτημα των προαγωγικών εξετά­ σεων της Α ' τάξης όλων των λυκείων. Η τράπεζα θεμάτων δόθηκε φέτος στη δημοσιότητα 4 μέρες πριν ξεκινήσουν οι εξετάσεις των λυκείων. Στις 34 μέρες που χώριζαν την δημοσίευση από της εξε­ τάσεις όπως, ήταν φυσικό, μαθητές και καθηγητές έσπευσαν να λύσουν τις ασκήσεις για να δουν το μέγεθος της δυσκολίας ή ευκολίας των. Βρέθη­ καν κάποια λάθη και κάποιες ακρότητες και αυ­ τό είχε σαν αποτέλεσμα μέχρι την έναρξη των εξε­ τάσεων να αποσυρθούν μερικές από τις ασκήσεις αυτές. Στη συνέχεια ήρθε η εφαρμογή της. Αρχικά υπήρχε ένας φόβος για το πώς θα λειτουργήσει η διαδικασία της απόκτησης των θε­ μάτων. Όμως δεν υπήρξε κανένα πρόβλημα. Ο δι­ ευθυντής κάθε σχολείου λίγο πριν τις εξετάσεις κατέβαζε τα δύο θέματα από το «σύστημα» και τα έδινε στους καθηγητές. Από το σημείο αυτό και μετά αρχίζουν τα προβλήματα: 1 . Η επιλογή του 2ου και του 4ου θέματος γίνεται με τυχαίο τρόπο με αποτέλεσμα σε πολλές πε­ ριπτώσεις να προέρχονται από το ίδιο κεφά­ λαιο αλλά και από την ίδια ενότητα ύλης. Ποτέ δεν θα συνέβαινε αυτό αν τα θέματα τα επέλεγαν οι διδάσκοντες καθηγητές. Το πρό­ βλημα αυτό θα μπορούσε να μην ήταν τόσο έντονο αν το 4° θέμα ήταν πραγματικά συν­ δυαστικό οπότε δεν θα ήταν από το ίδιο κεφά­ λαιο ή ενότητα ύλης με το 2° θέμα. Για το θέ­ μα αυτό θα αναφερθούμε και πιο κάτω. 2. Αναφέρθηκε ότι σε μερικές περιπτώσεις οι δι­ δάσκοντες καθηγητές, επειδή δεν είχαν επαρκή χρόνο για να ετοιμάσουν τα δικά τους θέματα ( 1 ο και 3°) μετά την κλήρωση ώστε αυτά να μη συμπίπτουν στα κεφάλαια με αυτά της τράπε­ ζας. Αποτέλεσμα το διαγώνισμα αυτό συνολι­ κά να μη καλύπτει όλη την έκταση της ύλης. Ακούσαμε ότι υπήρξε διαγώνισμα στο οποίο έλειπαν τα κεφάλαια: 2° , 3° και 4°.

3. Ένα άλλο θέμα που αναφέρθηκε, πολύ σημα­ ντικό, είναι η ανομοιογένεια του επιπέδου δυ­ σκολίας των θεμάτων, πάντα μέσα στην ίδια κατηγορία. Υπάρχουν εύκολα και &ύσκολα θέματα για το 2 ° καθώς εύκολα και δύσκολα θέματα για το 4° . Για παράδειγμα αναφέρουμε τα παρακάτω:

ΑΛΓΕΒΡΑ 4 ° ΘΕ Μ Α ( Δύσκολο)

Τέσσερις αθλητές, ο Αργύρης, ο Βασίλης, ο Γιώρ­ γος και ο Δημήτρης τερμάτισαν σε έναν αγώνα δρόμου με αντίστοιχους χρόνους (σε λεπτά) t A , t 8 , tr και tΔ , για τους οποίους ισχύουν οι σχέσεις:

t + 2 t B και t - tΔ - t 8 - tΔ . t A < t 8 , tr = A 1 A 1 _1 1 3 ' tΔ = t A + t B . Α) ι) . Να δει'ξετε οτι: 2 ii)

Β)

Να βρείτε τη σειρά με την οποία τε�τισαν οι αθλητές. Να αιτιολογήσετε την απάντηση σας. Δίνεται επιπλέον ότι ισχύει: t A + t 8 = 6 και t A · t 8 = 8 i) Να γράψετε μια εξίσωση 2ου βαθμού που έχει ρίζες τους αριθμούς t A και t 8 ii) Βρείτε τους χρόνους τερματισμού των τεσσάρων αθλητών.

4 ° ΘΕ Μ Α ( Ε ύ κολο)

Για τους πραγματικούς αριθμούς α,β ισχύει: Ι α - 2 1 < 1 και I β - 3 1 � 2 . Α) Να αποδείξετε ότι: 1 < α < 3 Β) Να βρείτε μεταξύ ποιων αριθμών βρίσκεται ο β Γ) Να βρεθεί μεταξύ ποιων αριθμών βρίσκεται η παράσταση 2α 3 β Δ) Να βρεθεί μεταξύ ποιων αριθμών βρίσκεται η -

, παρασταση

βα

Η ανομοιογένεια αυτή έγινε αφορμή τα σχο­ λεία να χαρακτηριστούν λιγότερο ή περισσότερο τυχερά. Φυσικά το πρόβλημα εντείνεται τη στιγμή που η βαθμολογία της Α ' Λυκείου προσμετράται πλέον στην διαμόρφωση του βαθμού του απο­ λυτηρίου του Λυκείου που με τη σειρά του προ­ σμετρά στη διαμόρφωση των μορίων για την εισαγωγή για τις ανώτατες σχολές. Επίσης μεγάλο πρόβλημα παρουσιάζεται με την ανομοιογένεια των θεμάτων που τυχαίνουν με­ ταξύ των σχολείων αφού η βάση του «10» που απαιτείται στα Μαθηματικά και στα Ελληνικά (έ­ στω και με τη διαφοροποίηση της 4ης νομοθετικής

ΕΥΚΛΕΙΔΗΣ Β' 93 τ.l/43


Μ αθηματικά για την Α' Λυκείου ρύθμισης) για να προαχθεί ο μαθητής στην επόμε-

νη τάξη «καίει» πολλούς μαθητές.

4. Ένα άλλο θέμα που σχετίζεται με το προηγούμενο είναι οι διαφορετικές διευκρινήσεις που δόθηκαν καθώς και ο διαφορετικός τρόπος διόρθωσης των γραπτών από τον διδάσκοντα καθηγητή . Μήπως κάποιοι συνάδελφοι έδωσαν διευκρινήσεις που έφτασαν στα όρια της υπόδειξης; Επίσης, μήπως η διόρθωση ήταν από άλλους αυστηρή και απόν άλλους ελαστική . Υπόψη ότι στις εξετάσεις αυτές δεν υπάρχει η ασφάλεια του 2ου διορθωτή. 5. Επίσης αναφέρθηκε ότι το 4° θέμα δεν είναι κατά κανόνα συνδυαστικό. Σύμφωνα με τις οδηγίες που είχαμε πάρει, «Το τέταρτο θέμα ελέγχει την ικανότητα των μαθητών να επιλύουν προβλήματα περισσότερο σύνθετα από αυτά του τρίτου θέματος αναπτύσσοντας κατάλληλες στρατηγικές και δημιουργώντας συνδέσεις και συσχετισμούς μέσα στην ίδια μαθηματική περιοχή καθώς και μεταξύ διαφορετικών περιοχών. Ελέγχει επίσης την ικανότητα τους

να άνασύρουν τις απαιτούμενες πληροφορίες από την εκφώνηση του προβλήματος και να επιλέγουν το κατάλληλο μαθηματικό μοντέλο για την επίλυσή του. Ενδεικτικά, τα θέματα αυτού του τύπου στοχεύουν στο να ελέγξουν αν οι μαθητές είναι ικανοί: Ι.

2.

JI. Να επιχειρηματολογούν για τις επιλογές τους χρησιμοποιώντας σωστά τη μαθηματική

γλώσσα και τη μαθηματική λογική, σε περισσότερα από ένα πλαίσια (π.χ. αλγεβρικό, γεωμετρικό).»

Παραθέτουμε ένα παράδειγμα από τα 4α θέματα και αναρωτιόμαστε πως εφαρμόζονται τα παραπάνω: 4 ° Θ ΕΜ Α ( Απ λή εφ αρ μ ογή τ ύπ ων)

Σε μια ομάδα που αποτελείται από 7 άνδρες και 1 3 γυναίκες, 4 από τους άνδρες και 2 από τις γυναίκες παίζουν σκάκι. Επιλέγουμε τυχαία ένα από τα άτομα αυτά. Α) Να παραστήσετε με διάγραμμα Venn και με τη χρήση της γλώσσας των συνόλων το ενδεχόμενο το άτομο που επιλέχτηκε: ί) Να είναι άνδρας ή να παίζει σκάκι ίί) Να μην είναι άvδρας και να παίζει σκάκι Β) Να υπολογίσετε τη πιθανότητα το άτομο που επιλέχτηκε να είναι γυναίκα και να παίζει σκ α' κ:.:.:ι::.. . --..J ι_:: .:.:.= Σε ένα τυχαίο δείγμα 50 θεμάτων από τα 4α θέματα έχουμε τη παρακάτω κατανομή όσον αφορά τα κεφάλαια στα οποία αναφέρονται: -

Κεφάλ αιο

Να προσαρμόζουν τα διαθέσιμα μαθηματικά εργαλεία στις ιδιαιτερότητες της κατάστασης

ιο

0

2 0

που διερευνούν. Να συνθέτουν τις γνώσεις που διαθέτουν για

3

την επίλυση ενός μη οικείου προ-βλήματος.

4

3. Να διερευνούν ενδεχόμενα για τα οποία η εκ­

φ ώνηση δεν δίνει πληρο φ ορίες.

4. Να διαμορφώνουν ένα μαθηματικό μοντέλο για

μια σύνθετη (πραγματική ή μαθηματική) κατά­ σταση. 5. Να διερευνούν και να διατυπώνουν εικασίες τις οποίες να αποδεικνύουν επιλέγοντας την κα­ τάλληλη στρατηγική. 6. Να αξιολογούν την αξιοπιστία μιας πληρο φ ορί­ ας, την αποτελεσματικότητα μιας στρατηγικής ή την ορθότητα μιας απόδειξης. 7. Να γενικεύουν συμπεράσματα και να επεκτείνουν το πεδίο εφαρμογής τους. 8. Να προσαρμόζουν τη λύση ενός προβλήματος όταν τα δεδομένα της εκφ ώνησης μεταβάλλο ­ νται.

9. Να συγκρίνουν, να σχολιάζουν και να αντιπα­

ραθέτουν τρόπους σκέψης και στρατηγικές επί­

0

0 5 0 6 0 7 0- 0 2 4 0 0 2 -6 0- 0 3 4 0- 0 4 6

ΣΥΝΟΛΟ

Πλήθος

8 22 16 8 14 12 2 10 2 4 2 1 00

4 11 8 4 7 6 1 5 2 1 50

!2 ιο

l

8 ί

t c

Β

6

4

λυσης.

Ι Ο. Να εκφράζουν το νόημα μιας μαθηματικής

έννοιας, διαδικασίας, αναπαράστασης με περισσότερους από έναν τρόπους.

Ποσοστό f; %

α σ κήσεων ν;

6 α

11

24

• 26

I 34

• 46

Αν θεωρήσουμε τις συνδυαστικές δύο κεφα­ λαίων (συνδυαστικές τριών κεφαλαίων δεν

ΕΥΚΛΕΙΔΗΣ Β' 93 τ.l/44


Μαθη ματικά για την Α ' Λυκείου υπάρχουν) μια ομάδα τότε έχουμε τη παρακάτω κατανομή;

Κεφάλαιο Συνδυαστικές

Πλήθος ασκή-

Ποσοστό

11 4 11 8 4 7 6 1 50

22 8 22 16 8 14 12 2 1 00

ιο

Κατανομή ασκήσεων στ α κεφάλαιο

2 10 8

r--

6 4

,._,.

r--

2 ο

Γ='

α

π

Σημείωση: Προ φ ανώς η άσκηση από το 7' κεφ άλαιο θα αποσύρθηκε τελικά αφ ού το 7' κεφ ά­ λαιο ήταν εκτός εξεταστέας ύλης. 6. Ένα άλλο θέμα που αναφέρθηκε από τους συ­ ναδέλφους ήταν η διαφορετικότητα των α­ σκήσεων της τράπεζας από αυτές του σχο­ λικού βιβλίου. Στο σημείο αυτό θέλουμε να επισημάνουμε ότι σε πολλές ασκήσεις απαιτεί­ το κριτική ικανότητα από τους μαθητές, κάτι που ασφαλώς το επικροτούμε. Όμως οι μαθη­ τές της επόμενης χρονιάς θα έχουν έναν και επιπλέον χρόνο για να εκγυμναστούν στο πνεύμα αυτό τωv ασκήσεων. Παρουσιάζεται έτσι μια αδικία για τους μαθητές της φετινής χρονιάς που θα ενταθεί αν αυτό το φαινόμενο επαναληφθεί και την επόμενη χρονιά στη Β ' Λυκείου.

ΓΕΩΜΕΤΡΙΑ 1 . Υπάρχει τεράστια διαφορά δυσκολίας στα θέ­

ματα της ίδιας ομάδας (2° ή 4°) 2. Δεν καλύπτονται επαρκώς πολλά κομμάτια της ύλης (εγγράψιμα - ανισοτικές σχέσεις­ χαρακτηριστικά σημεία τριγώνου κλπ) 3. Στις εκφωνήσεις τ α δεδομένα συνήθως δίνο­ νται όλα στην αρχή και όχι όπου χρειάζονται (κατά ερώτημα) με συνέπεια να δυσκολεύει κυρίως τους αδύνατους μαθητές. 4. Δεν υπάρχει κάποια λογική για το πότε δίνε­ ται το σχήμα. Σε πολλές περιπτώσεις δυσκο-

λεύει την επίλυση απλών ερωτημάτων αφού είναι πιο πολύπλοκο. 5. Η πλειοψηφία των ασκήσεων είναι γνωστές ασκήσεις ή ασκήσεις του βιβλίου με μικρές παραλλαγές. (Σε αρκετές παραλλαγές δυσκο­ λεύει ο τρόπος που δίδεται το σχήμα). 6. Οι ασκήσεις του 4ου θέματος ανήκουν στην κατηγορία ΣΥΝΘΕΤΑ ΘΕΜΑΤΑ (τα οποία -ΣΩΣΤΑ- αναλύονται σε επιμέρους ερωτήμα­ τα)

Π ΡΟΤ ΑΣΕΙΣ ΓΙΑ ΒΕΛΤΙΩΣΕΙΣ 1 . Να γίνει επεξεργασία - ταξινόμηση των θε­ μάτων που ήδη υπάρχουν για να γίνου κατά το δυνατόν ισοδύναμα. 2. Να συμπληρωθούν όπου λείπουν και να α­ φαιρεθούν όπου είναι πολλά. Μπορούν να χρησιμοποιηθούν αρκετές ασκήσεις του βι­ βλίου με κατάλληλη επεξεργασία (υποερωτή­ ματα κλπ) αν χρειάζεται. Στη συνέχεια να τα­ ξινομηθούν ΟΛΑ τα θέματα σε ΙΣΟΔΥΝΑ­ ΜΕΣ ομάδες. (ένα 2° και ένα 4° θέμα) Σημ ε ίωση : Αν αυτό είναι δύσκολο να δοθεί η δυνατότητα σε κάθε σχολείο να κληρώνει δυο θέ­ ματα από το 2° θέμα και δυο θέματα από το 4° θέ­ μα και να επιλέγει ένα και ένα.

ΓΕΝΙΚΕΣ ΠΡΟΤΑΣΕΙΣ 1 η : Να υπάρξει αλγόριθμος συσχέτισης μεταξύ

2η :

3η :

4η :

5η :

6η :

του 2ου και του 4ου θέματος ώστε να μην υ­ πάρχει επικάλυψη της ύλης στα δύο τουλάχι­ στον θέματα. Να τονιστεί και να απαιτηθεί από τους συνα­ δέλφους (αν και δημιουργούνται λειτουργικά προβλήματα), να ολοκληρώνουν το διαγώνι­ σμα με το 1 ο και το 3° θέμα μετά την κλήρω­ ση των θεμάτων από τη τράπεζα. Να επανεξεταστούν οι ασκήσεις της τράπεζας θεμάτων ώστε: α) Να αποκτήσουν ομοιογένεια ως προς το επίπεδο δυσκολίας τους αλλά και β) Να είναι συνδυαστικές όπως πρέπει να εί­ ναι το 4° θέμα για ένα διαγώνισμα. Να μη διορθώνονται από τους οικείους διδά­ σκοντες, να επανέλθουν οι επιτροπές στα ιδι­ ωτικά σχολεία και να απαγορεύονται οι διευ­ κρινήσεις. Το σωστότερο θα ήταν να είναι κοινά θέματα για όλα τα λύκεια της χώρας με επιτηρητές από άλλα σχολεία. Να μειωθεί η εξεταστέα ύλη ώστε να είναι ε­ φικτή η ολοκλήρωσή της και να υπάρχει συγ­ χρόνως επαρκής χρόνος για επαναλήψεις. Η τράπεζα θεμάτων της Β ' και της Γ Λυκείου να ετοιμαστεί και να δοθεί στη δημοσιότητα εγκαίρως ώστε να μπορέσουν οι μαθητές να ετοιμαστούν κατάλληλα.

ΕΥΚΛΕΙΔΗΣ Β' 93 τ.l/45


Μαθη ματικά για την Β' Λυκείου

ΣΥΣΤΗΜ ΑΤΑ ΓΡΑΜΜΙ ΚΩΝ ΕΞΙΣΩΣΕΩ Ν Καμπουρίδης Γιάννης - Παράρτημα Ημαθίας

Β ' ΛΥΚΕΙΟΥ

με δυο Παρ άδειγ μα 2. Να λυθεί γραφικά το αγνώστους ονομάζουμε δυο γραμμικές παρακάτω σύστημα . εξισώσεις μαζί, των οποίων ζητάμε αν 2χ+ψ=4 (ε ι ) υπάρχουν, τις κοινές λύσεις π.χ το σύστημα 4χ+2ψ=Ι Ο (ε 2 )

{

Σύστη μα γραμμικ ών εξισ ώ σ εων

Επίλυ ση του συστήματος είναι η διαδικασία εύρεση ς των κοινών λύσεων των δυο εξισώσεων αν υπάρχουν .

ει :

2

ΓΡΑ Φ Ι ΚΗ Ε Π ΙΛΥΣΗ ΣΥΣΤΗΜΑΤΟΣ

4χ+2ψ= 1 0

Αφού οι εξισώσεις του (Σ) παριστάνουν γραφικά ευθείες (ε ι ) και (ε 2 ) αντίστοιχα τότε στο ίδιο σύστημα αξόνων . ο );;> Αν οι ευθείες τέμνονται τότε το (Σ) ο -1 3 4 5 6 έχει μοναδική λύση τις συντεταγμένες του κοινού τους σημείου (χ , ψ). Οι ευθείες είναι παράλληλες άρα το σύστημα );;> Αν οι ευθείες είναι παράλληλες τότε το είναι αδύνατο . (Σ) είναι αδύνατο .Αφού οι ευθείες δεν έχουν κανένα κοινό σημείο . Παρ άδειγ μα 3 . Να λυθεί γραφικά το );;> Αν οι ευθείες συμπίπτουν τότε το (Σ) παρακάτω σύστημα . είναι αόριστο ή έχει άπειρο πλήθος 2χ+ψ=4 λύσεων . Αφού οι ευθείες έχουν άπειρα 4χ+2ψ=8 κοινά σημεία. Παρ άδειγ μα 1 : Να λυθεί γραφικά το παρακάτω σύστημα. 2χ+ψ=8 (ε ι ) χ-ψ= l (ε 2 )

{

{

4 : t

11

-1

ει · :

3 2 ---------------------

ο

χ- ψ = 1

2

I

2χ+ ψ = 8 ο

2

5

6

7

-

1

ο -1

ο

3

4

5

6

7

Οι ευθείες συμπίπτουν άρα το σύστημα είναι Το σύστημα έχει μοναδική λύση τις αόριστο . συντεταγμένες του κοινού σημείου Α (3,2) .Δηλ. (χ,ψ)=( 3,2) ΕΥΚΛΕΙΔΗΣ Β' 93 τ.l/46


Μαθη ματικά για την Β' Λυκείου ΑΛΓΕΒΡΙΚΕΣ ΜΕΘΟΔΟΙ ΕΠΙΛΥΣΗΣ ΓΡΑΜΜΙΚΟΥ ΣΥΣΤΗΜΑΤΟΣ � Μέθοδο ς αντικατάσταση ς :

Λύνουμε την μια εξίσωση ως προς έναν άγνωστο π.χ. ως προς χ και aντικαθιστούμε στην άλλη .Λύνουμε την εξίσωση που προκύπτει και βρίσκουμε το ψ. Μετά με αντικατάσταση του ψ σε μια από τις εξισώσεις και βρίσκουμε το χ . Παράδειγμα : 2χ+ =8 <=> 2 +ψ=8 <=> 2 ( 1+ψ ) +ψ=8 χ-1+ψ χ-ψ-1 χ=1+ψ <=> 2+2 ψ+ψ=8 <=> 3ψ=6 χ=1 +ψ χ=1 +ψ <=> ψ=2 <=> ψ=2 χ=3 x=l +2 Άρα το ζεύγος (χ , ψ ) = ( 3 , 2 ) είναι η μοναδική λύση του συστήματος .

{

{ {

{�

{

{

{

συντελεστών των αγνώστων . Ορίζουμε επίσης τις ορίζουσες : γ β D = ι ι = γ ι · β 2 - γ 2 · β ι και γ2 β 2 α γ D = ι ι ι = αι · γ 2 - α 2 · γ ι α2 γ 2 Αν D :F- Ο τότε το σύστημα έχει χ

Ψ

μοναδική λύση : (χ , ψ)=

{ � , �)

Αν D = Ο , τότε το σύστημα θα είναι αδύνατο ή αόριστο . Στην περίπτωση αυτή λύνουμε το σύστημα με έναν από τους προηγούμενους τρόπους . Παράδειγ μα : Να λυθεί το σύστημα : 2χ+ψ=8 Υπολογίζουμε τις ορίζουσες: χ-ψ=1 2 1 = 2 · (- 1 ) - 1 · 1=-2 - 1=-3 D= 1 _1 8 1 = 8 . (- 1 ) - 1 · 1 =-8 - 1 =-9 D = 1 -1 2 8 = 2 1 - 1 · 8=2 -8=-6 D = ψ 1 1 Αφού D * Ο το σύστημα έχει D -9 μοναδική λύση χ= = - = 3 D -3 D -6 =2 . ψ= -ψ = D -3 •

{

� Μέθοδος αντίθετων συντελεστών :

Προσπαθούμε να κάνουμε αντίθετους τους συντελεστές του χ ή του ψ πολλαπλασιάζοντας με κατάλληλους αριθμούς τις εξισώσεις .

χ

{

Παράδειγ μα :

·

2χ+ψ=8 Αν θέλουμε να κάνουμε χ-ψ=1 αντίθετους τους συντελεστές του χ τότε πολλαπλασιάζω την πρώτη εξίσωση με το ( 1 ) και την δεύτερη με το (-2 ) τότε έχουμε : 2χ+ψ=8 · ( 1 ) 2χ+ψ=8 -2χ+2ψ=-2 χ-ψ=1 · (-2 ) Προσθέτουμε κα-�ά μέλη τις εξισώσεις και έχουμε : i 3ψτ=6 <:::> ψ=2 Αντικaθιστώ την τιμή του ψ σε μια από τις δυο εξισώσεις , προτιμώ την <=> δεύτερη και έχω : χ-2=1 χ=3 . Ι1 Άρα η λύση είναι : ( χ , ψ ) = ( 3 , 2 )

{

i

χ _

{ :

Η μέθοδος αυτή ενδείκνυται περισσότερο σε παραμετρικά συστήματα όπως για παράδειγμα : λχ+ ψ = 1 υπολογίζουμε Ι τις ορίζουσες i χ+λ ψ=- 1 λ 1 � Μέθοδο ς ο ρ ιζου σ ών : = λ 2 - 1 = ( λ - 1 } ( λ+ 1 ) D= Έστω το σύστημα : 1 λ α ι χ+ β ι ψ=γ ι με αι , α 2 , β ι , β 2 ' γ ι ' γ 2 Ε JR.D = 1 1 =' λ+1 (Σ): α 2 χ+ β 2 ψ=γ 2 -1 λ Ονομάζω ορίζουσα την παρακάτω λ 1 ορθογώνια διάταξη αριθμών : D= = -λ - 1 = - ( λ+ 1 ) 1 1 α β D= ι ι = α1 β 2 -α 2 β 1 ορίζουσα των αz β 2 .

.

{

·

{

1

'

χ

ΕΥΚΛΕΙΔΗΣ Β' 93 τ.l/47


Μαθη ματικά για τη ν Β ' Λυκείου D=O � λ2 - 1 = Ο � λ2

x

= 1 � λ= 1

Αν λ * 1 και λ * - 1 τότε το σύστημα

. ' ' D εχει μοναδ ικη λ:υση χ- - _

D

( ) -1 ψ- Ι> = ( λ- 1 ){ λ+ 1 ) = λ- 1 _

{

- λ+1

_

λ+ 1 { ){ λ- 1 λ+ 1

)=

ΠΡΟΤΕΙΝΟΜΕΝΕΣ ΑΣΚΗΣΕΙΣ

{

i)

{

Άρα το σύστημα είναι

- χ+ψ= 1

χ - ψ= - 1

Άρα τ ο σύστημα είναι αόριστο .

ΜΙΑ ΔΙΑΦΟ ΡΕ ΤΙΚΗ ΓΡΑΦΙΚΗ ΛΥΣΗ ΕΞΙΣΩΣΗΣ 2ου ΒΑΘΜΟΥ

Έστω η εξίσωση : χ 2 - χ - 2 =0 � χ 2 = χ+2 θεωρούμε το ψ=χ 2 οι εξισώσεις μή γραμμικό σύστημα : ψ=χ+2 του οποίου παριστάνουν παραβολή και ευθεία αντίστοιχα όπως φαίνεται παρακάτω.

{

5

-3

-2

-1

8χ - y = 6 x+4 y + =1 2 3

{

2(χ + 1 ) +

�=1

=

Για τις διάφορες τιμές του λ να λυθούν τα συστήματα : . 2χ + λy = 4 . . λ 2 χ + λy = ι ι) η) λχ + 2y = λ χ + λy = λ 2χ + y + 7 = λ(χ + 2) . . . λχ + 2y = 2χ . ηι ) ιν) 2x + λy = 2y (λ + 2)y = 5 (χ + ι) 2.

{ {

{ {

z

3 . Αν για ένα σύστημα δυο γραμμικών εξισώσεων με δυο αγνώστους χ και y ισχύει : 2D x = 3D y = ι 2D -:/: Ο . Να λυθεί το σύστημα αυτό . 4

. Αν για ένα σύστημα δυο γραμμικών εξισώσεων με δυο αγνώστους χ και y ισχύει : D 2 + D � + D � 2D Y - 4D - 5 .

--------------

ο

IV )

!

10

3x - y = 4 x+y=l vi) -9x + 3y = 9 5x + 5y 5 5 3 8 3 - - - = -ι - + - = -2 χ Υ 6 χ Υ vii) viii) 9 4 -2 + -9 = -5 ---= 6 χ Υ 6 χ Υ

� Οχ+Οψ=Ο

4

ii)

{

)

ν

αδύνατο .

Αν λ=- 1 το σύστημα γίνεται :

3χ - y = 5

7

.

. χ+ψ= 1 ·1 � � Οχ+Οψ=2 χ+ψ=- 1 · (- 1 ) -χ-ψ= 1

χ+ψ= 1

{

1 . Να λυθούν τα συστήματα : 5 χ - 2y = 4 χ + 3y =

1 λ- 1

Αν λ= 1 το σύστημα γίνεται :

{

ή λ= - 1

=

2

3

Οι λύσεις του συστήματος είναι : ( χ ,ψ ) = ( - ι , ι ) και ( χ , ψ ) = ( 2 , 4 )

4

i)

Να αποδείξετε ότι : (D + 2) 2 + D � + (D y - ι ) 2 = 0

ii)

Να λυθεί το σύστημα αυτό .

! ! )

j ι

5 . Να λυθούν τα συστήματα

i) ...

ιη

:

2y-x 2 =0 y=2x 2 ii) x=y-4 4x-y=2 x 2 +y2 =2 . xy-2y=x-4 ιν) 2χ-y=ι χy=-ι

ΕΥΚΛΕΙΔΗΣ Β' 93 τ.l/48


Μαθη ματικά για την Β' Λυκείου

Β ' ΛΥΚΕΙΟΥ

Τριγωνομετρ ι κές εξ ισώσεις

Σίσκου Μαρία 1.

Να λυθεί η εξίσωση

ημ(χ-!!.) ημχ

4

ή Ο < κ - !4 < 1 και κ ε Ζ � κ=Ο ή στο 3° κ=l . 3π δ εκτες. , , Επομενως χ = 2π η, χ = 4π η, χ = 4

= -1

Λύση: Πρέπει ημχ* Ο � χ* κπ, κ ε Ζ ημ (χ- �) π = - ημχ � ----'--=- = - 1 �ημ(χ- -) 4 ημχ Ζ:) = ημ(-χ) � χΖ:4 = 2κπ -χ ή ημ(χ- 4 χ- Ζ:4 = 2κπ+ π + χ κ ε Ζ, �2χ= 2κπ �4 , κ ε Ζ ή Οχ= 2κπ+ π � Αδύνατη χ=κπ� , κ ε Ζ δεκτές . 2.

5.

'71 ιu

=

6.

ιu

Να λυθεί η εξίσωση - 300σφχ+ 300σφ2χ - 100σφ3χ - 100 =

'71 ιu

7.

Να λυθεί η εξίσωση εφ(χ- �) σφ(2χ �)

8.

Να λυθεί η εξίσωση σφχ·συνχ -1 =ημχ

=

Λύση: Για να ορίζεται η εφ(χ- �) πρέπει συν(χ- �)* Ο και για να ορίζεται η σφ(2χ�) πρέπει ημ(2χ�)* Ο . 4π και Το συν(χ- 3π) * Ο � χ * 2κπ � Sπ το ημ(2χ �π)* Ο � χ * κπ - π η, χ* κπ+ 12 . 2 1 εφ(χ- 3π) = σφ(2χ �π) �εφ(χ- 3π)= εφ[2π -(2χ�π)]� 2π 2π π π => εφ(χ- 3)=εφ(6 -2χ)�χ- 3 -κπ + 6 -2χ, κ ε Ζ κπ 2 π 2 π , κ ε Ζ, χ= 3 ' . + 9, κ ε Ζ δεκτες 3χ= κπ �

Να αποδείξετε ότι l η μ2χ- Sημχ + 51 ::; 11 για κάθε χ ε IR 3.

Λύση : Γνωρίζουμε ότι : Ι α+ β l $ Ι α l + l β l και 1 ημχ l:5 1 Άρα lη μ2χ- 5ημχ + 5 1 $l η μ2χ i + Ι - 5ημχ l + I 5 1 => lη μ2χ- 5ημχ + 5I $ 1 + 5 + 5� lη μ2χ- 5ημχ + 5I$ 1 1 . 4. Να λύσετε την εξίσωση στο (Ο,π) εφ2οιι;(χ- z:) + σφ2οι4χ = 0 2

2

'

Λύση:Με ημχ* Ο ή χ* κπ έχουμε -300σφχ+ 300σφ2χ - 1 00σφ3χ = : 100 � 1 00- 300σφχ+ 300σφ2χ - 1 00σφ3 χ = Ο � 1 00(1- 3σφχ+ 3σφ2χ - σφ3χ) = ο� 1 �φχ+ 3σφ2χ - σφ3χ = Ο � (1 -σφχ)3= ο � 1 -σφχ= ο �σφχ= ι � ' σφχ= σΨ4π � χ=κπ + 4π , κ ε δεκτες.

'71 ιu

'71 ιu

6εφ2χ +εφ4χ = Ο =

Λύση: Μ ε συνχ* Ο Ξ$ χ * (2κ + 1)π έχουμε ../3 Ο και (ημχ-z../3-)2 + Ι εφ2χ -3 1 =Ο �ημχ - 2 εφ2χ-3=0�ημχ= ../3 και (εφχ+ νf3)·(εφχ- vf3)=0� 2 ημχ= ημΖ: και εφχ=-v13 ή εφχ= vf3 � 3 χ=2κπ �π η, χ=2κπ +π- 3π , κε και εφχ = - εΨJπ η, 2 π κε Ζ εφχ=εφ3π �χ=2κπ �π η' χ=2κπ + 3, εφχ = εφ(- 3π) ;η' εφχ=εΨJπ �χ=2 κπ �π η' 2 π κε Ζ και χ=κπ -π η' χ=κπ+ -π κ ε Ζ � χ=2κπ + ' 3' 3 3 χ= 2κπ+i , κε Ζ δεκτές .

_

9-

Λύση : Με συνχ* Ο Ξ$ χ * (2κ + 1)π έχουμε 9- 6εφ2χ +εφ4χ = Ο �3 2 -2 · 3 εφ2χ + (εφ2χ)2 =Ο � (3-εφ2χ)2=0�(3-εφ2χ)=Ο� vf32 - ε φ 2 χ Ο � (vf3 -εφχ)(v13 + εφχ) = Ο �εφχ= vf3 ή εφχ= -vf3 �εφχ= εφΖ: ή εφχ= εφ(- Ζ:) � 3 3 ' . χ=κπ+ 3π , κ ε η χ=κπ- 3π , κ ε '71 δ εκτες

Να λυθεί η εξίσωση (ημχ -:)2 + Ι εφ2χ -31 =Ο

Λύση: Για να ορίζεται η εφ(χ- �) πρέπει συν(χ- �)* Ο και για να ορίζεται η σφχ πρέπει ημχ* Ο. Το συν(χπ 3π 2)* Ο �χ* 2κπ +z και το ημχ* Ο �χ*κπ. Επομένως: εφ2οι6(χ- z:) + σφ2οι4χ = 0 � 2 εφ2οι6 [-( z:2 -χ)] + σφ2οι4χ = 0 � -σφ2οι6χ + σφ2οι4χ = 0 �-σφ2οι4χ (σφ2χ 1) = 0 � σφχ= Ο ή σφχ = Ι ή σφχ = - 1 � σφχ= σφ-π η, σφχ= σφ-π4 η, σφχ= σφ ( - -π4 ) � 2 π κε και χε (Ο ,π) χ=κπ +zπ η, χ=κπ +;jπ η, χ=κπ - 4, Άρα Ο< χ< π �ο < κπ+Ζ: <π ή Ο < κπ�4 <π 2 ή ο < κπ-Ζ:4 <π �ο < κ� <1 ή ο < κ + !4 < 1

Να λυθεί η εξίσωση

Λύση: Πρέπει ημχ* Ο Ξ$ χ * κπ χ σφχ·συνχ -1 =ημχ � -·συνχ = ημχ + 1 � χ ημ συν2χ = ημ2χ +ημχ � l -ημ2χ= ημzχ +ημχ � 2ημ2χ +ημχ- 1 = Ο . Θέτουμε ημχ= y 2y2+ y -1 = Ο � y= -1 ή y= ! 2 ημχ= - 1 η' ημχ- 21 ' ημχ=ημ ( - 2π) η' ημχ=ημ6π χ= 2κπ - 2π η, χ= 2 κπ+zπ η, χ=2κπ �π ' . S π , κ ε δ εκτες η, χ= 2 κπ � συν

·

ΕΥΚΛΕΙΔΗΣ Β ' 93 τ.l/49

'71 ιu


Μαθη ματικά για την Β' Λυκείου

Β ' ΛΥΚΕΙΟΥ

Εγγράψιμα τετράπλευρα Ευαγγελόπουλος Αναστάσιος

Σύμβουλος Μαθηματικών Ανατολικής Θεσσαλονίκης Μαθηματικός MSc Στατιστικής και Επιχειρησιακής Έρευνας Α

Ασ κή σεις

_..._

1) Σε τρίγωνο ΑΒΓ με Α = 45° , φέρουμε τα ύψη

Δ

ΒΔ και ΓΕ. Αν το σημείο Μ είναι μέσο της ΒΓ δείξτε ότι

......_

ΔΜΕ 90° . =

Β

Α

Μ

2) Δίνεται τρίγωνο ΑΒΓ εγγεγραμμένο σε περιφέρεια (C) .Παίρνουμε τα μέσα Δ και Ε των τόξων ΑΒ και ΑΓ αντίστοιχα , φέρουμε την χορδή ΔΕ η οποία τέμνει την ΑΒ στο Μ και την ΑΓ στο Ν. Δείξτε ότι ΑΝ=ΑΜ.

4) Δίνεται τρίγωνο ΑΒΓ, τα ύψη του ΑΔ, ΒΕ και Μ σημείο της ΕΓ τέτοιο ώστε ΑΕ=ΕΜ και Η το ορθόκεντρο του ΑΒΓ. Να δείξετε ότι: α) Το τετράπλευρο ΑΒΔΕ είναι εγγράψιμο. β) Οι γωνίες ΕΑΔ και ΕΒΔ είναι ίσες. γ) Το τρίγωνο ΑΗΜ είναι ισοσκελές. δ) Το τετράπλευρο ΗΜΓΒ είναι εγγράψιμο. Α

Α

_..._

5) Σε ορθογώνιο τρίγωνο ΑΒΓ ( Α "'

=

90° )

έχουμε

ότι Β = 30° . Φέρουμε το ύψος ΑΗ και τη διάμεσο ΑΜ του τριγώνου ΑΒΓ . Από την κορυφή Β φέρνουμε κάθετη στη διάμεσο ΑΜ, η οποία την τέμνει στο σημείο Ε. Να αποδείξετε ότι: α)

ΒΕ

ΑΒ

= -

2

, β) ΑΗ=ΒΕ, γ) Το τετράπλευρο

ΑΗΕΒ είναι εγγράψιμο, δ) ΕΗ//ΑΒ .

Γ

3) Iσόπλευρο τρίγωνο ΑΒΓ είναι εγγεγραμμένο σε κύκλο . Παίρνουμε τα μέσα Δ και Ε των τόξων ΑΒ και ΑΓ αντίστοιχα. Δείξετε ότι το ευθύγραμμο τμήμα ΔΕ τριχοτομείται από τις πλευρές ΑΒ και ΑΓ. Β

ΕΥΚΛΕΙΔΗΣ Β' 93 τ.l/50

Α


Μαθη ματικά για την Β' Λυκείου 6) Δίνεται τρίγωνο ΑΒΓ είναι εγγεγραμμένο σε

κύκλο (O,R) και Δ το αντιδιαμετρικό σημείο του Α. Φέρουμε ακτίνα ΟΖ ..L ΑΓ που τέμνει την ΑΓ στο Μ . Αν η εφαπτομένη του κύκλου στο Δ τέμνει την ευθεία ΑΓ στο Ε , να αποδείξετε ότι : α) Το Μ είναι μέσο της ΑΓ. β) Το τετράπλευρο ,._

ΟΜΕΔ είναι εγγράψιμο. γ)

"'

ΑΟΜ = ΔΕΓ = Β .

7) Δίνεται η γωνία xOy , ένα σημείο Ε πάνω στη

διχοτόμο της Οδ και δύο σημεία Α και Β πάνω στην Οχ . Οι περιγεγραμμένοι κύκλοι στα τρίγωνα ΟΑΕ και ΟΒΕ ,τέμνουν την Oy στα σημεία Ζ και Η αντίστοιχα . Να αποδείξετε ότι ΑΒ=ΗΖ.

9) Πάνω σε μία ευθεία παίρνουμε τέσσερα σημεία Α, Β, Γ, Δ έτσι ώστε ΑΒ=ΒΓ= Γ Δ. Με πλευρά την ΒΓ κατασκευάζουμε ισόπλευρο τρίγωνο ΜΒΓ, κατόπιν φέρουμε την ευθεία ΑΜ , η οποία προεκτεινόμενη τέμνει την κάθετο στην ΑΔ που άγεται από το σημείο Δ στο σημείο Ν. Να αποδείξετε ότι : α) ΑΜ= ΜΝ. β) ΒΜ / / ΓΝ . γ) Το τετράπλευρο ΜΓΔΝ είναι εγγράψιμο.

Ε

Β

Α

Ν

Δ

Γ

1 0) Έστω Α, Β, Γ συνευθειακά σημεία με ΑΒ=2ΒΓ. Θεωρούμε το μέσο Μ της ΑΒ . Προς το ίδιο ημιεπίπεδο κατασκευάζουμε τα ισόπλευρα τρίγωνα ΑΔΒ και ΒΕΓ. Να αποδείξετε ότι: α) Το τετράπλευρο ΑΔΕΒ είναι τραπέζιο (ΑΔ//ΒΕ). β) Τα τρίγωνα ΔΜΒ και ΔΕΒ είναι ίσα. γ) Το τετράπλευρο ΔΜΒΕ είναι εγγράψιμο. Δ

8) Σε κύκλο (O,R) φέρουμε τις χορδές ΑΒ και ΑΓ

και την εφαπτομένη του ΑΕ στο Α. Επίσης φέρουμε μια χορδή παράλληλη προς την εφαπτομένη ΑΕ, η οποία τέμνει τις ΑΒ και ΑΓ στα σημεία Δ και Ζ αντίστοιχα . Να δείξετε ότι τα σημεία Β, Γ, Ζ και Δ ορίζουν εγγράψιμο τετράπλευρο. Α

ΕΥΚΛΕΙΔΗΣ Β' 93 τ.l/51

Μ

Β

Γ


Μ αθηματικά για την Β ' Λυκείου 1 1) Δίνεται οξυγώνιο τρίγωνο ΑΒΓ. Κατασκευάζουμε εξωτερικά του τριγώνου τα ισόπλευρα τρίγωνα ΑΕΒ ,ΑΓΔ .Ονομάζουμε Ζ το σημείο τομής των ευθυγράμμων τμημάτων ΒΔ, ΓΕ. Να αποδείξετε ότι: α) Τα τρίγωνα ΑΕΓ και ΑΒΔ είναι ίσα και να γράψετε τα ζεύγη των ίσων γωνιών. β) Τα τετράπλευρα ΑΖΓΔ ,ΑΖΒΕ είναι ......

εγγράψιμα. γ) Η γωνία

Β Ζ Γ = 1 20° .

ο

Ε

Γ

1 2) Θεωρούμε ισόπλευρο τρίγωνο ΑΒΓ και τα σημεία Δ και Ε των πλευρών ΑΒ και ΑΓ αντίστοιχα ώστε να είναι ΑΔ= ΓΕ . Έστω Ο το σημείο τομής των ΓΔ και ΒΕ. Να αποδείξετε ότι: '"'

......

α)

ΒΕ Γ = ΓΔΑ . ΒΟ Γ = 1 20° .

1 4 ) Δίνεται τετράπλευρο ΑΒΓΔ και ο περιγεγραμμένος του κύκλος (Ο,ρ) ώστε η διαγώνιος του ΔΒ να είναι διάμετρος του κύκλου. Η γωνία Β είναι διπλάσια της γωνίας Δ και οι πλευρές ΑΒ και ΒΓ είναι ίσες. Φέρουμε κάθετη στη ΒΔ στο Ο , η οποία τέμνει τις πλευρές ΑΔ και ΓΔ στα σημεία Ε και Ζ αντίστοιχα. α ) Να υπολογίσετε τις γωνίες του τετραπλεύρου ΑΒΓΔ .

.-..

β)

γ) Να εξετάσετε αν το τετράπλευρο ΑΕΟΔ είναι εγγράψιμο. Α

β) Να συγκρίνετε τα τρίγωνα ΔΑΒ και ΔΓΒ.

γ) Να αποδείξετε ότι το τετράπλευρο ΑΒΓΟ είναι ρόμβος. δ ) Να αποδείξετε ότι το τετράπλευρο ΑΒΟΕ είναι εγγράψιμο σε κύκλο. Α

Β

Γ

Ι\ I \

13) Δίνεται κύκλος με κέντρο Ο και ακτίνα ρ.

\ \ι.

Έστω σημείο Α εξωτερικό του κύκλου και τα εφαπτόμενα τμήματα ΑΒ και ΑΓ ώστε να ισχύει -

ΒΑ Γ 60° . =

1

Έστω η εφαπτόμενη του κύκλου

στο Δ τέμνει τις ΑΒ και ΑΓ στα Ε και Η αντίστοιχα . Να αποδείξετε ότι: α) Το τετράπλευρο ΑΒΟΓ είναι εγγράψιμο με ΟΑ=20Β. β) Το τρίγωνο ΑΕΗ είναι ισόπλευρο. γ) 2ΖΒ=ΑΖ. δ) Το τετράπλευρο ΕΗΒΓ είναι ισόπλευρο τραπέζιο.

ΕΥΚΛΕΙΔΗΣ Β' 93 τ.l/52

I

I/

z

/ Γ


:\Ιαθηματικά κατεύθυνσης για την Β' Λυκείου

Δ Ι Α Ν Υ Σ Μ Α Τ Α

Θανάσης Χριστόπουλος Από την πρώτη τάξη είναι απαραίτητες κάποιες μαθηματικές έννοιες για τη διδασκαλία άλλων μα­ θημάτων. Στα αναλυτικά προγράμματα των μαθηματικών δεν έχει προβλεφθεί αυτό ώστε να προηγείται η διδασκαλία στα Μαθηματικά. Τα διανυσματικά μεγέθη είναι απαραίτητα στη Φυσική από την πρώ­ τη τάξη, γι' αυτό γίνεται μια αποσπασματική ενημέρωση από τους Φυσικούς. Αυτό όμως έχει τη θετική αλλά και την αρνητική του πλευρά. Στη Β ' τάξη λοιπόν γίνεται η διδασκαλία των διανυσμάτων. Είναι γνωστή η θεωρία από το σχολικό βιβλίο αλλά κάποια σημεία χρειάζονται ιδιαίτερη προσοχή για την κατανόηση τους αυτά είναι:

l)

Πρόσθεση-αφαίρεση 2 διανυσμάτων i) ΑΒ + ΑΓ = ΑΔ ΑΒ + ΑΓ = 2 · ΑΜ ή (Μ, μέσον ΒΓ)

ii) ΑΒ - ΑΓ = ΓΒ

-

-

-

-

-

iii) ΑΒ + ΒΓ = ΑΓ κάθε άλλη περίπτωση ανάγεται σε αυτά. -

-

-

Παράδειγμα: ΑΒ - ΒΓ = - (ΒΑ + Β Γ) •

ΑΓ - ΒΓ = ΑΓ + ΓΒ = ΑΒ

α

προκύπτει διάνυ­

σμα συγγραμμικό με αυτό και αντίστροφα

α.

3) Αν δύο διανύσματα είναι συγγραμμικά τότε

υπάρχει πραγματικός αριθμός τέτοιος ώστε το γινόμενό του με το ένα διάνυσμα να μας δίνει το άλλο. 4 ) Αν πολλαπλασιάσουμε ένα διάνυσμα με το μηδέν, τότε προκύπτει το μηδενικό διάνυσμα το οποίο έχει οποιαδήποτε κατεύθυνση. 5) Αν α , είναι μη συγγραμμικά διανύ σματα και

β

ισχύει κ : α + λ ·

β

κ, λ ε JR τότε κ=λ=Ο α = ( x0 , y0 ) τότε

= δ με

6) Αν δοθεί διάνυσμα

I α I=

Jx � + Υ�

Μέτρο του α :

Συντελεστής διεύθυνσης: λ α-

·

α = Ο <:::::> χ 0 = y0 = Ο α ι ι χ 'χ <=::> y 0 = 0 , α ι ι y 'y <=::> χ 0 = 0 7) Αν δοθούν 2 διανύσματα: α = (χ1 , y1 ) και β = ( χ2 , y2 ) τότε α = β <:::> χ1 = χ2 και Υι = y2 α + β = ( χ, + Χυ Υι + yz ) α - = ( χ, - Χz , Υι - yz ) Υ Αν α ι τότε •

β /β

Υ

• •

λ α = λ β , χ, , χ2 "# Ο ή τα διανύ σματα είναι

κατακόρυφα α · ι β ι . Ειδικότερα αν α . β =

ι α .βι ι ι =

β ενώ αν

β Αν α .1 β τότε: α·β=Ο α t..ι,

λ -α · λβ

=

-

α . β = - J αJ · Ι βl

J� · Ι� τότε

1 ή ένα είναι οριζόντιο και το

άλλο κατακόρυφο αρατή ρηση : Οι παραπάνω προτάσεις ισχύουν Π και αντίστροφα. Α = (χ , , y , ) και 8) Αν δοθούν 2 σημεία Β = ( χ2 , y 2 ) τότε είναι γνωστά και τα παρακά­ τω: • Το διάνυσμα ΑΒ = ( χ2 - x , , y2 - y, ) •

0 =Υ Χο

(όταν χ 0 "# Ο , ενώ όταν χ 0 = Ο τότε το διάνυσμα είναι κατακόρυφο) κ α = ( κχ 0 , κy0 ) , κ ε JR

� 2

τότε α tt

2) Όταν πολλαπλασιάσουμε ένα μη μηδενικό αριθμό με ένα διάνυσμα

· α , κΕR (ειδικότερα αν κ>Ο είναι β ενώ αν κ<Ο, α t..!, β)

ι:�� ι = Ο α tt

-

ΑΓ + ΒΓ = -(ΓΑ + ΓΒ)

β=κ

Η απόσταση των δύο σημείων, που ισού­ ται με το μέτρο του διανύσματος δίνεται ΑΒ τύπο τον από

J J = �(x z - x , )z + (Yz - y, )z

(AB ) AB =

Ο συντελεστής διεύθυνσης λ = ΑΒ αν

Το

Yz - y, Xz - χ,

χ, "# χ2 μέσον

Μ

του

ΑΒ

είναι:

Μ χ' + Xz Υι + Yz ) ( 2 ' 2

9) Για να βρούμε το εσωτερικό γινόμενο δύο δια­ νυσμάτων έχουμε τις παρακάτω δυνατότητες:

β = ιαι ·J βJ συν(�)

Με τον ορισμό α ·

Με την αναλυτική έκφραση του εσωτερια = ( χ 1 , y 1 ) και Αν γινομένου. κού

ΕΥΚΛΕΙΔΗΣ Β' 88 τ.4/53


Μ αθη ματικά για τη ν Β · Λυκείου

β = (χ 2 , y2 ) τότε α . β = χ 1 · Χ 2 + y1 · y2

Με αντικατάσταση του ενός διανύσματος από την προβολή του πάνω στο άλλο διάνυσμα:

α.β = α·Πf'Ο/3αβ ή α . β = β . προββ α

Από γνωστές σχέσεις μεταξύ διανυσμάτων Αντίστοιχα παραδείγματα ί. Αν

ι αι = 2 ,

τότε

α . β = -2 . 4 + 3 . 2 = -2 το αρνητικό αποτέλε­ σμα φανερώνει, ότι η γωνία των διανυσμάτων είναι αμβλεία ίίί. Αν ΑΒΓ ορθογώνιο τρίγωνο με Α = 90° και να βρεθεί το ΑΒ · ΒΓ είναι ΑΒ · ΒΓ = ΑΒ · προ β ΒΓ = ΑΒ · ΒΑ = 2 ΑΒ · ΑΒ -- = - I ΑΒ 1 = - 82 = -64 Αν 2<i -β -y = O και α,β,γ μοναδιαία, να βρε­ θεί το εσωτερικό γινόμενο α · β . Είναι - -- 2<i - β = γ οπότε (2ii-βf =f <=:>4α2 -4α· β+β2 =Υ 2 2 ισοδύναμα 4Jαι - 4α . β+ J β J = l γl 2 � α . β = 1

1 0) Π αρατήρηση : Πολλές ιδιότητες που μάθαμε

στους αριθμούς, δεν ισχ6ουν στα διανύσματα, έτσι κάποια από τα παρακάτω μας ξαφνιάζουν Α ) Γ εν ικ ά • •

• •

--

-

--

u 4· ΓΔ

ii) Αν Λύση i)

ΑΒ

iv.

αυτό το κλάσμα έχει νόημα γιατί οι όροι

α·γ

i) Να δειχτεί ότι το u είναι ανεξάρτητο από τη θέση του σημείου Ο.

-

γ=8

·�

του είναι αριθμοί. Δεν μπορούμε όμως να a­

β = (4, 2) τότε

α = ( -2, 3) και

πλοποιήσουμε το α . Β) Ασκήσεις Άσκηση l η Θεωρούμε τρίγωνο ΑΒΓ και Δ το μέσον της πλευράς ΑΒ. Αν Ο, τυχαίο σημείο και U = X · OA + y · OB + z · OΓ με x, y, z ε lR και x+y+z=O -

- · β- 2 · 3 · συ -π = 2 · 3 · -1 3 ν3 α = 2=

ίί. Αν

=

u = x · OA + y · OB + (-x - y) · OΓ � u = x · OA + y · OB - x · OΓ - y · OΓ � u = x · OA - x · OΓ + y · OB - y · OΓ � u = χ · (ΟΑ - ΟΓ) + y · (ΟΒ - ΟΓ) � u = χ · ΓΑ + y · ΓΒ u = 4, ΓΔ � χ · ΓΑ+y·ΓΒ = 2 · 2ΓΔ � χ · ΓΑ + y · ΓΒ = 2 · (ΓΑ + ΓΒ) � (χ - 2) · ΓΑ + (y - 2) · ΓΒ = Ο � χ = y = 2 -

ii)

να υπολογιστούν τα x,y,z

-

-

-

--

-

-

-

-

-

-

-

Άσκηση 2 η Αν για τα μη συγγραμμικά ανά δύο διανύσματα α, β, γ ισχύουν:

α ι /6β - y .

α - 2β I ι γ

και

3α + y I ι β

τότε

Λύση <α · β) . γ * α . <β . γ) -α - 2β- I /γ- προκύπτει η ύπαρξη κ Ε JR ώστε Από ( α · β ) 2 ;e α 2 · β 2 διότι α-2β = κΎ ή α = 2β+κΎ ( 1 ) <α . β)2 = <Ιαi · Ι βl συνφ)2 = Ια1 2 · Ι β1 2 συν2φ Από 3α +γ I ι β προκύπτει η ύπαρξη λ Ε JR ώστε 2 2 2 2 Ενώ α · β = ι α ι · J βJ επομένrος θα ισχ6ει η 3α + γ = λβ � 3α = λβ - γ (2) . Από ( 1 ),(2) προ­ 6β + 3κγ = λβ - y � (3κ + l)γ = (λ - 6)β ισότητα όταν crυ ν 2 φ =1 δηλαδή όταν φ=Ο ή κύπτει φ=π ή α ι ιβ οπότε κ = _.!_ και λ=6. Από τη σχέση (2) έχουμε 3 ι α . βj 'Φ ια ι · ιβι , γενικά ισχι)ει: ια . βj ::; ιαι · ι β ι , 3α = 6β - γ � α ι /6β - γ αφού JσυνφJ::;1 Αν α· γ = β· γ δεν συνεπάγεται α = β . Συγκε­ Άσκηση 3 η κριμένα έχουμε αν α.γ-β.γ = Ο�(α-β).γ = Ο Αν τα διανύσματα a, β,γ,χ ικανοποιούν τη σχέ­ οπότε α - β .i γ ή α = β ή γ = δ . ση a + χ = (β . χ)γ (1) να δείξετε ότι: Δεν ορίζεται η διαίρεση α : β i) a . β = (β · i)(β . γ - 1) -3

-4

Δεν ορίζονται οι δυνάμεις α , α , ... παρά μό-

-- νο το α 2 που σημαίνει α · α = α 2

ii) Αν β · γ "Φ 1 να εκφράσετε το χ σαν συνάρτη­

ση των a, β,γ

Λύσ η i) Πολλαπλασιάζουμε και τα δύο μέλη της ( 1 ) επί

ΕΥΚΛΕΙΔΗΣ Β' 93 τ.l/54


Μαθη ματικά για την Β' Λυ κείου

β οπότε α . β + β . χ = (β . x)(y . β) άρα (β . χ )(γ . β - 1) = α . β α β οπότε με αντικα ii) από i προκύπτει β · χ = _ _" ­ γ·β-1 α β τάσταση στη σχ έση. ( 1 ) έχ ουμε χ = - _. . γ - α γ·β-1 Άσκηση 4 η

2α + β = u -

-

-

(1),

α-β= v

-

-

-

(2)

� � = 1 2 , � �� = 3J3 και (� , � ) = : να βρεθούν

Αν �

τα μέτρα των

α, β

ΑΓI

I

ίί) Υπολογίστε το ΑΒ + ίίί) Να βρείτε τις συντεταγμένες του σημείου Α iv) Δείξτε ότι το τρίγωνο ΑΒΓ είναι ορθογώνιο και ισοσκελές .

Άσκηση 2 η

Δίνονται τα διανύσματα ΟΑ = 5Ci + 8β - 9y , ΟΒ = 9Ci + 2β + y και ΟΓ = 3Ci + 1 1β - 14y . Να αποδείξετε ότι τα σημεία Α,Β,Γ είναι συνευθειακά και το σημείο Α βρίσκεται μεταξύ των Β ,Γ το ο­ ποίο χ ωρίζει σε λόγο 2: 1 Άσκη ση 3 η

ιι Ι1

II

Αν α = β = 5 και Ύ = J7 με 2α + 3β - 5γ = δ ί) Υπολογίστε τη γωνία (α, β) ίί) Αν ΟΑ = α , ΟΒ = β , ΟΓ = γ δείξτε ότι τα Α , Β ,Γ είναι συνευθειακά ---

Λύση

Από το σύστημα των (1),(2) προκύπτει α = .!. (u + v) και β = .!.ίi - � v αφού u = 1 2 , Άσκη ση 4η 3 3 3 Αν A(2,-2 + .J3) , Β(1 ,- 2), Γ(l, -2 - .J3) , να υv = 3.J3 και (�) = είναι ίi - v = 12 · 3..f3 · συν πολογίσετε τη γωνία Β του τριγώνου ΑΒΓ. J3 1 - - προ- Άσκηση 5η = 12 · 3-νj� · = 36 . Από τη σχ έση -α=-(u+ν) Δίνονται τα διανύσματα α = (1 , 2) και β = (2, 3) . 3 3 Να βρείτε -- - 1 - ;vJ 1 κύπτει α 2 = -(u + ν) 2 <:::::> ι; 2 =-(U2 +2 · U · ν+ν2 ) ί) το μέτρο του διανύσματος γ = sα - 3β 9 9 από και προκύπτει ii) τη γωνία που σχηματίζει το γ με τον άξονα αυτό χ 'χ ιαι 2 = .!. (144 + 2 · 36 + 27) = 243 = 27 άρα ιαι = 3.J3 iii) τον πραγματικό αριθμό κ, ώστε το διάνυ σμα 9 9 ίi = ( κ 2 - κ, κ) να είναι κάθετο στο α Άσκηση 5η iv) την προβολή του α πάνω στο β Αν Κ(-3,2) το σημείο τομής των διαγωνίων ενός

ll

li

παραλληλογράμμου ΑΒ

= (1, 4)

ΑΒΓ Δ

με

Α(-5,4)

και

να βρεθούν οι συντεταγμένες των

κορυφών Β,Γ,Δ. Λύση

Με τη βοήθεια των διανυσμάτων μπορούμε να αποδείξουμε και τις προτάσεις της γεωμετρίας όπως: •

Αφού Κ μέσον της ΑΓ θα είναι -5 + χr = -3 2 4+y ' Γ ( -1, ο) . <:::::> Xr = -1 και τ = 2 <:::::> Yr =0 αρα Για την κορυφή Β έχ ουμε ΑΒ = (1, 4) <:::::> χ8 - χΑ = 1 άρα χ 8 = -4 και Υ Β - Υ Α = 4 <:::::> Υ 8 = 8 άρα Β(-4,8) και για την κο­ ρυφή Δ έχ ουμε ότι Κ μέσον της ΒΔ, οπότε Δ(-2,-4)

Να δειχ τεί ότι τα μέσα των πλευρών ενός τετραπλεύρου σχηματίζουν παραλληλόγραμμο

Α πό δειξη

Αν ΑΒΓ Δ το τετράπλευρο και Κ,Λ,Μ,Ν τα μέσα των πλευρών ΑΒ,ΒΓ,ΓΔ,ΔΑ αντίστοιχ α, για να δείξουμε ότι ΚΛΜΝ παραλληλόγραμμο αρκεί να δείξουμε ότι ΚΑ ΝΜ . Αν Ο οποιοδήποτε ση­ μείο του επιπέδου σχ έση γίνεται ΟΒ + ΟΓ ΟΛ - ΟΚ = ΟΜ - ΟΝ <:::::> 2 ΟΑ + ΟΒ = ΟΓ + ΟΔ ΟΑ + ΟΔ <:::::> Ο = Ο 2 2 2 =

-

-

-

-

-

-

-

-

_

Άσκηση 1 η

ΑΣΚΗΣΕΙΣ ΓΙΑ Λ ΥΣΗ

Δίνεται τρίγωνο ΑΒΓ με Β(α-2,β), Γ(α-4,β+2) , αν Μ(α-3,β+ 1) όπου α,βΕR σημείο τέτοιο ώστε ΑΜ=( 1 , 1 ) i) Δείξτε ότι το Μ είναι μέσο του ΒΓ

1) 2) 3)

Π ΡΟ Τ Α Σ Ε Ι Σ Π ΡΟ Σ Α Π Ο ΔΕ ΙΞ Η

Να δειχ τεί ότι οι παρά τη βάση γωνίες ισο­ σκελούς τριγώνου είναι ίσες. Να αποδειχ θεί ο νόμος συνημιτόνων Να αποδειχ τεί το πρώτο θεώρημα διαμέσων .

ΕΥΚΛΕΙΔΗΣ Β' 93 τ.l/55


Μαθη ματικά για την Β ' Λυ κείου ΓΡΑΠΤΗ

ΕΞΕΤΑΣΗ ΣΤΗ Ν λλΓΕ ΒΡΑ της Β 'ΛΥΚΕΙΟΥ

Τσαγκά ρης Κωνσταντίνος Κεφάλαια: Ι Γραμμικά Συστήματα, 2: Ιδιότητες συναρτήσεων(Από Α ' Λυκ θεωρία Κεφ 7) Χρόνος: δύο διδακτικές ώρες

Θ Ε Μ Α 1 °. Α . Να εξετάσετε

αν ο ισχuρισμός « μια συνάρτηση f με πεδίο ορισμού το σύνολο Α, παρουσιάζει στο Χ0ΕΑ (ολικό) μέγιστο, αν f(x):?: f(xo) για κάθε Χ0ΕΑ ». είναι σωστός ή λάθος; (Μονάδες 7) Στην περίπτωση που απαντήσετε λάθος να διατυπώσετε το σωστό. Β. Να χαρακτηριστεί η κάθε πρόταση ως Σωστή Σ D ή Λάθος Λ D α) Η συνάρτηση f(x) = αχ + β με α > Ο είναι γνησίως φθίνουσα στο IR. (σελ 32Β Λυκ) Σ D Λ D β) Αν για τη συνάρτηση f ισχύει f (t) :?: f (O) = 3για κάθε tε [Ο ,24] , τότε η συνάρτηση παρουσιάζει στο t = Ο ελάχιστο το 3. (σελ 33Β Λυκ) ΣD ΛD γ) Η συνάρτηση f με πεδίο ορισμού το σύνολο Α λέγεται άρτια αν για κάθε χε Α ισχύει f(-x) - f(x) = Ο. (σελ 35Β Λυκ) ΣD ΛD δ) Η γραφική παράσταση της συνάρτησης f(x) = lxl + Ι αποτελείται από τις ημιευθείες ψ = -χ+ Ι , με ΣΟ ΛD χ :s; Ο και ψ = χ+ Ι , με χ:?: Ο (σελ 40Β Λυκ) ε) Η γραφική παράσταση της συνάρτησης f (χ) = (χ+2)3 προκύπτει από οριζόντια μετατόπιση της ( h(χ)=αχ3 εφαρμ σελ Ι 9 Ι Α Λυκ) φ(χ) = χ3 κατά δύο μονάδες δεξιά. ΣD ΛD (Μονάδες

Γ.

10)

Να συμπληρώσετε τα κενά : Η γραφική παράσταση της συνάρτησης f με f (χ) = φ(χ+ . . . . ) , όπου c > Ο , προκύπτει από μία οριζόντια μετατόπιση της γραφικής παράστασης της συνάρτησης φ κατά c . . . . . (σελ 44 Β Λυκ) (Μονάδες 8)

Θ Ε ΜΑ 2°. ( Β Λυιc. : ασιc B l σελ 22, εφαρμ σελ 36,ασιc 8 σελ 39) Η συνάρτηση f είναι άρτια στο διάστημα [-4, 4] και τμήμα της γραφικής της παράστασης,

δίνεται στο i - - - - - - - - - - - - - - - - -; - - -Γ-_-_-_-::_-:_-:_-:_� σχήμα Ι . Α. Να συμπληρώσετε τη γραφική της παράσταση. (Μονάδες 6� 1 _ _ _σ_� � !.� _ ! _ ; Β. Να γράψετε τις τιμές των εικόνων f( l ), f(3) και να υπολογίσετε την τιμή της παράστασης : (Μονάδες 7) Α = 3 f(-I ) - 2 f(-3) + f(-4) Γ. Ποιος είναι ο τύπος της συνάρτησης (Μονάδες 7) .s f στο διάστημα [- Ι , Ι ]; . Δ . Να μελετήσετε τη συνάρτηση f ως προς την μονοτονία της 3.

..ι

Θ Ε Μ Α 3°

( Β' Λυκ σελ 1 0, 24, 26 . Α ' Λυκ ασκ A4;i) σελ1 98 )

.J

:

4

.ι 0 .ι

------------------------------------

Δίνονται οι εξισώσεις, I χ+ψ I = 3 (Ι) και χ2 + ψ2 = 5 (2) Α. Να αιτιολογήσετε ότι η εξίσωση (Ι) παριστάνει δύο ευθείες που τέμνουν τον άξονα χ'χ σε σημεία (Μονάδες 5) που έχουν αντίθετες τετμημένες και τον ψ'ψ με αντίθετες τεταγμένες - - Β. Στον κύκλο που παριστάνει εξίσωση (2) , να γράψετε το κέντρο και - - - - - - - �:--��-��;;---� ' ".. " - -.\- - - - - - - - (Μονάδ ες 8) την ακτίνα . Επίσης να δείξετε ότι χ ε [- .J5 , .J5 ]. Γ.

Να λυθεί το σύστημα

{

Ι χ +ψ1=3 χ 2 +ψ2 =5

(Μονάδες 7)

Στο σχήμα 2, να βρεθούν τα σημεία Α , Β, Α', Β ' ,όπου τέμνονται οι (Μονάδες 5) γραμμές που παριστάνουν οι εξισώσεις (Ι) , (2).

----�----+-�

��

Δ.

Θ Ε ΜΑ 4°

( Β Άυκ ασκ 12 σελ 23 και από Α Λυκ σελ 200-20 1 , εφαρ σελ 202 )

-1

/

:

8-

ο

·j �.._j__ '\!.', . /

Στο σχήμα 3, που δίνεται, φαίνεται η γραφική παράσταση του τριωνύμο: μορφή f(x) = αχ2 + βχ + γ , (α:;t:Ο). Γ - !.. - - - - - - - - - - - -,=--=--=--=--=--=--=--=--=--=--=--=-:; � Α. Με βάση το σχήμα 3, να αιτιολογήσετε ότι γ = Ο κ(ι,ι ) σ χ ή μ α 3 : (Μονάδες 5) και να υπολογίσετε τα α, β. Για την συνάρτηση f με f(x) = - χ2 + 2χ που έχει (3,0) πεδίο ορισμού το διάστημα [- Ι , 3]. (Μονάδες 7) Β. α)Να γράψετε τα διαστήματα μονοτονίας της f. Να βρείτε τις θέσεις Χ0 , όπου η συνάρτηση f παρουσιάζει β) (Μονάδες 8) ακρότατα ; Να γραφούν τα αντίστοιχα f(x0) . (Μονάδες 5) Γ. Αν χ1>2 και χ2>Ι να δείξετε ότι f 3 (x1)+ f(x2 )- I< Ο . �

Ψ

. . ι . -. .-.,- -- .-. ---'!'" - - -- - - - -

Σ

ΕΥΚΛΕΙΔΗΣ Β' 93 τ.Ι/56

- .ι


Μαθη ματικά για την Γ Λυκείου

Συναρτήσε ι ς κα ι Όρ ια

Γ ' ΛΥΚΕΙΟΥ

Κονόμης Άρτι Άσκηση 1 Να βρείτε το πεδίο ορισμού των συναρτήσεων :

+χ χ-3 f(x) = 52 + 2 χ - 16 χ - 3χ f(x) = �3 - l2 - xl + �1 - e'

α)

β)

Λύση α) Παρατηρούμε ότι

:

lim(�x χ -+ 2 2 - 1 + συν( π( χ - 1)) = J3 - 1 * Ο και lim χ-+2 [ eημπx + ln(x - 1)] e. Άρα : eημmι + ln(x -1) e e(.J3 + 1) lim �-+2 �χ2 -1 +συν(π(χ -1) = J3 - 1 = 2 =

γ)

β)

δ)

(

)

Έχουμε: lim x 3 - 2 7 = Ο . Όμως, Χ -+3

χ+3 χ2 - 9 (χ - 3)(χ + 3) να ορίζεται η συνάρτηση f πρέπει και χ 3 - 27 -- (χ - 3)(χ 2 + 3χ + 9) -- χ 2 + 3χ + 9 ' � αρκεί : χ 2 - 16 * Ο και χ 2 - 3χ * Ο . Έχουμε : 9 χ2 -16 = 0 <=>(χ -4Χχ +4) =0 <=> χ = 4 ή χ = -4 , lim(x χ -+3 2 + 3χ + ) = 27 * Ο . 2 9 . χ+3 = 1 χ 2 - 3χ = ο <=> χ(χ - 3) = ο <=> χ = ο ή χ = 3 Ά ρα Ι.χιm χ - = 1χιm -+3 χ 3 - 27 -+3 χ 2 + 3χ + 9 3 .Άρα A = R - {-4, 0 ,3, 4} . β) Για να ορίζεται η συνάρτηση f πρέπει και γ) Έχουμε: lim x 3 - 3x + 2 = 0 . Όμως αρκεί: 3 - 1 2 - x l � Ο και 1 - e' � Ο . Έχουμε: 2χ 2 - 5χ + 3 = (2χ - 3)(χ - 1) = 2χ - 3 με 3 - 1 2 - x l � Ο <::::> 1 2 - x l � 3 χ 3 - 3χ + 2 (χ - 1)(χ 2 + χ - 1) ' χ 2 + χ - 1 ' (1) -3 � 2 χ � 3 <=> -1 � χ � 5 <=> 2 + χ - 1) = 1 * Ο . lim(x χ-+1 (2) 1 - e' � Ο <=:>e' � 1 <=:> χ � Ο . 2χ - 3 _ 1 Οι κοινές λύσεις των (1) και ( 2) είναι το Άρα: Ι.ιm 2χ 2 - 5χ + 3 1ιm ---,-- . χ 1 χ -+ χ 3 - 2χ + 1 -+Ι χ 2 + χ - 1 διάστημα [ -1, Ο] .Άρα Α = [ -1, Ο] . 3 γ) Για να ορίζεται η συνάρτηση f πρέπει και δ) Έχουμε: lim x -3χ + 2 = Ο . Όμως 2 2 χ +-2 -2.J2x�χ2 +χ+ 2 +2.J2) αρκει:, e ' -4e' +3 > Ο . ,Εχουμε: e ' - 4ex + 3 > Ο �i +χ+2-2.J2 (�.-χ2+e' + 2 e' +2 i -3x+2 (x2 -3x+2X�x2 +x+2+2.J2) <=:> e2 x - 4χ + 3 > 0 (e' - 1)(e' - 3) > 0 χ2 + χ - 6 <=> e' < 1 ή e' > 2 <=> χ < Ο ή χ > ln3 . Άρα = (. . .) Λύση α) Για •

χ -+ 1

(

)

-

=

χ-+1

Α = (-οο,Ο) υ (ln 3,+oo) .

Για να ορίζεται η συνάρτηση f πρέπει και αρκεί : χ 2 - χ � Ο και χ - �χ 2 - χ � Ο . Εχουμε χ2 -χ � Ο <=:> χ(χ -1) � 0 <=:>χ � Ο ή χ � 1 (1) χ - �χ 2 - χ � Ο <=> �χ 2 - χ � χ (2) Αν χ < Ο τότε η (2) είναι αδύνατη. Αν χ � Ο τότε (1) <::::> χ 2 - χ � χ 2 <::::> χ � Ο (3) Οι κοινές λύσεις των (1) και (3) είναι το σύνολο : {Ο} υ [l,+οο) . Άρα Α = {Ο} u [1,+oo) . δ)

'

Άσκηση 2 Να υπολογιστούν τα όρια :

χ2 - 9 eημπχ + ln(x - 1) . lιm β) lim �-+2 �χ 2 - 1 + συν( π( χ - 1)) �-+ 3 Χ3 - 27 2 - χ+3 �x z + χ + 2 - 2J2 5 γ) lim 2χ δ) lim Χ3 - 3χ+ 2 Χ 2 - 3χ + 2 .

α)

Hl

Η

(

)

(x - 1)(x - 2)((�x 2 + x + 2 + 2.Ji) χ+3

------;====------==-

=

(χ - 1)(�χ 2 + χ + 2 + 2.Ji) � ιim [ (x - 1)(�χ 2 + χ + 2 + 2.Ji) ] = 4.Ji * ο . χ-+2 · �x 2 + x + 2 - 2.Ji Ά ρα .. ιχιm =,! -+2 χ 2 - 3χ + 2 χ+3 lim = s.Ji 8 ....2 (x - 1)(�x 2 + x + 2 + 2.Ji)

Άσκηση 3 Δίνεται κύλινδρος � ακτίνα βάσης ρ και με

εμβαδόν της ολικής επιφανείας του ίσον με 36πcm 2 α) Να εκφράσετε ως συνάρτηση του ρ το ύψος h του κυλίνδρου και να βρείτε το πεδίο ορισμού της. β) Να βρείτε για ποιες τιμές του ρ το ύψος h

ΕΥΚΛΕΙΔΗΣ Β '

93 τ.l/57


Μαθη ματικά για την Γ Λυκείου του κυλίνδρου είναι μεγαλύτερο από 7cm γ) ί) Να εκφράσετε ως συνάρτηση του ρ τον

όγκο V του κυλίνδρου. ίί) Να βρείτε τον όγκο V όταν το ύψος του είναι ίσον με 3cm . Λύση α) Η ολική επιφάνεια του κυλίνδρου δίνεται από

τον τύπο: Εολ = 2πρ2 + 2πρh , όπου ρ είναι η ακτίνα της βάσης και h είναι το ύψος του . Έχουμε: 36π = 2πρ2 + 2πρh � 1 8 = ρ2 + ρh � h = 1 8 - ρ 2 ρ Άρα: h(ρ) = 1 8 - ρ 2 . Επειδή h(ρ) είναι ύψος ρ κυλίνδρου είναι h(ρ) > Ο Έχουμε 18 2 h(ρ) > 0 <::> ----=.e._ > 0 <::> 18-ρ2 >0 <=>-3fi < ρ < 3fi . ρ Όμως ρ > Ο ως ακτίνα κυλίνδρου . Άρα Α, =(0,3J2.) 1 8 - ρ2 β) 'Εχουμε : h(ρ) > 7 <::> > 7 <::> ρ <=>ρ2 + 7ρ-18 < 0 <=>(ρ+9Χρ -2) < 0 <=> -9 < ρ < 2 . Από το (α) ισχύει Ο < ρ < 3J2 . Άρα: Ο < ρ < 2 . γ) ί) Ο όγκος του κυλίνδρου δίνεται από τον τύπο 1 8 - ρ 2 = πρ2 (1 8 - ρ2 ) Άρα: . V = πρ2h = πρ2 ρ V(ρ) = πρ2 (1 8 - ρ2 ) και Α ν = (0,3J2) . ίί) Έχουμε: V(ρ) = πρ2 (1 8 - p 2 ) και h(ρ) = 1 8 - ρ 2 . Είναι: 3 = 1 8 - ρ 2 � 1 8 - ρ2 = 3ρ ρ ρ � ρ2 + 3ρ - 1 8 = ο � ρ = -6 ή ρ = 3 � ρ = 3 (αφού ρ > Ο ) . Τότε για ρ = 3 ο όγκος είναι: V (3) = π · 3(1 8 - 32 ) = 27π cm2 . Άσκηση 4

, Δινεται α, β Ε JR

η •

, συναρτηση

f(x)

χ 2 + αχ + β = I χ2 + χI

Α( 1, 6)

και

τότε: α) Να βρείτε τις τιμές των α , β

με

Β( 2, 13° )

Ε 1R .

β) Να βρείτε για ποιες τιμές του χ η γραφική παράσταση της f είναι κάτω από τον άξονα χ ' χ

f(x)

γ) Να βρείτε το όριο: lim 2 . χ-+-3 χ + 2χ - 3

{f(x)-f(l) , χ-1

χ Ε (0, 1) υ (1, +οο)

λ,

χ=1

Να βρεθεί η τιμή της παραμέτρου λ , ώστε η g( χ) να είναι συνεχής στο χ 0 = 1 .

Λύση α)

Επειδή τα σημεία Α ( 1 , 6 ) και

' 1�) είναι στη 2,

γραφική παράσταση της f ισχύει ότι : f(l) = 6 και f(2) = 10 . 'Εχουμε: f(1) = 6 � ... � α + β = 1 1 (1) 3 και f(2) = !2. � ... � 2α + β = 1 6 (2) . Από τις , 3 (1) , (2) βρίσκουμε α = 5 και β = 6 . χ2 +5χ+6 Για να β) Για α = 5 και β = 6 , f(x) . l x2 + � ορίζεται η f πρέπει και αρκεί: χ2 + χ ::ι:. Ο . Όμως χ2 + χ = Ο <::> χ(χ+ 1) = 0 <::> χ = Ο ή χ=-1 . Άρα: A = JR - {-1,0} . Οι τιμές του χ για τις οποίες η γραφική παράσταση της f είναι κάτω από τον άξονα χ ' χ είναι οι λύσεις της ανίσωσης f(x) < Ο . 'Εχουμε: f(x) < Ο <::> χ2 +5χ+6 < Ο <::> χ2 +5χ +6 < 0 I χ2 +χI <::> -3 < χ < -2 f(x) _ χ2 + 5χ + 6 , . Υ) Ε ιναι .. χ 2 + 2χ - 3 - ι χ2 + χ I (χ2 + 2χ - 3) Όμως χ2 + χ > Ο <::> χ ( χ +1 ) > 0 <=> χ < -1 ή χ > Ο . Άρα για χ κοντά στο -3 ισχύει: χ 2 + χ > Ο . (χ + 2)(χ + 3) χ2 + 5χ + 6 - -'Άρα ·· -�(χ2 + χ)(χ2 + 2χ - 3) ι χ2 + x l <x2 + 2χ - 3) χ+2 = , με ιim [(x2 +x)(x -1)] =-24 -::F- 0. (χ 2 + χ)(χ - 1) f(x) · χ+2 1 . Άρα : x--+ ιιm-J x2 + 2x - 3 = x--ι+ιm-J (x2 x)(x 1) = 24 + δ) Για να είναι η g συνεχής στο χ0 = 1 πρέπει και αρκεί : ιimg(x) = g(l) = λ (1)

---:

-'---

χ-+-3

Αν η γραφική παράσταση της f

διέρχεται από τα σημεία

g(x) =

Χ -> 1

Για χ -:i' 1 είναι:

g (x ) = f (x) - f (l) =

χ-1

-S x 2 - χ + 6

(x 2 + x)(x - l)

= -5χ - 6 , με lim x(x + 1) = 2 ::ι:. O . Άρα x(x + l) ι. ( ) - ι f(x) - f(l) 1 1 . Άρα·. _ -S x - 6 -_ - ιm g χ - ιm χ-.Ι

·

δ) Έστω g συνάρτηση που δίνεται από τον τύπο

χ-+ Ι

χ-+ Ι

11 (1) <::> λ = - - . 2

ΕΥΚΛΕΙΔΗΣ Β ' 93

τ.Ι/58

χ-1

χ(χ + 1)

2


Μαθη ματικά για την Γ' Λυκείου

Γ λΥΚΕΙΟΥ '

Μιγαδικοί Αριθμοί .

Η

Άλγεf}ρα και

η Γ εwμετρία

τους

των Βασίλη Καρκάνη και Φραγκίσκου Μπερσίμη Οι Μιγαδικοί Αριθμοί αποτελούν το ]0 κεφάλαιο των Μαθηματικών Κατεύθυνσης της ΓΆυκείου. Οι μαθητές γρήγορα αντιλαμβάνονται τη σημασία της Άλγεβρας και της Γεωμετρίας των Α ' και Β ' τάξεων του Λυκείου για την απόκτηση νέων γνώσεων επί των μαθηματικών εννοιών. Επιπλέον, με αρκετές από τις παρακάτω ασκήσεις που παραθέτουμε γίνεται φανερή και η σημασία της Αναλυτικής Γεωμετρίας καθώς επίσης και η αξία της σχηματικής απεικόνισης.

ΑΣΚΗΣΕΙΣ

Άσκηση ι η

Για τις διάφορες τιμές του v Ε Ν να υπολογιστούν οι δυνατές τιμές της παράστασης:

( ι + ; ν ) . ( 2 + ; 2 ν+ l ) π= { 2 + ;3 ν+2 )

εξίσωση ως προς χ είναι αδύνατη διότι για v Ε Ν η διακρίνουσα του τριωνύμου είναι Η

....:.,_----,--'--..ο..,_----,--�

Λύση

Διακρίνουμε τις ακόλουθες περιπτώσεις: Αν ν=4κ με κ Ε Ν , τότε ισχύει:

( Ι + ί4κ) · ( 2 + ί2(4κ)+ι ) (Ι + Ι ) · ( 2 + i) 2 ( 2 + i) π- ( 3(4κ)+2) - 2 Ι 2+ί Αν ν=4κ+ Ι με κ Ε Ν , τότε ισχύει: ( Ι + ί4κ+Ι ) · ( 2 + ί2(4κ+Ι)+Ι ) (I +i) · ( 2-i) _ 7 -ί = Π 2 +ί ( 2+ ί3(4κ+1)+2) 5

Αν ν=4κ+2 με

κΕ

Ν,

τότε ισχύει:

( Ι + ί4 κ+ Ζ ) · ( 2 + ί 2(4 κ+ Ζ)+ Ι ) (Ι - Ι ) · ( 2 + i) -ο π2+Ι ( 2 + ί 3( 4 κ+2)+2 ) Αν ν=4κ+3 με κ Ε Ν , τότε ισχύει: ( Ι + ί4κ+3 ) · ( 2 + ί2(4κ+3)+ 1 ) (I - i} · ( 2 - i} = 2 = (Ι - ι) Π= ( 2 + ί3(4κ+3)+2 ) -ί

.

Άσκηση 2 η Δίνονται οι αριθμοί

α = 0 ή β = Ο <::> vx2 + ( v + Ι ) χ + v + 2 = Ο ή ln [ vx2 + ( v + I) x + v + 3] = ln l <::> vχ2 + ( v + 1) χ + v + 2 = ο .

α = vx2 +(v+ι) x + v + 2 και β = ln [ v χ 2 + ( v + ι ) χ + v + 3 J με v Ε Ν * , χ Ε 1R και οι μιγαδικοί αριθμοί Ζν = α + βί . Να

*

Δ = ( v + l)2 - 4v ( v + 2 ) = ... = -3v 2 - 6v + I < O . * β) Έστω Vι ' V Ε Ν ' Vι * V και χ Ε IR ώστε να z z

ισχύει

= zv2 <::> Vι χ2 + ( Vι + Ι ) χ + Vι + 2 + +iln [v1 x 2 + ( v1 + l)x + v1 + 3] = = v2x2 + ( v2 + l) x+v2 + 2+iln[v2x2 + ( v2 + l) x+v2 +3J <::> �J.'ι_i +( J.'ι + l)X+ J.'ι +2 = v� +( V2 +l) x+V2 +2 ι <::> lln[Yιi +( Υι + l) χ+ Υι +3] = 1n[v� +( Vz +l) x+ Vz +3]J zv,

v1 -v2 ..0

<::>( 1ι] -v2 ) i + ( 1ι] -v2 } x+ Jι] -v2 =0 <::> που είναι αδύνατη γιατί Δ < Ο . Άσκηση 3η

Έστω

α E 1R

+ W = z αι E C , z - αι .

.

με

� +x+l =O

z Ε C - { αί }

και

• •

α) Αν ο αριθμός w είναι φανταστικός να δειχτεί ότι οι εικόνες του z ανήκουν σε κύκλο με κέντρο το 0(0,0) και ακτίνα ρ=α. β) Αν ισχύει jwj = ι να βρεθεί ο γεωμετρικός

τόπος των εικόνων του z. Λύση α) Έστω z τότε για τον w έχουμε:

= χ + yi , (x + ( y + a ) i) · (x - ( y - a)i) x + ( y + a)i - _,_ v Ε Ν* και χ Ε 1R ο Ζν δεν είναι __,_ w__,__,_ x + ( y - a)i (x + ( y - a ) i) · (x - ( y - a ) i) πραγματικός ούτε φανταστικός. β) Για κάθε V1 ,v Ε Ν* με V1 ;e v και χ Ε 1R _- χ2 - x ( y - a ) i + x ( y + a ) i+ y 2 - a2 2 2 ισχύει Ζν ;ι!: Ζν xz + ( y - a) z Λύ ση 2 2 2 * = χ + y - a 2 + 2χαί 2 , και ο αριθμός w α) Έστω v Ε Ν και χ Ε IR ώστε ο zv να είναι xz + ( y - a) xz + ( y - a) πραγματικός ή φανταστικός. Αυτό ισοδυναμεί με: δειχτεί ότι: α) Για κάθε

_ _ _

I

1

ΕΥΚΛΕΙΔΗΣ Β' 93 τ.Ι/59

_ _ _


Μαθη ματικά για την Γ Λυκείου

είναι φανταστικός � Re(w)=O, δηλαδή:

χ2 + Υ 2 _ a 2 = Ο � χ2 + y 2 = a2 , οπότε οι εικόνες χ2 + ( y - a )2 M(z) ανήκουν σε κύκλο με κέντρο το 0(0,0) και

ακτίνα ρ=α με εξαίρεση το σημείο Α(Ο,α) .

β) Ισχύει l wl = Ι , οπότε x + ( y + a)ί � l x + ( y + a)ίl ι -�ι = Ι l x + ( y - a)ί l = Ι x +--'( y - a )ί � Ιχ + ( y + α)ί 1 2 = lx + ( y - a)ίl 2 � � χ2 + ( y + a ) 2 = χ2 + ( y - a) 2 � � 2ya = -2ya � 4ya = O�y = O ,

l w -8-6il= 2

και

γ) Να βρεθεί η ελάχιστη τιμή των

οπότε ζητούμενος γ.τ. είναι τα σημεία του άξονα χ ' χ.

ο

u

για

και

u l z - uι.

l l

και

δ) Να βρεθεί ο πραγματικός αριθμός α και ο φανταστικός αριθμός βί, οι εικόνες των οποίων ισαπέχουν από τις εικόνες των μιγαδικών και

z0 =-3 - 4ί w0 =8 + 6ί .

Λύση α) Ισχύει:

z * Ο.

l

και

έστω Μ, Ν, Ρ οι εικόνες των μιγαδικών z,w,u αντίστοιχα. α) Να βρεθεί ο γ.τ. των εικόνων των μιγαδικών αριθμών z, w και u. Να βρεθεί η ελάχιστη και η μέγιστη τιμή των και '

z w l z -wι.

a .. o

w

z + 3 + 4il = 3, l u + 3+4il=l u -8-6il

β) ll l l

-

Άσκηση 4 η Δίνεται ο μιγαδικός αριθμός

Άσκηση 5η Δίνονται οι μιγαδικοί αριθμοί z, τους οποίους ισχύουν:

l z + 3 + 4ίl = 3 � lz - ( -3 - 4ί)l = 3 ,

δείξετε ότι οι εικόνες του w για κάθε λ ε R ανήκουν στον γ.τ. που βρέθηκε στο 1° ερώτημα. Λύση

οπότε ο γ . τ . των εικόνων M(z) είναι τα σημεία του κύκλου με κέντρο Κ1 (-3,-4 ) , ακτίνα p1 = 3 και εξίσωση C1 : ( χ + 3 ) 2 + ( y + 4 ) 2 = 9 . Επίσης ισχύει: l w - 8 - 6ί l = 2 � l w -, (8 + 6ί)l = 2 , οπότε ο ζητούμενος γ . τ . των εικόνων N(w) είναι τα σημεία του κύκλου με κέντρο Κ2 (8 ,6) και ακτίνα ρ2 = 2 . Η εξίσωση του κύκλου αυτού είναι:

α) Για z =F- 0 , έχουμε: z + � + 2 + z + � - 2 = 4 �

Ακόμη:

α) Αν ισχύει

l z +�+21+1 z +� - 21 =4 , w = ι + λί λ

να βρεθεί

ο γεωμετρικός τόπος των εικόνων του z.

,

β) Δινεται ο μιγαδικος

l

ll

l

-.

1 - λι

l

με

Να ε n. Π]) •

11

2 2 � z + :+ 2z + z + :- 2z = 4 �

2 + 2 � (z I) + (z - I) = 4 � z z l (z + I)2 1 l (z - I )2 1 4 � l z + I I 2 + l z - I I 2 = 4 l zl � + � zl l l zl � (z + l)( z + I) + (z - I)( z - Ι ) = 4 l z l � � z · z + z + z + I + z · z - z - z + Ι = 4 lzl � � 2 l zl 2 - 4 l zi + 2 = 0 � I zl 2 - 2 l z i + I = O � � ( l zi - I )2 = Ο � l zl = Ι .

Επομένως ο ζητούμενος γ . τ. των εικόνων M(z) είναι τα σημεία του μοναδιαίου κύκλου .

I

Ι+ �

C2 : ( χ - 8) 2 + ( y - 6) 2 = 4 .

i u + 3 + 4ίl = l u - 8 - 6ίl � Ι u - ( -3 - 4ί)l = l u - (8 + 6ί)l

οπότε ο ζητούμενος γ . τ. των εικόνων P(u) είναι τα σημεία της μεσοκαθέτου του ευθυγράμμου τμήματος Κ1 Κ2 • Για την εύρεση της εξίσωσης της μεσοκαθέτου, αν u = χ + yί , x,y ε JR , τότε η παραπάνω εξίσωση γράφεται:

1 ( χ + yί ) + ( 3 + 4ί ) I = 1 ( χ + yί ) - ( 8 + 6ί )I � � l(x + 3) + ( y + 4 )ίl = l(x - 8) + ( y - 6 )ίl � ... � � ( χ + 3) 2 + ( y + 4) 2 = ( χ - 8)2 + ( y - 6 ) 2 � ... � � 22x + 2 0y - 75 = 0 . Άρα η ευθεία ε: 22x + 2 0y - 75 = 0 αποτελεί τον γ.τ. των εικόνων

του υ.

Είναι l zl = ( ΟΜ) , οπότε � �πin =(ΟΑ) =( α\:; ) - Α και l z lmax = ( ΟΒ ) = ( ΟΚ1 ) + Ρι . β)

�(

I + λ� = ' λ� = Ι , Είναι: ( Κ1 ) = -3)2 + ( -4)2 = 5 ι � Ο β) Για κάθε λ ε IR είναι l wl = Ι λ ι Ι - λι οπότε οι εικόνες του μιγαδικού w ανήκουν στο άρα l z lmin = (ΟΑ) = 5 - 3 = 2 και μοναδιαίο κύκλο. l z lmax = { ΟΒ) = 5 + 3 = 8 . ΕΥΚΛΕΙΔΗΣ Β' 93 τ.l/60


Μαθηματικά για την Γ Λυκείου

Ό μοια l wl = ( ΟΝ ) , οπότε

l wlmin = ( ΟΓ ) = ( ΟΚ2 ) - p2 και l wlmax = ( ΟΔ ) = ( ΟΚ2 ) + Ρ2 · Είναι: ( ΟΚ2 ) = �8 2 + 62 = 10 άρα l wlmin = ( ΟΓ ) = l Ο - 2 = 8 και l wl max = 10 + 2 = 1 2 . Επίσης l z - wl = ( ΜΝ ) , οπότε l z - wlmin = ( ΕΖ ) = ( Κ1 Κ2 ) - p1 - p2 και l z - wl max = ( ΗΘ ) = ( Κ1 Κ2 ) + Ρι + Ρ2 •

χ = 7Ji2 άρα π ( 7Ji2 ,o ) . Επίσης από την ε, για χ = Ο έχουμε y = 1 % άρα Λ ( ο, 1 % ) · Άσκηση 6η

Δίνεται η εξίσωση

z2 - 4z + 8 = Ο ( 1)

α) Να βρd)ούν οι ρίζες β) Να βρεθεί η Α = z;οι4 + z;οι4

Ζ1

και

τιμή

z2

της εξίσωσης της

(1) .

παράστασης

γ) Αν Ο το κέντρο των αξόνων και Α,Β οι εικόνες των z1 και z2 αντιστοίχως, να δείξετε

ότι το τρίγωνο ΟΑΒ είναι ορθογώνιο και ισοσκελές. δ) Να βρεθεί ο γ.τ. των εικόνων του μιγαδικού αριθμού z αν z1 = 2 + 2ί , z2 = 2 - 2ί και ισχύει

l z - zι l 2 l z - z2 l · =

10

P(u)

Είναι ( Κ1 Κ2 ) = Jm άρα l z - wlmin = Jm - 5 και l z - �1max = mt + 5 . γ) Ισχύει

1 22 · 0 + 2 0 · 0 - 75 1 διότι l u l mΙn. = d( Ο ,ε) = .J22 2 + 2 0 2 = ....Ε_ .J884 l u l = ( ΟΡ ) . Επίσης l z - u l = ( ΜΡ ) και αν η ε τέμνει το τμήμα Κ1 Κ2 στο I, τότε: l z - u l min = ( ΕΙ ) = ( Κ/ ) - Ρι = d( Κ1 , ε) - 3 = 122 · (-3) + 2 0 · (-4 ) - 75 1 = - 3 = ... = Jm2 - 6 . .J22 2 + 202

Οι εικόνες των z0 και w0 είναι τα κέντρα Κ1 και Κ2 των κύκλων C1 και C2 αντιστοίχως. Αν Π ( α, Ο ) και Λ ( Ο, β ) οι εικόνες των α και βi αντιστοίχως, θέλουμε ( ΠΚ 1 ) = ( ΠΚ 2 ) και ( ΛΚ 1 ) = ( ΛΚ 2 ) , οπότε τα σημεία Π και Λ είναι τα σημεία τομής της ε με τους άξονες χ ' χ και y 'y αντιστοίχως. Από την ε, για y = Ο έχουμε δ)

·

Λύση α)

Η

διακρίνουσα

της

εξίσωσης

είναι

Δ = β2 - 4αy = ( -4)2 - 4 · 1 · 8 = 1 6 - 32 = -16 < Ο , Ρ ± ίr-Δ = -4 ± 4ί = 2 ± 2ι. οπότε άρα z1 . 2 = ' 2 2 z1 = 2 + 2ί και z2 = 2 - 2 ί είναι οι ρίζες της εξίσωσης ( 1) . 014 01 Ι ' β) σχυει: z120 1 4 = ( 2 + 2 ι ) 2 = 2 20 1 4 · (1 + ι ) 2 4 = [ 1 007 1 = 2 20 1 4 . ( 1 + ί ) 2 ] = 2 20 1 4 · (1 2 + 2 ί + ί 2 ) 007 = = 2 20 1 4 . ( l + 2 ί - 1 ) 1 007 = 2 20 1 4 . ( 2 ί ) 1 007 = = 2 302 1 . ( ί4 ) 25 1 . ί3 = -2 302 1 . ί ' Ε πισης z220 1 4 = ( 2 - 2 ι' ) 20 1 4 = 2 20 1 4 · ( 1 - ι' ) 20 1 4 = 7 = 2 20 1 4 { (1 - ί ) 2 τ 00 = 220 1 4 . ( 1 2 - 2 ί + ί 2 ) 1 007 = = 2 20 1 4 . ( I - 2 ί - Ι ) 1 001 = 22ο ι 4 . ( _2 ί ) 1 001 = 1 = 2 302 1 · ( ( -ί )4 ) 25 · (-ί )3 = -2 302 1 · ( -ί ) = 2 302 1 ί Οπότε Α = z:0 1 4 + zio ι 4 = -2 3ο2 ι . ί + 2 302 1 . ί = Ο γ) Έστω z1 = 2 + 2ί και z = 2 - 2 ί με εικόνες τα 2 σημεία Α(2,2 ) και Β(2, -2 ), οπότε: δΑ. = ( 2 , 2 ) και δΒ = ( 2 ,-2 ) . Τότε ισχύει: l zι l = l δAJ = �22 + 22 = .J8 = 2fi και l δBI = J22 + ( -2 )2 = .J8 = 2J2 , οπότε l δΛI = l δBI . Επίσης δΑ. · δΒ = 4 - 4 = Ο άρα δΑ _i δΒ , οπότε

ΕΥΚΛΕΙΔΗΣ Β' 93 τ.l/61


Μαθη ματικά για την Γ Λυκείου

το τρίγωνο ΟΑΒ είναι ορθογώνιο και ισοσκελές.

Λύ ση α ) Έχουμε:

10 ( 5 z + 1 γ0 1 5 = + 20 1 5 � 10( 5 z + 1 ) 20 1 5 = (z 5) 8 + 6ί 8 + 6ί 01 I= ( z + 5 )2ΟΙ 5 1 � 10 I 58z++6ί112 5 = I z + 5l 2o 1 5 � I I 1 1 0 0 5 5 2 2 � l 5z + 11 = l z + 51 � � l 5z + 11 = l z + 51 � l 5z + 112 = l z + 512 � � (5z + 1)( 5 z + 1) = (z + 5 )( z + 5 ) � � 2 5 zz + 5 z + 5 z + 1 = zz + 5 z + 5 z + 2 5 � � 24zz = 24 � zz = 1 � l z l 2 = 1 � l z l = 1 δ) Έστω z = x + yί , x,y ε lR και M ( x,y) η εικόνα του z , τότε: ' ' u = 3 ι'w = ρ7ι· jί -;7 - z = - -;7 + z αρα β) Ι σχυει: l z - z 1 1 = 2 l z - z2 1 � l z - z 1 1 2 = 4 l z - z2 1 2 � ! (χ - 2 ) + ( y - 2 )ί 1 2 = 4 i ( x - 2 ) + ( y + 2 )ί 1 2 � 2 - z = 2 = J2l (a) 7 + (ΑΒ)= = z z u l l z z l zl � ( χ - 2 ) 2 + ( y - 2 ) 2 = 4 [( χ - 2 ) 2 + ( y + 2γ ] � γ) Έστω: w = χ + yί με x,y ε lR και Μ(χ,y) η � 3 ( χ - 2 ) 2 - ( y - 2 ) 2 + 4 ( y + 2 ) 2 = 0 � ... � εικόνα του του w . Αν z = α + βί , α, β ε JR με 2 0 � χ2 + y 2 - 4x + -y + 8 = 0 ( 2 ) (α, β) ;t (0,0) , λόγω του (α) είναι: α2 + p2 = 1 (1) . 3 20 Οπότε . Α = -4 , Β = 3 και Γ = 8 . Οπότε η Ισχύει: 7 . - ( α + βι' ) � εξίσώση (2) εκφράζει κύκλο με κέντρο το w = -ί -7 - z � χ + yι. = -ί -3 3 α + βι z 10 και ακτινα ' ρ = 38 διοτι κ 2 , -3 ' ισχυει: ' � χ + yί = -ί 7 (α2 - βί) - (α + βί) � 2 + 3 α β 2 6 5 Α 2 + Β 2 - 4Γ = > Ο . (I ) 9 8β � � χ + yι. = -ί ( 6α - 8β')ι � χ + yι. = 2αι. + 2 4 3 3 Ο κύκλος C: ( x - 2 ) 2 + y + 1 = είναι ο 3 8 � χ = και y = 2α � α = � και β = ; ( 2 ) ζητούμενος γεωμετρικός τόπος. e

( ) l l 11

( )

( )

Άσκηση 7η Για τους

10 ( S z + l ) 8 + 6; z ;t ο .

( �) �

μιγαδικούς

ms

( z + S ) zoιs

α) Να δειχθεί ότι

l zl = 1 .

z

και και

J

}{

)

}

Από σχέση (1), λόγω της (2 ) θα ισχύει: 2 χ +/ = 1 οπότε ο ζητούμενος γ.τ. των -2 ί 7 2 ' w = � - z , (%) 2 J εικόνων M(w) είναι σημεία έλλειψης με εξαίρεση w

ισχύουν:

( )

β) Αν Α η εικόνα του μιγαδικού z και Β η εικόνα του μιγαδικού u = 3iw , να βρεθεί το μήκος

{ {

(

(

(ΛΒ) .

γ) Να δειχθεί ότι ο γ.τ. των εικόνων των μιγαδικών w είναι σημεία έλλειψης. δ) Ν α βρεθεί η ελάχιστη και η μέγιστη τιμή του

l wl .

ΕΥΚΛΕΙΔΗΣ Β'

τα σημεία τομής της με τους άξονες, δηλαδή τα σημεία (0,2), (0, -2 ), (8/3,0) και (-8/3,0).

Από το ερώτημα (γ), για την έλλειψη που ' ' α = -8 και β = 2 , οποτε β ρε' θηκε ισχυει ' οτι δ)

3 l vl min = β = 2 και l vl max = α = % 93 τ.l/62


Επιμέλεια: Γιάννης Στρατής - Βαγγέλης Ευσταθίου

ΕΠΙΛΥΣΗ ΕΞΙΣΩΣΕΩΝ ΤΗΣ ΜΟΡΦΗΣ f(f(x))=x. Διονύσης Γιάνναρος,

Στο παρόν άρθρο παρουσιάζουμε μια μέθοδο επίλυσης εξισώσεων-συστημάτων που έχουν ή ανάγονται στη μορφή f(f(x))=x. Θεωρούμε μια εξίσωση της μορφής f(f(x))=x ( Ι ) καθώς και την εξίσωση της μορφής f(x)=x. (2) 1.

Θα αποδείξουμε τις παρακάτω προτάσεις:

Κάθε ρίζα της εξίσωσης (2) είναι και ρίζα της εξίσωσης (1 ).

Απόδ ε ιξη : Πράγματι, έστω ότι ο αριθμός χ0 είναι ρίζα της εξίσωσης (2). Τότε θα ισχύει f(x0)=x0• Χρησιμοποιώντας αυτή την ισότητα δύο φορές έχουμε f(f(xo))=f(xo)=xo . Δηλαδή f(f(xo))=xo που σημαίνει ότι ο αριθμός χο είναι ρίζα της εξίσωσης f(f(x))=x. 2. Έστω συνάρτηση f γνησίως αύξουσα σε ένα διάστημα Δ και έστω f(χ0) ε Δ για κάθε χ0 ε Δ. Τότε οι εξισώσεις ( Ι ) και (2) είναι ισοδύναμες στο Δ. Απόδ ε ιξη : Αποδείξαμε στην πρόταση ( Ι ) ότι κάθε ρίζα της εξίσωσης f(x)=x είναι και ρίζα της εξίσωσης f(f(x))=x. Θα αποδείξουμε τώρα ότι κάθε ρίζα της εξίσωσης f(f(x))=x είναι και ρίζα της εξίσωσης f(x)=x. Έστω ότι ο αριθμός χ0 ε Δ είναι ρίζα της εξίσωσης f(f(χ) )=χ, τότε θα ισχύει f(f(x0))=x0. Υποθέτουμε ότι f(x0):;tx0• Τότε αν f(xo)>xo επειδή f(xo), Χ0 Ε Δ και fl" Δ θα είναι: f(f(xo))> f(xo), δηλαδή Χο > f(xo), αφού f(f(xo))=xo πράγμα άτοπο. Όμοια αν f(x0)<x0 με ανάλογους συλλογισμούς καταλήγουμε σε άτοπο. Επομένως ο ισχυ ρισμός f(x0) =Ι= Χ0 δεν είναι αληθής. Τότε όμως θα ισχύει η ισότητα f(xo)=x0, δηλαδή ο αριθμός χ0 θα είναι ρίζα της εξίσωσης f(x)= χ. Παρ ατή ρη ση : Αν η συνάρτηση f είναι γνησίως αύξουσα στο διάστημα Δ και f(χ0) ε Δ για κάθε χ0 ε Δ τότε οι εξισώσεις: f(x)=x, f(f(x))=x, f(f(f(x)))=x κ.τ.λ. είναι ισοδύναμες στο Δ. 1.

με την εξίσωση f(x)=x δηλαδή την χ3+6=χ που έχει μοναδική ρίζα τον αριθμό -2.

Να λυθεί η εξίσωση: (χ3+6)3+6=χ.

Λύ ση : Η συνάρτηση f(x) = χ3+6 είναι γνησίως

αύξουσα στο R και f(x0) ε R, για κάθε χ0 ε R. Παρατηρούμε ότι οι προϋποθέσεις της πρότασης (2) ικανοποιούνται, επομένως η εξίσωση (χ3+6)3+6=χ που είναι της μορφής f(f(x))=x, θα είναι ισοδύναμη

Να λυθεί η εξίσωση:

2.

�Vx + 24 + 24 = χ , χ�-24.

Η

συνάρτηση f(x)= Vχ + 24 είναι γνησίως αύξουσα στο [-24,+οο) και f(x0) ε [Ο,+οο) ς [-24,+οο) για κάθε χ0 ε [-24,+οο). Ικανοποιούνται οι προϋποθέσεις της πρότασης (2) και επομένως η Λύ ση :

2 4 + 24 = + 24 =

εξίσωση �Vχ + εξίσωση : Vχ αριθμό 3 .

Η

; 2 = V3x - 2 , Λύ ση :

χ3

χ είναι ισοδύναμη με την χ , που έχει μοναδική ρίζα τον

Να λυθεί η εξίσωση : x3+ 2=3· V3x - 2 , χ� � .

3.

( ;2 ) 2 = 2 3

χ3

+

( ; 2 ) = 2, ( ) +2 =

εξίσωση ή

γράφεται 3 χ3

χ3 + 2 3 3

3χ , ή

3

διαδοχικά:

3χ -

ή

χ είναι δηλαδή

3

της μορφής f(f(x)) = χ, όπου f(x)= ... Παρατηρούμε π

62

f -i

2

, +οο) . [ - ,+οο) και f(χ) ε [ - ,+oo) c - [3 81 3 Ικανοποιούνται επομένως οι προϋποθέσεις της με συνέπεια η εξίσωση : πρότασης (2) 3 χ3 πως

( ; 2 ) +2 --'---- --'-- = χ +2 -=χ 3 -

3

-

εξίσωση:

χ

να είναι ισοδύναμη με την

3

που έχει μοναδική (αποδεκτή)

ρίζα τον αριθμό 1. 4.

Ε φ αρ μογές

1° Λύκειο Πύργου

Να λυθεί η εξίσωση:

χ ε ( -οο , -1 ] υ [ 1 , +οο ) .

/�lxl - t + � + 2 = x , 4 4

ν

Λύση : Παρατηρούμε ότι αν η εξίσωση αυτή

4

έχει λύση τότε: χ� 2 . Δουλεύουμε επομένως στο

ΕΥΚΛΕΙΔΗΣ Β' 93 τ.l/63


-----

Το Βήμα του Ευ κλείδη

διάστημα [ 2. ,+οο). Σ' αυτό το διάστημα η εξίσωση

4

γράφεται:

f..ι_χ___1_+ 2.___1 + 2. = χ . Θεωρούμε την 4 4 ΊJ '

__

μο ρ φή :

{Υ=

f(x ) η τρίτη . Επειδή η t l' R, z = f(f(x )) χ = f(f(f(x)))

εξίσωση του συστήματος είναι ισοδύναμη με την χ3 + 2 ' εξίσωση : f(x)=x δη λαδη' με την εξισωση -3- =χ <=>

7 7 συνάρτηση f(x)= -νι---:; χ - 1 + - στο [ - ,+οο). 4 4 7 J3 7 7 . Η f l' [ - ,+οο) και f(x0) ε [ - + - ,+οο) c [ - ,+οο) 2 4 4 4 7 ' ' για κα' θε χ0 ε [ - ,+οο) . Ά ρα συμφωνα με την προταση 4

χ3 - 3χ +2 =0. Η τελευταία έχει ρίζες τους αριθμούς Ι και -2. Άρα το σύστημα έχει ως λύσεις τις τριάδες: (x,y,z)=( l , Ι , Ι ), (x,y,z)=(-2,-2,-2).

(2)

Ά σ κη ση 2.

η

ισοδύναμη

εξίσωση στο

/-Γχ="ϊ + 2. - 1 + 2. = χ

ΊJ

7

[ 4" , +οο)

4

4

με

την

είναι

εξίσωση :

.Jχ - 1 + '!... =χ η οποία έχει μοναδική (αποδεκτή) 4 13 ρίζα τον αριθμό - . Άρα και η αρχική εξίσωση έχει 43

1 ρι'ζα τον αριθ μο' 4' . 5.

Να λυθεί η εξίσωση : ημ(ημχ)= χ.

Λύ ση : Γνωρίζουμε ότι:

I ημα I � Ι για κάθε

α ε R, επομένως, αν η εξίσωση έχει ρίζες, τότε αυτές θα βρίσκεται στο διάστημα Δ=[- Ι , Ι ] . Στο διάστημα Δ η συνάρτηση f(χ)=ημχ είναι γνησίως αύξουσα και f(x0) ε Δ για κάθε χ0 εΔ. Σύμφωνα με την πρόταση (2) η εξίσωση ημ(ημχ)=χ είναι στο Δ ισοδύναμη με την εξίσωση ημχ=χ η οποία έχει μοναδική ρίζα τον αριθμό Ο. Ας δούμε τώρα πως (κυρίως) με τη βοήθεια της πρότασης (2) αντιμετωπίζουμε συστήματα που

�::·: :::�:.:::μα: {�=��=�στο i' =3x-2 f( )

' την συναρτηση ' ,t = Λύ ση : Θεωρουμε

R.

t 3 +-2 3

-

·

Παρατηρούμε ότι: f l' R και f (R)=R. Το σύστημα χ3 + 2 3- = y ισοδύναμα γράφεται:

{

y3 + 2 -3 =z z3 + 2 -3 =Χ

ή χρησιμοποιώντας την συνάρτηση f έχουμε: Υ = f(x) z = f(y) και τελικά με αντικαταστάσεις, στη χ = f(z)

+ 2χ2 + 2x = y y3 + 2y2 + 2y = z . z3 + 2z2 + 2z = x

r Να λυθεί το σύστημα: {

Λύ ση : Θεωρούμε την συνάρτηση f(t)= t3+2t2+2t. Είναι: tεR, tl' R (επειδή f' (t)=3t2+4t+2>0, tεR) και f(t) ε R για κάθε t. Όπως και στην προηγούμενη άσκηση Ι το σύστημα παίρνει την Υ = f(x) μορφή: z = f(f(x)) . Η τρίτη εξίσωση του χ = f(f(f(x))) συστήματος είναι ισοδύναμη με την εξίσωση : f(x)=x, ή την χ3+2χ2+2χ= χ, η οποία έχει ρίζες τους αριθμούς Ο και - Ι . Άρα το σύστημα έχει ως λύσεις τις τριάδες: (x,y,z)=(O,O,O), (x,y,z)= (- I ,- Ι ,- Ι ). Ανάλογα (της πρότασης 2) συμπεράσματα για γνησίως φθίνουσες συναρτήσεις δεν ισχύουν. Μερικές φορές όμως η πρόταση ( Ι ) μπορεί να μας βοηθήσει να επιλύσουμε εξισώσεις της μορφής f(f(x))=x.

{

Ας δούμε ένα

ε παράδ ξ θ ε ε Να λυ ί η ίσωση : ι(γχμα.2-5χ+5)2 - 5(χ2-5χ+5)+5=χ.

Λύ ση : Θεωρούμε την συνάρτηση f(x)= χ2-5χ+5,

οπότε η παραπάνω εξίσωση έχει τη μορφή f(f(x))=x. συνάρτηση f είναι ορισμένη στο R και f(xo) ε R για κάθε χ0 ε R, αλλά δεν είναι στο R γνησίως αύξουσα. Όμως από την πρόταση ( Ι ) η εξίσωση f(f(x))=x έχει ως ρίζες της, όλες τις ρίζες της εξίσωσης f(x)=x δηλαδή της εξίσωσης χ2 -5χ+5=χ που έχει ρίζες τους αριθμούς Ι και 5 . Αν τώρα κάνουμε τις πράξεις στην εξίσωση (χ2-5χ+5)2 - 5(χ2-5χ+5)+5=χ καταλήγουμε σ ' ένα πολυώνυμο 4°υ βαθμού με ρίζες Ι και 5 . Αυτό μας επιτρέπει την παραγοντοποίηση του και μετά απ ' αυτό καταλήγουμε στη μορφή : (χ2-6χ+5)(χ24χ+ Ι )= 0. Η τελευταία εξίσωση έχει ρίζες τους αριθμούς Ι ,5, 2- J3 , 2+ J3 που είναι ρίζες και της αρχικής εξίσωσης. Η

Ως εφαρμογή δίνουμε την παρακάτω: Να λυθεί η εξίσωση : (χ2-20 Ι 4χ+20 Ι 4)2 - 20 Ι4(χ220 Ι 4χ+20 Ι4)+20 Ι 4=χ.

ΕΥΚΛΕΙΔΗΣ Β' 93 τ.l/64


ΑΝΑΛΥΤΙΚΕΣ ... ΣΥΖΗΤΗΣΕΙΣ Η γρ απτή αξιολόγηση στα Μ θ η μ α τ ι κ

α

Νίκος Θ. Αντωνόπουλος

ά

Το (Π. Δ. 60 - 2006) μεταξύ άλλων αναφέρεται και στην αξι­ ολόγηση των μαθητών του Λυκείου. Ειδικότερα για τις γρα­ πτές ανακεφαλαιωτικές εξε-τάσεις στα Μαθηματικά το ίδιο Π. Δ. προβλέπει: Ι. Στους μαθητές δίνονται τέσσερα (4) θέματα από την εξε­ ταστέα ύλη, τα οποία μπορούν να αναλύονται σε υποερωτη­ ματα, με τα οποία ελέγχεται η δυνατότητα αναπαραγωγής γνωστικών στοιχείων, η γνώση εννοιών και ορολογίας και η ικανότητα εκτέλεσης γνωστών αλγορίθμων, η ικανότητα του μαθητή να αναλύει, να συνθέτει και να επεξεργάζεται δημι­ ουργικά ένα δεδομένο υλικό, καθώς και η ικανότητα επιλογής και εφαρμογής κατάλληλης μεθόδου. 2. Τα τέσσερα θέματα που δίνονται στους μαθητές διαρθρώ­ νονται ως εξής: α. Το πρώτο θέμα αποτελείται από ερωτήματα θεωρίας που αφορούν έννοιες, ορισμούς, λήμ-ματα, προτάσεις, θεωρήματα και πορίσματα. Με το θέμα αυτό ελέγχεται η κατανόηση των βασικών εννοιών, των σπουδαιότερων συμπερασμάτων, κα­ θώς και η σημασία τους στην οργάνωση μιας λογικής δομής. β. Το δεύτερο και το τρίτο θέμα αποτελείται το καθένα από μία άσκηση που απαιτεί από το μαθητή ικανότητα συνδυα­ σμού και σύνθεσης εννοιών αποδεικτικών ή υπολογιστικών διαδι-κασιών. Η κάθε άσκηση μπορεί να αναλύεται σε επιμέ­ ρους ερωτήματα. γ. Το τέταρτο θέμα αποτελείται από μία άσκηση ή ένα πρό­ βλημα που η λύση του απαιτεί από το μαθητή ικανότητες συνδυασμού και σύνθεσης προηγούμενων γνώσεων, αλλά και την ανάληψη πρωτοβουλιών στη διαδικασία επίλυσής του. Το θέμα αυτό μπορεί να αναλύεται σε επιμέρους ερωτήματα, τα οποία βοηθούν το μαθητή στη λύση. 3. Η βαθμολογία κατανέμεται ανά είκοσι πέντε (25) μονάδες στο καθένα από τα τέσσερα θέματα.

Η

τεχνική της κατασκευής θεμάτων

Με την ευκαιρία της συζήτησης που έχει ξεκινήσει σχετι­ κά με την διεξαγωγή εξετά­ σεων τύπου Πανελληνίων από τράπεζα θεμάτων σε όλες τις Λυκειακές τάξεις, θα επιχειρήσουμε την κατα­ σκευή τέτοιων θεμάτων, τα οποία, σε ότι αφορά στις {I δυο πρώτες τάξεις, αναφέ­ � ρονται στηv τελική αξιολό­ γηση. Τα θέματα θα αναδει­ κνύονται μέσα από υποτιθέμενο διάλογο του διδάσκοντα με τρεις μαθητές, οι οποίοι, σε κάθε περίπτωση, θεωρούμε ότι βρίσκονται στο αντίστοιχο γνωστικό επίπεδο. Καθηγητής: Ας ξεκινήσουμε με ένα θέμα πάνω στις ρίζες μιας εξίσωσης δευτέρου βαθμού. Πότε λέμε ότι μια εξίσωση με άγνωστο χ έχει ρίζα έναν αριθμό ρ; Μαθητής Γ: Όταν την επαληθεύει. Καθηγητής: Δηλαδή; Μαθητής Γ: Όταν αντικαθιστώντας το χ με ρ, τα δυο μέλη της είναι ίσα. Καθηyητής: Ας θεωρούμε την εξίσωση ,( +2χ+1-Κ =0. Μπορούμε να βρούμε για ποιες τιμές του λ έχει ρίζα το μηδέν. Μαθητής Α: Ναι, το ζητούμενο συμβαίνει, αν και μόνο αν, 0 + 2 · 0 + 1 - λ 2 = 0 (1)

και (1) <=> 1 - λ 2 = Ο <=> λ 2 = 1 <=> λ = -1 ή λ = 1 Καθηγητής: Γιατί χρησιμοποίησες το «αν και μόνο αν»; Μαθητής Α: Δεν μας ζητείται να αποδείξουμε κ(ιτι. Μας ζητείται να προσδιορίσουμε «μαθηματικό αντί­ κείμενο» και, όπως έχουμε πει, σ' αυτές τις περιπτώ­ σεις χρειαζόμαστε ισοδυναμίες. Καθηγητής: τι καθορίζει το πλήθος των ριζών της; Μαθητής Γ: Η διακρίνουσα, που είναι Δ=4-4+4λ2=4λ2 Καθηyητής: Ωραία. Για ποιες τιμές του λ έχει μια δΙΠλή ρίζα; Μαθητής Γ: Αν και μόνο αν Δ=Ο, που σημαίνει λ=Ο. Καθηyητής: Και πότε έχει δυο ρίζες πραγματικές και άνισες; Μαθητής Γ: Πάντα! Καθηγητής: Μα, πριν δεν είπαμε ότι όταν ο αριθμός λ είναι μηδέν, έχει διπλή ρίζα; Μαθητής Γ: Α, ναι . . . Έχουμε: Δ>Ο <=> λ :t= Ο . Καθηγητής: Στην περίπτωση που έχουμε δυο ρίζες, χ, , χ 2 και χ 1 χ 2 :t= Ο μπορούμε να υπολογίσουμε το χ χ άθροισμα -, + -2 ,. χ2 χ, Μαθητής Β: Γιατί όχι; Αν κάνουμε ομώνυμα βρίχ2 + χ2 , , , σκουμε -1--2 δηλαδη, εχουμε αθροισμα τετραγωχ,χ 2 νων στον αριθμητή, που ξέρουμε να το χειριζόμαστε και το γινόμενο στον παρονομαστή. Καθηγητής: Και τι γίνεται αν μια ρίζα είναι το μηδέν; Μαθητής Β: Τότε δεν ορίζεται η παράσταση. Ναι, αλ­ λά αυτό δεν είναι έντιμο! Δηλαδή, αν μας ζητήσουν να δουλέψουμε σε μια παράσταση, πέρα από την προσπά­ θεια μας να απαντήσουμε στο ερώτημα, θα έχουμε και την ευθύνη της διερεύνησης μήπως αυτή δεν ορίζεται; Καθηγητής: Κοιτάξτε. Όταν ο «κατασκευαστής» του θέματος ζητάει να βρούμε κάτι, εδώ το συγκεκριμένο άθροισμα, τότε η ευθύνη για την πλήρη διερεύνηση βαρύνει το λύτη, γι' αυτό άλλωστε χρησιμοποιούμε ι­ σοδυναμίες. Απεναντίας, όταν ζητάει να αποδείξουμε κάτι, το αν ορίζονται ή όχι οι παραστάσεις που περιέ­ χονται στο ερώτημα, είναι ευθύνη του «κατασκευα­ στή». Γι' αυτό παραπάνω δώσαμε ότι � · Χι :t=O. Πότε είπαμε ότι η εξίσωση έχει ρίζα το μηδέν; Μαθητής Β: Αν και μόνο αν, λ=- 1 ή λ=l . Καθηγητής: Εντάξει. Ας μετατρέψουμε το ερώτημα σε αποδεικτικό: Αν λε!R- { - 1 , 1 } να αποδείξετε ότι •

λ2 + 2 χ 2 = -­ χ, + Χ ι χ 1 Ι - λ2

Μαθητής Β: Εύκολο. Ισχύει: χ 1 + χ 2 = -2 και χ 1 χ 2 = 1 - λ 2 . 'Ετσι, έχουμε: •

( χ, + χ ι ) 2 - 2 χ,χ 2 χ 1 χ2 �= χ� + χ ;

4 - 2(1 - λ 2 ) = \ - λ2

λ2 + 2 = Ι - λ2

Καθηγητής: Αν τώρα, στη συνέχεια του ερωτήματος, θέλαμε να βρούμε για ποιες τιμές του λ η παράσταση είναι ίση με -2, πως θα συνεχίζαμε;

ΕΥΚΛΕΙΔΗΣ Β' 93 τ.l/65


Αναλυτικές συζητήσεις Μαθητής Α: Στο !R- {- 1 ,

Ι } , έχουμε: � + � = -2 � Χ2

Χι

� λ2 + 2 -2 � λ2 + 2 = 2λ2 -2 �λ=::!2που είναι αποδεκτές. Ι -λ2 Καθηγητής: Ωραία! Ας διατυπώσουμε ολοκληρωμένο το θέμα μας, που θα μπορούσε να είναι δεύτερο θέμα σε μια τελική εξέταση. =

Θέμα 2

Δίνεται η εξίσωση χ 2 + 2χ + 1 - λ 2 = Ο , λε !R . α. Να βρείτε για ποιες τιμές του λ ί. Ο αριθμός Ο είναι ρίζα της. ίί. Η εξίσωση έχει μια διπλή ρίζα. β. Να αποδείξετε ότι η εξίσωση έχει πραγματικές ρίζες. γ. Αν λε !R - { - 1 , 1 } , και χ 1 , χ είναι οι ρίζες της, εξί2 2 ' , χ 2 = λ + 2 και κατοπιν, Χι + σωσης να αποδει'ξετε οτι 2 χ2

Χι

--

1-λ

να βρείτε για ποιες τιμές του λ ισχύει

� + � = -2

Χ2

Χι

Μαθητής Β: Μα αυτό δεν είναι δύσκολο θέμα! Καθηγητής: Ένα θέμα δεν καταξιώνεται από το

βαθμό δυσκολίας του. Αυτό που μας ενδιαφέρει είναι τι θέ­ λουμε να εξετάσουμε βάζοντας το. Μαθητής Β: Εδώ δηλαδή τι εξετάζουμε; Καθηγητής: Κάποια από τα σημεία που εξετάζουμε, είναι κατά πόσο ο μαθητής: • Έχει κατανοήσει την έννοια της ρίζας εξίσωσης. • Έχει ξεκαθαρίσει τα σχετικά με το πλήθος των ρι­ ζών μιας εξίσωσης δευτέρου βαθμού σε συνδυα­ σμό με το πρόσημο της διακρίνουσας. • Είναι σε θέση να χρησιμοποιεί τους τύπους του Vieta για να λύνει απλά ερωτήματα. • Χειρίζεται την ταυτότητα α2+β2=(α+β)2-2αβ, αναγνωρίζει πότε ένα γινόμενο είναι ίσο με το μηδέν κλπ. Μαθητής Γ: Ναι, αλλά ανισώσεις δεν χρησιμοποιήσα­ με πουθενά! Καθηγητής: Μην ξεχνάς ότι αυτό είναι το δεύτερο θέμα. Σε κάποιο από τα επόμενα, φυσικά μπορούμε να έχουμε και aνίσωση. Εξάλλου και εδώ είδαμε μια πολύ βασική ισοδυναμία, που αναφέρεται σε aνίσωση, εννοώ φυσικά την λ2 > Ο � λ '* Ο που συχνά παρερμηνεύεται. Μαθητής Α: Μπορούμε να φτιάξουμε ένα τέτάρτο θέμα; Καθηγητής: Ναι. Ας ξεκινήσουμε εισάγοντας και την έννοια της συνάρτησης που είναι κομβική για την πα­ ραπέρα πορεία μας. Πριν όμως από αυτό να διατυπώ­ σουμε ένα βοηθητικό ερώτημα. Μαθητής Α: Δηλαδή; Καθηγητής: Να παραγοντοποιήσετε το τριώνυμο Ρ(χ)=χ2-4χ+4. Μαθητής Β: Μα αυτό είναι απλό! Καθηγητής: Να θυμάστε ότι σε θέματα που είναι σω­ στά δομημένα, σχεδόν πάντα μπορούμε να βρούμε εύ­ κολες μονάδες. Ποια είναι εδώ η απάντηση; Μαθητής Β : Είναι η ταυτότητα χ2 -4χ+4= (χ-2)2 . Καθηγητής: Ωραία. Ας θεωρήσουμε τη συνάρτηση f(x) _- χ2 - 4χ- Ι χ - 2 1 +4 . Ι χ -2 1 -1

Φαντάζομαι να βλέπετε το τριώνυμο που έχουμε στον αριθμητή «λίγο διασπασμένο». Στο πρώτο ερώτημα να ζητήσουμε να βρεθεί το πεδίο ορισμού της Α και να αποδειχθεί ότι για κάθε χε Α ισχύει f(x)=lx-2 1 . Μαθητής Α: Να δουλέψω εγώ; Καθηγητής: Ναι, πάμε. Μαθητής Α: Η συνάρτηση ορίζεται αν και μόνο αν Ιx2l- 1:;i:O. Θα βρω πρώτα τις τιμές που μηδενίζουν τον παρονομαστή και κατόπιν θα τις εξαιρέσω. Είναι: Ιχ-2 1 =1 � χ-2=1 ή χ-2=- 1 � χ=3 ή x=l . Επομένως, A=IR- { 1 , 3 } . Μαθητής Β: Και γιατί να λύσουμε πρώτα εξίσωση; Δεν μπορούμε να δουλέψουμε με το σύμβολο :;i:; Καθηγητής: Εδώ δεν υπάρχει ιδιαίτερο πρόβλημα. Θα πρέ­ πει να είμαστε όμως πολύ προσεκτικοί και σε ότι αφορά την αλλαγή των συνδέσμων «ή» και «ΚαΙ» αφού ως γνωστό αβ = Ο � α = Ο ή β = Ο και αβ '* Ο � α '* Ο και β 'i' Ο και στην εφαρμογή άλλων ιδιοτήτων. Μαθητής Β: Τι εννοείτε: Καθηγητής: Πιστεύω θυμάστε όλοι ότι, αν α=β και γ=δ τότε α+β=γ+δ. τι λέτε αν βάλουμε :;i: αντί για =; Μαθητής Γ: Υπάρχει πρόβλημα; Καθηγητής: Ας δούμε. Φαντάζομαι να συμφωνεί-τε ότι 3:;i:5 και 8:;i:6. τι λέτε αν προσθέσουμε; Μαθητής Γ: Ωχ! l l:;i:l l . Όχι, δεν ισχύει. Τώρα κατάλαβα γιατί μας είπατε να προτιμάμε να δουλεύουμε με το =. Μαθητής Α: Να συνεχίσω; Καθηγητής: Ναι, πάμε. Μαθητής Α: Για κάθε χε Α, χρησιμοποιώντας και την παραγοντοποίηση του τριωνύμου, έχουμε: I χ - 2 12 - 1 χ - 2 1 f(χ) = (χ - 2)2 - 1 χ - 2 1 = .:... ,....., -:-'-::---'---"" Ι χ - 2 1 -1 Ι χ - 2 1 -Ι , I χ - 2 1 (I χ - 2 1 -Ι) το ζητουμενο. που ειναι , l----,--'-"---,---:-'--"- =Ι χ - 2 1 1 χ - 2 1 -ι Εδω, χρησιμοποιησαμε την ισοτητα α2= I α 1 2 . , , Καθηγητής: Τι λέτε τώρα. Μπορούμε να κάνουμε τη γραφική παράσταση της για χ ε Α με χ�2; Μαθητής Β: Εύκολο! Με χ�2, έχουμε f(x)=x-2, οπότε είναι μια ημιευθεία με αρχή το Μ(2, 0). Καθηγητής: Θα μπορούσες να τη σχεδιάσεις; Μαθητής Β: Ναι, είναι απλό. Η γραφική παράσταση είναι αυτή που φαίνεται στο σχήμα.

'2

Καθηγητής: Τι λέτε οι άλλοι για το σχήμα; Μαθητής Α: Ναι αφού μας βγήκε απόλυτο

δεν «πέ­ φτει» κάτω από τον χ 'χ. Μαθητής Γ: Ναι, το ίδιο λέω και εγώ. Στο (2, Ο) έχει βάλει «χοντρή τελεία». Καθηγητής: Τι θα λέγατε για την τιμή χ=3 ; Μαθητής Β: Βγαίνει y= 1 . Μαθητής Α: Όχι. Δεν μπορούμε να βάλουμε όπου χ το 3. Το απαγορεύει το πεδίο ορισμού. Καθηγητής: Τι λέτε τότε για τη γραφική παράσταση.

ΕΥΚΛΕΙΔΗΣ Β' 93 τ. l /66


Αναλυτικές συζητήσεις Καθηγητής: Βεβαίως, αλλά κάποιος, εδώ, μπορεί να Να την ξαναδούμε; Μαθητής Α: Ναι. Θα αφήσουμε έξω το σημείο Ν(3, 1) το βρει και χωρίς αυτή την παρατήρηση. Αυτό που λες οπότε είναι αυτή εδώ. βέβαια θα ήταν απαραίτητο αν ζητούσαμε το άθροισμα των συντελεστών του πολυωνύμου Π(χ)=(χz-χ+ 1 )zoι 3+(xz+x- 1 ) zοι 3 Μαθητής Β: Καλά, τότε . . . καληνύχτα. Μαθητής Α: Μα, γιατί το λες αυτό; Το άθροισμα των συντελεστών είναι ίσο με Π( 1 )= 1 + 1 =2. Καθηγητής: Τι λέτε τώρα μπορούμε να συνεχίσουμε Καθηγητής: τι λέτε τώρα για τον υπολογισμό της παμε τη απάντηση του ερωτήματος; 1 + 1 , Μαθητής Β: Ναι. Οι αριθμοί α, β ικανοποιούν τις α­ π= ραστασης 1 + )f(5) 1 - )f(5) παιτήσεις του προβλήματος, αν και μόνο αν: 2 - α -1-1 - 3 + β -1 = -3 ( Σ ) Μαθητής Γ : Προφανώς f(5)=3, οπότε ·1

--.====

2 1 - .J3 + 1 + .J3 1 1 Π=- + __ = - 1 .J3 .J3 1 + .J3 1 - .JJ (l + )(1 - ) 1 - 3 _

_ _

_ _

Καθηγητής: Αυτό θα μπορούσε να είναι τέταρτο θέμα. Αντιλαμβάνεστε ότι στο τρίτο θα έπρεπε να έχουμε ε­ ρωτήματα με μια ή Περισσότερες ανισώ-σεις που ου­ σιαστικά δεν έχουμε ζητήσει. Μαθητής Γ: Πολύ καλό, αλλά αυτό το σημείο που ε­ ξαιρείται ποιος θα το έβλεπε; Καθηγητής: Δεν είναι κακό στο τέταρτο θέμα να υπάρχει κάποιο ερώτημα που απαιτεί αυξημένη ετοιμότητα από την πλευρά των μαθητών. Ας το διατυπώσουμε λοιπόν: Θέμα 4 α. Να παραγοντοποιήσετε το τριώνυμο Ρ(χ)= χ2 -4χ+4. t1(χ) = χ 2 - 4χ- Ι χ - 2 1 +4 . , , τη συναρτηση β. Θ " εωρουμε

Ι χ - 2 1 -1 Να βρείτε το πεδίο ορισμού της Α και να απο­ δείξετε ότι, για κάθε χεΑ ισχύει f(χ)=Ιχ-2 1 . ii. Να κατασκευάσετε τη γραφική παράσταση της συνάρτησης, όταν χε Α και χ�2. i ii. Να υπολογίσετε την τιμή της παράστασης Π= 1 + 1 . I + )f(5) 1 - )f(5) Μαθητής Β. Γιατί στο (i) ερώτημα δίνεται το αποτέλε­ σμα ενώ στο (iii) δεν δίνεται; Καθηγητής: Διότι, το αποτέλεσμα του (i) είναι καθο­ ριστικό για τη συνέχιση της ορθής λύσης. Αν δεν γίνει η απλοποίηση, ή γίνει λάθος απλοποίηση, το αποτέλε­ σμα επηρεάζει τα επόμενα ερωτήματα. Μαθητής Γ: Αν κάποιος μαθητής δεν αποδείξει το (i) μπορεί να χρησιμοποιήσει το αποτέλεσμά του για τα επόμενα ερωτήματα; Καθηγητής: Φυσικά. . . . . . . . . . . . . . . . . . . . . . . . . . Καθηγητής: Ας περάσουμε τώρα στην Άλγεβρα της Β ' Λυκείου και συγκεκριμένα στα πολυώνυμα και τις πο­ λυωνυμικές εξισώσεις. Να θεωρήσουμε λοιπόν το πολυώνυμο Ρ(χ)=2χ4-(α+ 1 )χ3-χ2 -3χ+β- 1 Τι λέτε, μπορούμε να βρούμε τους αριθμούς α, β ώστε το άθροισμα των συντελεστών του πολυωνύμου να εί­ ναι -3 και ο αριθμός 2 να είναι ρίζα του; Μαθητής Β: Ξαφνιάζει λίγο το άθροισμα των συντε­ λεστών, αλλά το ερώτημα είναι πολύ συνηθισμένο. Καθηγητής: Ναι, αλλά με αυτό ελέγχουμε τόσο την αντίληψη των εννοιών όσο και την εξοικείωση με δια­ δικασίες, όπως εδώ η επίλυση απλού συστήματος. Μαθητής Γ: Το άθροισμα συντελεστών δεν είναι το Ρ( 1 ); i.

..

των

και

{

{

{

32 - 8α -8 - 4 - 6 + β -1 = Ο

(Σ ) � -α + β = 1

-8α + β = -13

{

-α + β = 1 � α = 2 7α =14 β =3

Μαθητής Β: Γιατί χρησιμοποιείς ισοδυναμίες; Μαθητής Α: Διότι αναζητάμε κάποιο «μαθηματικό α­ ντικείμενο», εδώ τους αριθμούς α, β. Μαθητής Β: Δηλαδή, αν μας έλεγε: «Αν το άθροισμα των συντελεστών του πολυωνύμου είναι ίσο με -3 και το πολυώνυμο έχει ρίζα τον αριθμό 2, να αποδείξετε ότι α=2 και β=3, θα βάζαμε συνεπαγωγές;» Καθηγητής: Βεβαίως! Η συνεπαγωγή μας οδηγεί στο συμπέρασμα ότι αν υπάρχουν τέτοιοι αριθμοί, αυτοί εί­ ναι οι α=2 και β=3. Μαθητής Γ: Και αν δεν υπάρχουν; Καθηγητής: Αυτό δεν ακυρώνει τη λύση. Απλά θα έ­ χουμε μια συνεπαγωγή με ψευδή υπόθεση, και σε μια τέτοια, το συμπέρασμα μπορεί να είναι οποιοδήποτε. Η συνεπαγωγή είναι αληθής. Φυσικά, τέτοιες συνεχαyrο­ γές καλό είναι να αποφεύγονται σε εξετάσει;. Μαθητής Β: Κι αν, σε μια τέτοια περίπτωση, ι..-άΛοιος μαθητής καταλάβει από την αρχή ότι η υπόθεση είναι ψευδής, τότε τι γίνεται; Καθηγητής: Αυτός μπορεί να απαιτήσει αμέσως. Αλ­ λά αφού πήγαμε εδώ τη συζήτηση, λέτε να παρεμβά­ λουμε ένα παράδειγμα; Μαθητής Γ: Πάνω στο τελευταίο; Καθηγητής: Ναι. « Αν για τον πραγματικό αριθμό α ισχύει α2+α+ 1 =0, να αποδείξετε ότι α= 1 ». Μαθητής Γ: Αν ονομάσουμε (1) τη σχέση μας δώσατε πολλαπλασιάσουμε επί α έχουμε α3-!-iC+α=Ο (2), οπότε, αν ωrό την (2) αφαιρέσουμε την ( 1) βρίσκουμε α3= 1, δηλαδή, 1. Μαθητής Β: Μα η α=1 δεν επαληθεύει την ( 1 ). Καθηγητής: Το γεγονός ότι η τιμή α=1 δεν επα­ ληθεύει την ( 1 ) δεν σημαίνει ότι υπάρχει κάποιο πρό­ βλημα με τη λύση του συμμαθητή σας που είναι άψογη. Απλά, εδώ έχουμε συνεπαγωγή με ψευδή υπόθεση. Μαθητής Α: Κάποιος που θα καταλάβει ότι η υπόθεση είναι ψευδής (ας πούμε θεωρώντας το πρώτο μέλος τριώνυμο με Δ= -3) θα μπορούσε να κάνει κάτι άλλο; Καθηγητής: Ναι. Κάποιος μαθητής θα μπορούσε να γράψει, και να είναι επίσης άψογος, ότι: «παρατηρώ ότι τι

και

που

α=

( � 2 + � > Ο οπότε η υπόθεση είναι ψευ­ J δής και κατά συνέπεια, το ζητούμενο προκύπτει άμεσα». α 2 + α +1 = α +

Μαθητής Α: Χωρίς να ασχοληθεί καθόλου με το συ­ μπέρασμα;

ΕΥΚΛΕΙΔΗΣ Β' 93 τ.l/67


Αναλυτικές συζητήσεις Καθηγητής: Καθόλου. Το συμπέρασμα θα μπορούσε της. Κατόπιν, να αποδείξετε ότι αν π(χ) είναι το πηλίκο της να είναι οτιδήποτε. Ακόμα και α2<0, που προφανώς εί­ παραπάνω διαίρεσης, τότε, για κάθε χ ε IR , π(χ) ;;:: � . ναι ψευδές, απαντώντας όπως παραπάνω, η λύση του, 4 δεν θα είχε κανένα πρόβλημα. Αλλά, ας συνεχίσουμε Μαθητής Β: Εδώ, ουσιαστικά, πάλι έχουμε το αποτέλεσμα. με τα ερωτήματα του θέματός μας. Φυσικά θα δώσουμε Καθηγητής: Ναι. Αλλά εξετάζουμε επίσης τόσο τη δι­ τα αποτελέσματα. Έτσι θα μπορούσαμε να συνεχίσου­ αδικασία της διαίρεσης με διαιρέτη δευτέρου βαθμού, με ως εξής: Αν α=2 και β= 3, τότε: Να αποδείξετε ότι όσο και την δυνατότητα σχηματισμού της ταυτότητας. Κάποιος που είχε λύσει σωστά τα προηγούμενα, θα ο αριθμός .!.. είναι ρίζα του πολυωνύμου. μπορούσε να διαπιστώσει ότι, το σωστό αποτέλεσμα 2 Μαθητής Β: Εύκολο. Με α=2 και β=3, είναι είναι Ρ(χ)= (2χ2 -5χ+2)(χ2+χ+ 1 ) Ρ(χ)=2χ4-3χ3-χ2-3χ+2 οπότε έχουμε: Μαθητής Α: Το πηλίκο είναι π(χ)=χ2+χ+ 1 , οπότε ουσιαστικά το τε1 1 Ρ = 2 - 3 - -3 + 2 = - - - + = 0 λευταίο υποερώτημα το αποδείξαμε 1 προηγουμένως στη συζήτηση για τις ,:.', ��� συνεπαγωγές με ψευδή υπόθεση. Άρα, πράγματι ο αριθμός .!.. είναι ρίζα του. 2 Μαθητής Γ: Δηλαδή ήθελε οπωσδήποτε συμπλήρωση Καθηγητής: Ωραία! Συνεχίζουμε τα ερωτήματα. τετραγώνου; Να λύσετε την εξίσωση Ρ(χ)=Ο. Καθηγητής: Δεν είναι απαραίτητο. Κάποιος θα μποΜαθητής Γ: Να κάνουμε σχήμα Homer πρώτα με το 1 . Μαθητής Β : Μα, αφού μας έχει δώσει ρίζες! Εμείς θα ρούσε να γράψει: Αρκεί: χ2+χ+ 1 ;;:: � , ή αρκεί, χ2+χ+ 1 4 πάμε με δοκιμές; Να κάνουμε διαδοχικά σχήματα 3> 1 , διοτι , απορρεει 1 4 Ο η αρκει χ + χ + 4 ;;:: Ο που ισχυει, Homer με το 2 και το - ; 2 Καθηγητής: Σωστά. 2:0. Πως θα το χαρακτηρί­ Μαθητής Β: Αν κάνουμε αυτό που είπα βρίσκουμε ό­ από την ταυτότητα χ 2 2 τι: Ρ(χ)=Ο <=> (2χ -5χ+2)(χ +χ+ 1 )=Ο Και, όπως είπαμε ζατε σαν τρίτο θέμα; προηγουμένως x2+x+1:;i:O για κάθε χε!R, οπότε οι μοΜαθητής Β: Δεν βλέπω κάποια ιδιαίτερη δυσκολία. Καθηγητής: Έχουμε ήδη αναφέρει ότι ο σκοπός μας δεν είναι να φτιάξουμε δύσκολα θέματα, αλλά θέματα με τα ναδικές ρίζες της εξίσωσης είναι οι αριθμοί 2 και .!_ . 2 οποία θα ανιχνεύουμε αν πετύχαμε ή όχι τους στόχους που Καθηγητής: Πολύ σωστά. είχαμε για το συγκεκριμένο κεφάλαιο. Πάντως, αν θέλαμε Μαθητής Γ: Γιατί έγραψε «και» ανάμεσα στις ρίζες να το κάνουμε λίγο δύσκολο, θα μπορούσαμε να αντι­ και όχι «ή»; καταστήσουμε τα ερωτήματα β(ί. και ii.) με το εξής: «Να Καθηγητής: Εδώ ο συμμαθητής σου απλώς έγραψε τα αποδείξετε ότι για κάθε ρίζα ρ της εξίσωσης Ρ(χ)=Ο, ο αστοιχεία του συνόλου που σχηματίζουν οι λύσεις της ριθμός .!.. είναι επίσης ρίζα της και να λύσετε την εξίσωση εξίσωσης που είναι το Λ={ .!.. , 2 } . Άρα, σωστά είπε 2 ρ 2 Ρ(χ)=Ο». Ας αφήσουμε όμως την αρχική μας διατύπωση: 1 Θ Θέ μα 3 και . α μπορουσε, αντ , αυτου,, να γραψει, i Δίνεται το πολυώνυμο Ρ(χ)=2χ4-( α+ 1 )χ3 -χ2-3χ+β- 1 α . Να βρείτε τους αριθμούς α, β ώστε το άθροισμα των Ρ(χ) = Ο <=> χ 2 ή χ = .!.. <=> χ ε {.!_ , 2} . συντελεστών του πολυωνύμου να είναι -3 και ο αριθ­ 2 2 μός 2 να είναι ρίζα του. Από τις ρίζες ποια θα μπορούσε να είναι το συν­ β. Αν α=2 και β= 3, τότε: ημίτονο μιας γωνίας; ί.Να αποδείξετε ότι ο αριθμός .!.. είναι ρίζα του Ρ(χ). Μαθητής Α: Μόνο το .!_ . 2 2 ίί.Να λύσετε την εξίσωση Ρ(χ)=Ο. Καθηγητής: Ωραία. Να ζητήσουμε λοιπόν να λυθεί η ίίί. Να βρείτε για ποιες τιμές του θ ισχύει Ρ(συνθ)=Ο εξίσωση Ρ(συνθ)=Ο. ίν. Να κάνετε τη διαίρεση Ρ(χ): (2χ2 -5χ+2) και να γρά­ Μαθητής Α: Αν θέσουμε όπου συνθ το u, έχουμε 2u4-3u3-u2-3u+2=0 Οι ρίζες της τελευταίας είναι οι αριθμοί ψετε την ταυτότητά της. ν. Αν π(χ) είναι το πηλίκο της παραπάνω διαίρεσης, να 2 και .!.. . Προφανώς η εξίσωση συνθ=2 είναι αδύνατη, οπό2 αποδείξετε ότι για κάθε χ ε IR , π(χ) ;;:: � . 4 τε τελικά έχουμε να λύσουμε μόνο την εξίσωση συνθ= Καθηγητής: Ας περάσουμε τώρα να φτιάξουμε ένα 4° θέμα με κορμό την εκθετική και λογαριθμική συνάρτη­ � ση. Ξεκινάμε λοιπόν με το πρώτο ερώτημα: Να λύσέτε που έχει λύσεις τους αριθμούς θ με θ = 2κπ± , κ εΖ. 3 τις ανισώσεις 2χ<1 και 2'>2. Καθηγητής: Να δούμε και ένα ακόμα ερώτημα: Να κάνετε Μαθητής Γ: Εύκολο. Είναι: 2' < 1 <=> 2χ < 2° <=> χ < Ο τη διαίρεση Ρ(χ):(2χ2-5χ+2) και να γράψετε την ταυτότητά Και 2' > 2 <=> 2χ > 21 <=> χ > 1

Δ

� i� � i�% : :

,

_

,, , 2

( +υ2

πιο

w

,

,

=

ΕΥΚΛΕΙΔΗΣ Β' 93 τ.l/68

' · ·

,


Αναλυτικές συζητήσεις ίί. Να αποδείξετε ότι f(2)-f(- l )=lη8. Μαθητής Β: Πάμε σίγουρα για 4° θέμα; Καθηγητής: Μην βιάζεστε! Ας θεωρήσουμε τώρα τη iii. Αν κ Ε .IR ώστε 4" - 3 · 2" + Ι = Ο , να συγκρίνετε συνάρτηση f(x)= Ιη(4'-3 · 2'+2). Τι λέτε, μπορούμε να τους αριθμούς f(- I) και f(κ). βρούμε το πεδίο ορισμού της Α; Μαθητής Β: Με την Τριγωνομετρία δεν ασχοληθήκα­ Μαθητής Γ : Ναι. Πρέπει: 4'-3 · 2'+2>0 με καθόλου. Καθηγητής: Μια στιγμή! Να ξεκαθαρίσουμε ένα λε­ Εντάξει. Ας δούμε και ένα θέμα τριγωνο­ πτό σημείο. Αν σου ζητούσα το πεδίο ορισμού της Καθηγητής: μετρίας. Να ξεκινήσουμε πάλι με μια συνάρτηση και ας h(x)=lη(x- I ), τι θα έλεγες; θεωρήσουμε την f(χ)=συνχ-2ημχ+(συνχ-ημχ)2 Μαθητής Γ: Ότι, πρέπει χ> Ι . Να ζητήσουμε ως πρώτο ερώτημα να αποδειχθεί ότι Καθηγητής: Αν κάποιος σου έλεγε ότι πρέπει χ>Ο, τι f(x)=(συνχ+ Ι )(1 �2ημχ). θα του έλεγες; Μαθητής Β: f(χ)=συνχ-2ημχ+συν2χ+ημ2χ-2ημχσυνχ= Μαθητής Γ: Ότι, είναι λάθος. =Ι +συνχ-2ημχ-2ημχσυνχ= Ι +συνχ-2ημχ( Ι +συνχ)= Καθηγητής: Γιατί, θα μπορούσαμε να ορίσουμε τη =( Ι+συνχ)(Ι -ημ2χ) που είναι το ζητούμενο. συνάρτηση για κάποιο χ με χ:=:;Ο; Καθηγητής: Τώρα, με δεδομένη την παραγοντοποίη­ ση, μπορούμε να λύσουμε την εξίσωση f(x)=O; Μαθητής Γ : Όχι βέβαια! Μαθητής Β: Μήπως να λέγαμε, αρκεί: χ> Ι . Μαθητής Β: Ναι, δεν είναι δύσκολο. Η f(x)=O διασπά­ Καθηγητής: Κι αν κάποιος μας έλεγε ότι αρκεί χ>2, ται σε δυο εξισώσεις, τις συνχ= - Ι , (Ι) και ημ2χ= Ι , (2) θα ήταν λάθος; με (Ι) <:::} συνχ=-Ι <:::} χ=2κπ+π, κΕΖ Μαθητής Β: Το χ>2 βεβαίως είναι αρκετό, αλλά δεν π π είναι απαραίτητο. Θα μπορούσε να είναι χ=Ι ,5. (2) <:::} η μ2χ = Ι <:::} 2χ=2κπ+- <:::} χ=κπ+- , ΚΕΖ. 2 4 Μαθητής Α: Μάλλον η σωστή έκφραση έπρεπε να χρη­ Καθηγητής: Αυτό που έγραψες στην πρώτη περίπτωσψοποιήσουμε από την αρχή είναι <<Πρέπει και αρκεί, χ> Ι». Καθηγητής: Ακριβώς! Αυτό είναι το σωστό. Αλλά ας ση, δηλαδή χ=2κπ+π, ΚΕΖ, τι σημαίνει; Υπάρχει ακέ­ επιστρέψουμε τώρα στο θέμα μας. ραιος κ ώστε να ισχύει η ισότητα, ή αυτή ισχύει για Μαθητής Γ: Να συνεχίσω λοιπόν. Πρέπει και αρκεί: κάθε ακέραιο κ; 4'-3 · 2'+2>0 (Ι) Αν θέσουμε 2'=y, τότε η ανίσωση Μαθητής Β: Για κάθε ακέραιο κ. που θα μας βοηθήσει να λύσουμε την (Ι) είναι η y- Καθηγητής: Ο αριθμός 5π είναι λύση της εξίσωσης; 3y+2>0. Εύκολα βρίσκουμε ότι αυτή έχει λύσεις τους Μαθητής Β: Ναι. αριθμούς y για τους οποίους ισχύει y<Ιή y>2. Αυτό Καθηγητής : Προκύπtει για κάθε ακέραιο κ; Δηλαδή σημαίνει ότι ( 1 ) <:::} 2'<Ι ή 2'>2 <:::} χ<Ο ή χ> Ι όπως για οποιοδήποτε ακέραιο κ από την λύση που έχουμε προκύπτει από το πρώτο ερώτημα. παραπάνω προκύπτει ότι χ=5π; Καθηγητής: Να βάλουμε και ένα υπολογιστικό ερώ­ Μαθητής Β: Αν μιλάμε για συγκεκριμένη λύση, όχι. τημα. Ας υπολογίσουμε τη διαφορά f(2)-f(- I ). Μάλλον θα πρέπει να πούμε «υπάp-_.(ει». Μαθητής Γ: Είναι: f(2)=lη(I6- I2+2)=lη6 Καθηγητής: Αν θέλουμε να είμαστε αιφιβεi.; θα πρέm;ι Ι-6+8 3 , Ι -3 τονίσουμε ότι εννοούμε πως, για κάθε αριθμό εiναι iir και f(- Ι )= !η - - + 2 = !η = lη - οποτε ση της εξίσωσης, υπάρχει ακέραιος ώστε ισχύει 4 2 4 4 χ=2ισt+π. Κοιτάξτε η ε­ που μπορούσε μας οδηγήσει f(2)-f( -Ι )=Ιη2+1η3-Ιη3+1η4=Ιη2+21η2=31η2=lη8. σφαλμένη αντίληψη για τον ακέραιο Θυμάστε ποια σχέση Μαθητής Β: Καλά πήγαμε μέχρι εδώ. Καθηγητής: Αν υποθέσουμε ότι για κάποιο πραγματι­ συνδέει ημίτονα των παραπληρωματικών γωνιών; κό αριθμό κ ισχύει 4κ-3 · 2κ+ Ι =0, ποιος νομίζετε ότι εί­ Μαθητής Γ : Είναι ίσα. Καθηγητής: Δηλαδή η εξίσωση ημχ=ημ(π-χ) πόσες ναι ο πιο μεγάλος από τους αριθμούς f(- Ι ) και f(κ); λύσεις έχει; Μαθητής Γ: Το f(- I ) το βρήκαμε. Είναι ln3-lη4. Μαθητής Γ: Κάθε πραγματικό αριθμό χ. Καθηγητής: Με το f(κ) τι κάνουμε; Καθηγητής: Ωραία. Μπορείτε σας παρακαλώ να μου Μαθητής Γ: Από τη σχέση που μας έδωσαν να βρούμε την λύσετε με τη βοήθεια των τύπων; το κ και να αντικαταστήσουμε. Μαθητής Γ: Γιατί όχι. Έχουμε: ημχ=ημ(π-χ) Καθηγητής: Είστε σίγουροι; Μαθητής Α: Μάλλον όχι. Κάνοντας λίγο γρήγορα τις χ=2κπ+π-χ ή χ = κπ + � ή <:::} <:::} 2 3± χ=2κπ+π-π+χ, κ Ε Ζ πρα' ξεις καταληγω ' στην 2" = -J5 . Δεν φαντα' ζο μαι 0 · Χ = 2 ΚΠ, Κ Ε ΙLJ 2 Μαθητής Β: Μα . . . αυτό δεν γίνεται! Δηλαδή η εξίσω­ ότι πρέπει να συνεχίσουμε έτσι. ση έχει μόνο την πρώτη λύση; Καθηγητής: Σίγουρα θυμάστε τον τύπο της συνάρτησης; Μαθητής Β: Ωχ, ναι Το f{κ) είναι ίσο με ln(O+I) δηλαδή εί­ Καθηγητής: Δεν συμφωνήσαμε στην αρχή ότι έχει λύση κάθε πραγματικό αριθμό χ; ναι μηδέν. Επίσης, ln3<ln4, οπότε f{-I)<O. Άρα, f{κ)>f{-I). Μαθητής Α: Προφανώς κάτι πρέπει να γίνεται στη Καθηγητής: Πολύ ωραία. Ας διατυπώσουμε τώρα και δεύτερη περίπτωση. το θέμα αυτό. Καθηγητής: Ας το διατυπώσουμε αναλυτικά. Καταλήξα­ Θέ μα 4 με ότι, για οποιαδήποτε λύση χ της εξίσωσης, υπάρχει αα . Να λύσετε τις ανισώσεις: 2 <Ι και 2 >2. π β. Δίνεται η συνάρτηση f(x)=lη(4'-3 · 2'+2). , ' να ισχυει ' χ = κπ + "2 η' ο · χ = 2 κπ . κεραιος ωστε ί. Να βρείτε το πεδίο ορισμού της. που

(

)

να

χ

κ

ΕΥΚΛΕΙΔΗΣ Β' 93 τ.l/69

'u

ιc.

τα

{

κ

να

χ που

{

'7l


Αναλυτικές συζητήσεις Προφανώς για κάθε χ ε IR. υπάρχει ακέραιος κ και συ­ Ι στροφη της g(x)= + 1 ; γκεκριμένα κ=Ο ώστε Ο · χ = 2κπ . Επομένως, η παρα­ 2χ πάνω διάζευξη είναι αληθής για κάθε χ IR. . Καθηγητής: Ακριβώς! Μαθητής Α: Κατάλαβα, για τη συγκεκριμένη τιμή του κ, Μαθητής Β: Πρώτα θα αποδείξουμε ότι η g είναι «ένα κάθε πραγματικός αριθμός χ είναι λύση της εξίσωσης. Καθηγητής: Ακριβώς. Ας βάλουμε όμως άλλο ένα ε­ προς ένα». Για κάθε χ ι , χ2 ε!R.* με g(x 1 )=g(x2) έχουμε: ρώτημα στο θέμα μας που να σχετίζεται με την αναγω­ Ι 1 g(x, ) = g(x ) ::::} - + j = - +J :::} Χι = χ . Ά , ρα, πραγμαγή. Τι λέτε, μπορείτε να υπολογίσετε το f(χ+π)+f(π-χ); 2 2 2χ1 2χ2 Μαθητής Γ : Είναι: f(χ+π)=[συν(χ+π)+1][ 1-2ημ(π+χ)]=(-συνχ+1)(1+2η μχ τικά η g είναι «ένα προς ένα», οπότε αντιστρέφεται. και f(π-χ)=[ συν(π-χ)+ 1 ][ 1 -2ημ(π-χ) ] =(-συνχ+ 1)( 1 -2ημχ) Μαθητής Γ : Να βρω εγώ τον τύπο της g· 1 ; Οπότε f(χ+π)+f(π-χ)=(-συνχ+1)(1+2ημχ+1-2ημχ)=2(1-συνχ) Καθηγητής: Ναι! Πάμε. Καθηγητής: Επομένως, μπορούμε να διατυπώσουμε Μαθητής Γ: Θα λύσουμε, ως προς χ ε IR. , την εξίσω­ ως δεύτερο θέμα το επόμενο: ση g(x)=y, με yε !R.. Είναι: Θέμα 2 Δίνεται η συνάρτηση f(χ)=συνχ-2ημχ+( συνχ-ημχ) 2 g(x) = y _!_ +l = y _!_ = y- 1 2x(y- 1) = 1, (1) <:::> 2χ <:::> 2χ <:::> α. Να αποδείξετε ότι f(χ)=(συνχ+1 )( 1 -2ημχ) ;;�; Ο χ χ ε !R. X ;;f; Ο X ;;f; Ο β.Να λύσετε την εξίσωση f(x)=O. γ. Να αποδείξετε ότι f(χ+π)+f(π-χ)=2(1 -συνχ) Αν y= 1 , τότε η ( Ι ) είναι αδύνατη, οπότε y ;;�; 1 . 1 Καθηγητής: Στη Γ Λυκείου έχουμε Πανελλήνιες ε­ Με y * 1 έχουμε: (1) <:::> χ = -- . Η τιμή που προέ2(y1) ξετάσεις και οπωσδήποτε, τα θέματα, αν και διατηρούν την ίδια δομή, είναι πιο σύνθετα. Ωστόσο, αν κάποιος κυψε επαληθεύει τον περιορισμό χ * Ο , οπότε περιέχε­ είναι ιδιαίτερα προσεκτικός, και μέσα σ' αυτά τα θέ­ ται στο πεδίο ορισμού της g. Επομένως, το σύνολο τι­ μών της g είναι το IR. - {1} ματα, μπορεί να βρει εύκολα ερωτήματα. Μαθητής Γ: Μπορούμε να δούμε ένα θέμα Ανάλυσης; 1 Μαθητής Β: Καλύτερα, αν γίνεται, να είναι από τις οπότε g· ι : IR.- { l } �IR. με g " (x) = -- , x ;;�; 1 που 2(χ - 1 ) ενότητες που συνήθως «αδικούνται» στις εξετάσεις. πραγματικά είναι ίση με f(x1). Καθηγητής: Εννοείς ερωτήματα που αναφέρονται στις Μαθητής Β: Την επαλήθευση του περιορισμού χ ;;�; Ο ενότητες που συναντάμε πριν την παράγωγο. γιατί την ανέφερες; Δεν ισχύει πάντα; Μαθητής Β: Ακριβώς! Καθηγητής: ΕντάξεL Ας δούμε ένα θέμα το οποίο, θεωρη­ Καθηγητής: Δηλαδή, θεωρείς ότι ο συγκεκριμένος έ­ τικά, θα μπορούσε να τεθεί σαν τέταρτο σε διαγώνισμα του λεγχος είναι περιττός; Α ' τετραμήνου. Ξεκινάμε με το πρώτο ερώτημα. Αν για μια Μαθητής Β: Έτσι νομίζω. συνάρτηση f:JR.'�JR., για κάθε x,yε!R.* ισχύει xf(y)- Καθηγητής: 2 Ωραία. Ας πάρουμε τη συνάρτηση (2χ + 1) π ' ορισμού της; ' το πεδιο φ (χ ) = . οιο ειναι 4χ 2 + 2χ xyf(x)f(y)2: να αποδείξετε ότι f(x)= 21χ , για κάθε x:;tO.

-

ε

{

{

{

{

1

±

Μαθητής Γ: Από ανισότητα θα καταλήξουμε σε ισό­ τητα; Μαθητής Α: Γιατί σου φαίνεται περίεργp; Δεν θα μπορσό­ σε να προκύπτει από την αντισυμμετρική' ιδιότητα ή, mo ειδικά, να καταλήξουμε σε σχέση της μορφής [Α(χ)]2 � 0. Καθηγητής: Τι λέτε, μπορούμε να ξεκινήσουμε; Μαθητής Α: Ναι . . . Συνήθως σε τέτοια ερωτήματα βάζουμε αντί για χ ή y το μηδέν, (εδώ δεν μπορούμε) ή το ένα ή θέτουμε αντί για y το χ, κτλ. Εδώ με y=x, έχουμε: xf(x) - x2f2 (x) � -1 ::::} x2f2 (x) - xf(x ) + -1 � .0 4 4 ::::}

(

xf(x) -

�) � Ο Καλά έλεγες. Εύκολα τώρα βρί2

1 σκουμε ότι f(x) = - , για κάθε x:;t:O. 2χ Καθηγητής: Πάμε σε δεύτερο ερώτημα. Αν g(χ)=f(χ)+ 1 , να αποδείξετε ότι η g αντιστρέφεται και g · \x)=f(x- 1 ), για κάθε χ ;;�; 1 . Μαθητής Β: Δηλαδή τώρα πρέπει να βρούμε την αντί-

Μαθητής Β: . . . 2x(2x+l)=O . . . . Είναι το A=IR.- {- .!.. , Ο } 2 Καθηγητής: Ωραία. Μπορείς να aπλοποιήσεις τον τύ­ πο της; (2χ +1)2 (2χ +1)2 2χ +1 = 1 ? . -Μαθητή ς Β: Ναι q:(x) =2χ 4x _ 2 +2χ 2χ(2χ + 1) 2χ Είναι η g. Καθηγητής: Σίγουρα; Μαθητής Α: Όχι, δεν έχει το ίδιο πεδίο ορισμού με την g. Καθηγητής: Ωραία. Είναι διαφορετική από την g. Αν τώ­ ρα επαναλάβουμε τη διαδικασία για τον προσδιορισμό του συνόλου τιμών της, σύμφωνα με τη διαδικασία που περιγράψαμε παραπάνω θα καταλήξουμε στη διπλή διάζευξη 2(y-1)x = 1, (2) και χ ;;�; Ο και χ ;;�; -! Όπως και 2 1 προηγουμένως βρίσκουμε ότι (2) <:::> χ -- , y ;;�; 1 Τι 2(y-1) λέτε τώρα, η τιμή του χ που βρήκαμε περιέχεται στο πε­ δίο ορισμού της φ, όπως άφησες να εννοηθεί; Μαθητής Β: Δηλαδή τώρα η λύση που βρήκαμε δεν θα επαληθεύει τον περιορισμό χ ;;�; Ο ;

ΕΥΚΛΕΙΔΗΣ Β' 93 τ.l/70

=


Αναλυτικές συζητήσεις Καθηγητής: Για τον περιορισμό αυτό, όπως ήδη είδα­ Να υπολογίσετε το όριο lim (χ 2 + 1 )f(η μ 2 χ) . με δεν υπάρχει πρόβλημα. Ας επιχειρήσουμε να δούμε όμως τι συμβαίνει με τον δεύτερο περιορισμό. Μαθητής Β: Είναι: lim (x2 + I)f{ημ2x) = lim (x2 + I)-I2ημ2Χ Μαθητής Α: Θα λύσουμε την εξίσωση χ = _ .!._ ; Ι . Ι 2 η μχ ·2 = +οο Ιιm ( ! + -)(-) 2 χ Καθηγητής: Καλύτερα θα εξετάσουμε αν υπάρχει κά­ 2 χ --+ «> χ + ποια τιμή του y με y * 1 ώστε ο χ που προσδιορίστηκε Ι ημχ Διότι lim (1 + _!_) = 1 και lim _ _ = Ο => Iim = +οο χ με την επίλυση της (2) να είναι ίσος με χ = - .!._ . χ -++«> ( ημχ ? Χ Χ-++«> 2 χ Μαθητής Α: Να συνεχίσω εγώ; Μαθητής Γ: Το τελευταίο «=>» δεν το πολύ-κατάλαβα Καθηγητής: Ναι, πάμε. Μαθητής Α: Με y * 1 έχουμε: Μαθητής Β: Οι τιμές της συνάρτησης παραμέ­ x ---+ +cc

x-+W.o

2

= -

Χ_,..,

Ι

Ι

1

χ = -- � -- = -- � Ι = 1 - y 2 2(y- I) 2

y=Ο

Καθηγητής: Τι θα λέγατε τώρα για την τιμή που βρήκαμε;

Μαθητής Α: Επειδή ο αριθμός _ .!._ δεν περιέχεται στο πε2 δίο ορισμού της φ, ο αριθμός Ο δεν θα περιέχεται στο σύνολο τιμών της. Άρα, το σύνολο τιμών της φ είναι το JR - {Ο, 1} . Καθηγητής: Καταλάβατε λοιπόν, γιατί το σημείο α­ ναφοράς μας πρέπει να είναι οι αρχικοί περιορισμοί, έστω και αν, κάποια στιγμή στη διαδικασία επίλυσης λόγω κάποιας aπλοποίησης, δεν είναι όλοι εμφανείς; Με τη σειρά μου να ρωτήσω και εγώ κάτι; Στη διαδι­ κασία προσδιορισμού του συνόλου τιμών της g παρα­ πάνω, απ' όπου τελικά βρίσκουμε και τον τύπο της α­ ντίστροφής, δεν προέκυψε ότι για κάθε y:fl , υπάρχει μοναδικό χ ε JR' ώστε g(x)=y; Μαθητής Γ: Ναι, αυτό βρήκαμε. Καθηγητής: Από αυτό και μόνο δεν συμπεραίνουμε ότι η g αντιστρέφεται; Μαθητής Β: Δηλαδή εννοείται ότι η απόδειξη ότι η g είναι « 1 - 1 » είναι περιττή; Καθηγητής: Ακριβώς! Μετά τη διερεύνηση της εξί­ σωσης f(x)=y, θα μπορούσαμε να διαπιστώσουμε ότι είναι « 1 - 1 » αφού για κάθε y:f 1 η εξίσωση f(χ)=y έχει μοναδική λύση στο JR' την χ = --1- Άρα, η g αντι2(y- 1) στρέφεται και g· 1 : JR- { 1 } �JR με g\x) = -1- , χ oF- 1 » 2(χ - 1) Να πάμε όμως σε επόμενο ερώτημα; Τι λέτε για τη λύ­ ση της εξίσωσης ln ..Γχ. = ef(x) ; Μαθητής Α: Η εξίσωση ορίζεται μόνο για χ>Ο. Για

� � <::::>lnx -� =Ο

τυχαίο χ>Ο, έχουμε: In.Jx = ef{x) <::::> lnx =

Ο αριθμός e είναι μια προφανής ρίζα της. Καθηγητής: Μπορείς σε παρακαλώ να συνεχίσεις; Μαθητής Α: Ναι. Θα αποδείξουμε ότι είναι μοναδική. Θεωρούμε τη συνάρτηση h(x)=lnx- � , χ>Ο. Για κάθε χ

χ 1 , χ2 ε (Ο, +οο) με χ 1 <χ2 έχουμε lnx 1 <lnx2 και e , e I e -Ι >-Ι =>--e < -οποτε -I - <- - => h(x , ) < h(x 2 ) χ,

χ2

χ,

χ2

χ, χ,

χ2 χ2

Άρα η h είναι γνησίως αύξουσα, οπότε ο αριθμός e, εί­ ναι η μοναδική λύση της αντίστοιχης εξίσωσης. Καθηγητής: Μάλιστα. Να δούμε τώρα και ένα όριο;

{

x--++co

}

}

{

__

(η�)2

νουν μη αρνητικές και tείνουν στο μηδέν, όταν χ�+οο . Καθηγητής: Να διατυπώσουμε και αυτό το θέμα; Θ έμα 4 Έστω μια συνάρτηση f : JR' � JR τέτοια ώστε για κάθε χ, y ε JR να ισχύει xf(y)-xyf(x)f(y) � ..!.. . 4

α. Να αποδείξετε ότι f(x)= -1- , χ * Ο . 2χ β. Αν g(χ)=f(χ)+ 1 , να αποδείξετε ότι για κάθε χ ε JR - {1} ισχύει g-' (x) = f(x- 1) . γ. Να λύσετε την εξίσωση ln ..Γχ. =ef(x) δ. Να υπολογίσετε το όριο x�+co lim (χ 2 + 1 )f(ημ 2 χ) . Τελειώνοντας να κάνουμε κάποιες επισημάνσεις σχετικά με τη συμμετοχή σε μια γραπτή δοκιμασία. Βασικά προαπαιτούμενα για μια επιτυχή συμμετοχή σε μια τέτοια δ οκιμασία είναι : • Η πολύ καλή μελέτη της σχετικής θεωρίας από το αντίστοιχο σχολικό βιβλίο. • Η πλήρης κατανόηση της θεωρίας. • Η συστηματική ενασχόληση με το m'tlκείμενο χρησιμοποιώ­ ντας «χαρτί και μολύβι>> Θυμηθείτε: Τα .\Ιαθηματικά δεν είναι σπορ για θεατές.

Επίσης κατά τη διάρκεια της εξέτασης:

Διαβάζουμε προσεκτικά την εκφώνηση, μέχρι την πλήρη κατα­ νόησή της. Ξεκαθαρίζουμε ποια είναι τα δεδομένα και ποια είναι τα ζητούμενα του προβλήματος και, εφόσον απαιτείται, κατασκευάζου­ με προσεκτικά σχήμα που να είναι συμβατό με αυτό το οποίο θέ­ λουμε να απεικονίσουμε. • Αυτά που μπορούμε να επικαλεστούμε κατά την λύση ενός προ­ βλήματος είναι τα δεδομένα και οτιδήποτε προκύπτει από αυτά με βάση τα γνωστά θεωρήματα ή πορίσματα και τους κανόνες συμπερα­ σμού. Δεν αυτοσχεδιάζουμε «κατασκευάζοντας» δικά μας θεωρήμα­ τα, κριτήρια και προτάσεις. • Αν ζητείται ο προσδιορισμός «μαθηματικού αντικειμένου», και δεν διατηρήσουμε ισοδυναμίες, επαληθεύουμε αν αυτό που βρήκαμε ικανοποιεί τις δοσμένες συνθήκες. • Περιγράφουμε αναλυτικά κάθε βήμα της αποδεικτικής διαδικα­ σίας. Τα Ελληνικά είναι μια υπέροχη γλώσσα. Γράφουμε Ελληνικά! ! ! Αποφεύγουμε τις ελλιπείς προτάσεις, τις προτάσεις χωρίς ρήμα, τη γραφή σε συνθηματική γλώσσα. ΒΙΒΛΙΟΓΡΑΦΙΑ 1. Προεδρικό Διάταγμα 60 (30 - 3 - 2006). 2. Α. Κυριακόπουλου: Περί εξισώσεων. 3. Α. Κυριακόπουλου: Μέθοδοι απόδειξης και εύρεσης στα Μαθημα­ τικά. Εmσήμανση λαθών ποu μπορεί να γίνουν σε μια λύση. 4. Γ. Τασσόπουλου: Μαθηματικές εκφράσεις και συμβολισμοί. Γ. Τασσσόπουλου: Διδακτική προσέγγιση δυο θεμάτων Ανάλυ­ 5. σης (Ευκλείδης Β ' τ. 78).

ΕΥΚΛΕΙΔΗΣ Β' 93 τ.l/71


------ Ο Ευκλείδης προτείνει . . . -------

Ε υ κλ ε ί δ η ς π ρ οτ εfνει

, «Η

• • •

καρδιά των μαθηματικών είναι τα προ βλήματ α και οι λύσεις και ο κύριος λόγος ύπαρξης του μαθηματικού είναι να λύνει προβλήματα» .

P. R. HALMOS

Επιμέλεια: Γ. Κ. ΤΡΙΑΝΤΟΣ - Ν. Θ ΑΝΤΩΝΟΠΟΥ Λ ΟΣ - Θ. Α. ΤΖΙΩΤΖΙΟΣ

( ΤΕΥΧΟΥΣ 88 ) τρίγωνο ΑΒΓ με ΑΒ=ΒΓ=ΓΑ ισόπλευρο Δίνεται =2. Χωρίζουμε τη ΒΓ σε τρία ίσα τμήματα ΒΔ= ΔΕ=ΕΓ. Γράφουμε το ημικύκλιο με διάμετρο τη ΒΓ εξωτερικά του τριγώνου ΑΒΓ και προεκτείνουμε την ΑΔ μέχρις ότου τμήσει το ημικύκλιο στο Ζ. Αν Μ είναι το κέντρο του ημικυκλίου τότε να αποδειχθεί ότι η ευθεία ΖΜ διέρχεται από το μέσο Η της ΑΓ. ( Λ ευ τέρης Τσιλιακό ς - Γαλάτσι ) ΛΥΣΗ ι " ( Ευάγγελος. Ο ικονομίδης - Αθήνα) ΑΣΚΗΣΗ 22ι

Α

Επιλέγουμε ορθοκανονικό σύστημα αξόνων με αρχή το μέσον Μ της ΒΓ και άξονα χ'χ τον φορέα της ΒΓ. Η εξίσωση του ημικυκλίου είναι 2 2 χ + y = ι ( 1) με y ::::; Ο . Το ύψος ΑΜ έχει μήκος

.J3 οπότε A(O,.J3) ενώ Δ(- .!. ,0) . Ο συντελεστής 3 - Α διεύθυνσης της ΑΔ είναι λ = ΥΔ Υ = 3.J3 και η

εξίσωσή

της:

y - .J3 = 3.J3χ ή

ΧΔ - ΧΑ

= λ(χ - χ Α ) δηλαδή y = 3.J3x + .J3 (2). Οι Υ - ΥΑ

συντεταγμένες του Ζ προκύπτουν από την λύση .J3 του συστήματος των ( 1 ),(2 ) . Είναι Ζ(_ .!_ , - )

2 2 Η εξίσωση της ΖΜ είναι y = .f3x και

επαληθεύεται από τις συντεταγμένες του μέσου 1 .J3 Η (2 , 2) της ΑΓ. ΛΥΣΗ 3 8 ( Θωμάς Τσάκ ας - Πάτρα) λ

Έστω η τομή της προέκτασης της ΑΔ με την εκ του Μ παράλληλο προς την ΑΒ. Από το θ. Θαλή ΜΔ 1 ΜΖ1 ΜΔ 1 ) . Ό , εχουμε ΑΒ = ΔΒ ( μως, ΔΒ = 2 (2 ) . Απο, ΜΖ1 .!_ ΜΖ, .!_ ΑΒ ή = = ή ( 1 ) ,(2 ) έπεται ότι

2

ΑΒ 2

1 ΜΖ, = 2 ΒΓ = 1 , που είναι η ακτίνα του

ημικυκλίου. Άρα το σημείο Ζ1 ταυτίζεται με το Ζ. Επειδή η ΖΜ είναι παράλληλη στην ΑΒ και διέρχεται από το μέσον Μ της ΒΓ διέρχεται και από το μέσον Η της ΑΓ. ΛΥΣΗ 2 8 ( Γ εώργιος Τσιώλης - Τ ρίπολη ) J

Α(Ο;/))

I

I

I

I

I

I

Λ

Λ

Για να είναι ΖΜ I IΑΒ , αρκεί Α, = Ζ . Από το Πυθαγόρειο θεώρημα στο τρίγωνο ΜΑΔ έχουμε:

:

χ

ΑΔ 2 = ΑΜ2 + ΔΜ2 � ΑΔ 2 = ( 2 ) 2 + (�) 2 = .J7 = 3 + ! = 2 8 �.λΔ = 2 (1) 3 9 9 /

Από τον νόμο των συνημιτόνων στο τρίγωνο ΑΒΔ ΒΔ 2 = ΑΒ 2 + ΑΔ 2 - 2ΑΒ · ΑΔ · συν Αι � 2 .J7 ν Αι � 2 8 - 2 · 2 · --συ -4 = 4 + 9 3 9 Λ

Λ

ΕΥΚΛΕΙΔΗΣ Β' 93 τ.l/72


-------

Ο

Ευκλείδης προτείνει . .

Λ Λ 60 gJ7 5J7 -- συν Α ι = - � συν Αι = -- (2) 3 9 14 Από τον νόμο των ημιτόνων στο τρίγωνο ΑΖΜ:

ΑΜ

η μΖ

=

ΖΜ

η μ Α2

και συνεπως, ' η

ΑΜ

=

η μΖ

ΖΜ

:

J3

= 2J7

η μΖ

Λ

(3). Από (2),(3) έπεται ότι Α ι = Ζ Λύσεις έστειλαν: Γιώργος Δελη στάθ η ς - κάτω Πατήσια, Ροδ όλφος Μπό ρ η ς - Δάφνη, Αντώνης Ιω αννίδη ς - Λάρισα, Φίλιππος Σερέφογ λ ου Μελίσσια, Στέλιος Πετρο λέκας Δραπετσώνα, Διονύση ς Γιάννα ρος - Πύργος. ΑΣΚΗΣΗ 222 ( ΤΕΥΧΟΥΣ 88 ) Θεωρούμε ημικύκλιο Cι διαμέτρου ΒΓ και στο

-

-

εσωτερικό του ημικύκλιο C 2 με μικρότερη ακτίνα που εφάπτεται του Cι στο Β. Χορδή ΓΑ του Cι

εφάπτεται του C 2 στο Ε. Ημιευθεία που άγεται από το Γ τέμνει την ΑΒ στο Λ και το τόξο ΑΒ του Cι στο Μ. Αν ισχύουν: ΒΓ=α , ΑΓ=β , ΑΒ=γ και α + β + γ = k (cm) , (ABΓ) = k(cm 2 ) και ακόμη ότι ΓΛ · ΓΜ · ΑΒ = k 2 - 4k (cm3 ) , τότε :

1 ) Να δειχθεί ότι το μήκος του τμήματος ΑΘ, όπου Θ η τομή της ΒΕ με την ΓΛ, είναι ανεξάρτητο από τις τιμές που μπορεί να πάρει ο αριθμός k. 2) Να βρεθεί η ελάχιστη τιμή του k και να υπολογισθούν τα μήκη των ακτίνων των Cι , C 2 .

(Γιώργος Μήτσιος - Ράμια Άρτας)

ΛΥΣΗ (Από τον ίδιο)

1 ) Αποδεικνύουμε αρχικά ότι οι ΒΕ , ΓΛ είναι διχοτόμοι των γωνιών του τριγώνου ΑΒΓ. Έστω Βχ η κοινή εφαπτομένη των Cι , C 2 . Τότε Λ

Λ

ΑΒχ = ΑΓΒ (

χορδή Λ

εφαπτομένη Λ

εγγεγραμμένη στο Cι ) και χΒΕ = ΑΕΒ ( αφού ΑΕ

Λ

Λ

Λ

Λ

Λ

Λ

Λ

ΑΕΒ = ΕΓΒ+ ΓΒΕ ως εξωτερική γωνία του τριγώνου ΕΒΓ) δηλ. ΒΕ είναι διχοτόμος της γωνίας Β του τριγώνου ΑΒΓ. Από το ορθογώνιο Λ τρίγωνο ΑΒΓ ( Α = 90 ° , α 2 = β 2 + γ 2 ) και τις υποθέσεις του προβλήματος, έχουμε ότι:

21 sJ7 1 75 συν 2 Ζ = 1 - η μ 2 Ζ = 1 - - = - � συνΖ = -14 1 96 1 96

έχουμε:

Λ

ΑΒΕ = ΕΒχ- ΑΒχ = ΑΕΒ- ΑΓΒ = ΓΒΕ ( αφού

J2ϊ . Επειδη' μΖ = 14" Λ

-------

Λ

-

και Βχ είναι εφαπτόμενες στα άκρα της χορδής ΒΕ του ημικυκλίου C 2 ). Κατά συνέπεια, έχουμε ότι:

k = (ΑΒΓ) = .!.. βγ και β + γ = k - α οπότε 2 2 (β + γ) = (k α) 2 <::::> β 2 + γ 2 + 2βγ = k 2 + α 2 -2αk � α 2 + 4k = k 2 + α 2 - 2αk � _

2αk = k 2 - 4k <::::> 2αk = k(k - 4) � k - 4 = 2α Ακόμη, ΓΛ · ΓΜ · ΑΒ = k 2 - 4k = k(k - 4) �

1 ΓΛ · ΓΜ · γ = - βγ · 2α � ΓΛ · ΓΜ = αβ � 2 Λ Λ β ΓΜ - = - � συν Γι = συν Γ 2 ( από τα ορθογώνια . ΓΑ α Λ

Λ

τρίγωνα ΓΑΛ και ΓΒΜ ) οπότε Γι = Γ 2 και η ΓΛ είναι διχοτόμος της γωνίας Γ του τριγώνου ΑΒΓ με αποτέλεσμα το Θ να είναι το έκκεντρο του τριγώνου ΑΒΓ και η ΑΝ διχοτόμος της γωνίας Α. Φέρουμε ΝΡ .l ΑΒ . Το τρίγωνο Α ΡΝ είναι

ορθογώνιο και ισοσκελές, οπότε ΑΝ = AP.fi . Επειδή ΡΝ I I ΑΓ από το θεώρημα του Θαλή αβ ΓΝ ΑΡ β + γ ΑΡ , β-ι1εχουμε: - = - � - = -- � ΑΡ = β+y α ΑΒ ΒΓ γ βγ.,fi

β .,fi � ΑΝ = δα = y . β+γ β+γ Στο τρίγωνο ΓΑΝ η ΓΘ είναι διχοτόμος της γωνίας Γ, οπότε από το θεώρημα της διχοτόμου έχουμε: � AP.fi =

ΑΘ_ β _β_+_γ => __ αβ ΑΘ + ΘΝ α β+γ β + γ => ΑΘ β + γ => ΑΘ = ΑΝ · β + γ α+β+γ α + β + γ ΑΝ α + β + γ β + γ β γ h_ · β + γ βγh_ => ΑΘ δ α+β+γ β+γ α+β+γ α+β+γ 2kh ,-;:; => ΑΘ => ΑΘ 2ν2 Α_ Θ _ ΘΝ

=

Θ Α_ _ ΑΓ _ => _ ΓΝ ΘΝ

= __ =

=

=

=

α

=

·

=

k

--

2) Έχουμε k = 2α + 4 και β + γ = k - α . Άρα, β+γ=kγ=

2k β

(2). Από

2k

β+ β

k-4

-2-

=

k+4 2

ΕΥΚΛΕΙΔΗΣ Β' 93 τ.l/73

=

k+4

-2-

τις

<=>

, ( l ). Απο

ι

k = iβγ �

( 1 ) , (2) παίρνουμε:

2( β2 + 2k) (k + 4)β <=> =


---- Ο

Ευκλείδης προτείνει . .

2β 2 - (k + 4)β + 4k = 0 (3) Η (3) έχει πραγματική λύση όταν Δ � Ο , δηλ. όταν (k + 4) 2 - 32k � Ο <=> k 2 - 24k - ι 6 � Ο και δέχεται

k � 1 2 + sJ2 και επειδή είναι k � 1 2 + 8J2 . Για την ελάχιστη τιμή

λύσεις k>4

k :::; 1 2 - sJ2

ή

του k = 1 2 + 8J2 η (3) δίνει β=

k+4

=

ι 6 + 8J2

= 4 + 2 Ji

r;

γ.

Τότε

το

4 4 τρίγωνο ΑΒΓ είναι ισοσκελές με την διχοτόμο ΑΝ να είναι και ύψος, οπότε το Ν είναι το κέντρο του ημικυκλίου C 1 και συνεπώς η ακτίνα του είναι R = ΑΝ = 2 + 2Ji . Η κάθετη επί την ΑΓ στο σημείο Ε τέμνει την ΒΓ σε σημείο Κ που είναι το Λ

Γιώ ργος Τσιώλη ς - Τρίπολη, Δάφνη. ΠΡ ΟΤΕΙΝΟ ΜΕΝΑ ΘΕΜΑ Τ Α 244 . Να λυθεί το ( μη γραμμικό ) σύστημα: Χ ι + Χ2 + . . . + x n = ξ , όπου ,...-.-α 1 - χ� + α 2 - χ; + . . . + α n - χ� = θ

Λύσεις έστειλαν οι:

Ροδόλφος Μπόρη ς -

{�

=

τετράγωνα τέτοια, ώστε: Ο :::; xi :::;

ΒΚ = ΒΓ - ΓΚ ή ρ = 2R - ρ.Ji ή ρ =

2R ι;:;- = 1 + ν2

= 2R( Ji - ι) = 2(2 + 2 Ji)(Ji - ι) = 4 . Λύσεις έστειλαν οι: Διονύση ς Γιάνναρος ­ Πύργος, Θ ω μάς Τσάκας - Πάτρα, Αντώνης Ι ω ανvίδη ς - Λάρισα , Ροδόλφος Μπόρης - Δάφνη. ΑΣ ΚΗΣ Η 223 ( ΤΕΥΧΟΥΣ 89 ) Τρείς φίλοι είχαν κάποια χρονική στιγμή από ένα χρηματικό ποσόν ο καθένας. Συμφώνησαν καθένας από αυτούς, κατά σειράν, να διπλασιάσει τα χρήματα των δύο άλλων. Στο τέλος βρέθηκαν με ι 600 ευρώ ο καθένας. Να βρεθεί τι ποσόν διέθετε καθένας τους στην αρχή. ( Η ρακλή ς Ευ αγγελινός - Αθήνα ) ΛΥΣΗ ( Δ ιονύση ς Γιάνναρος - Πύργος ) Έστω α , β , γ οι τρείς φίλοι με αντίστοιχα αρχικά ποσά χ , y , z αντιστοίχως. Κατά τη συμφωνία: Ο ά διπλασιάζει τα χρήματα των δύο άλλων οπότε μετά από αυτό έχουν, ο β' :2y , ο γ ' : 2z ,ο α': x-y-z. Ο β ' διπλασιάζει τα χρήματα των δύο άλλων οπότε έχουν, Ο α ' : 2(x-y-z), ο γ ' : 4z, ο β ' : 2y-2z--(x-y-z)=-x+3y-z. Με τη σειρά του ο γ ' διπλασιάζει τα χρήματα των δύο άλλων οπότε έχουν, ο α ' :4(x-y-z), ο β ' : 2(-x+ 3y-z)=-2x+ 6y-2z, ο γ ' : 4z-2(x-y-z)--(-x+3y-z)=-x-y+7z. Επομένως, καταλήγουμε στο σύστημα: 4(x - y - z) = l 600

{

2(-χ + 3y - z) = 1 600 με λύση :

rα:

(x, y, z) = (2600 , 1 400 , 800 )

που ζητούμενα αρχικά ποσά του καθενός.

) 245 . Δίνεται ότι το ( ως προς x, y, z ) σύστημα χ+y+z=α+β+γ 2 2 2 αχ + βy + γz = α + β + γ (Σ) 2 2 β χ + γy + αz = α 2 + β + γ γχ + αy + βz = 4αβ

δέχεται τουλάχιστον μία λύση. Να βρεθεί η απαλείφουσα σχέση του συστήματος και όλες οι λύσεις του. (Ηρακλή ς Ευαγγελινός - Γλυφάδα ) 246. Σε τρίγωνο ΑΒΓ είναι: υ α = μΡ = δ1 , όπου υα

το ύψος που αντιστοιχεί στη πλευρά α, μβ η

διάμεσος που αντιστοιχεί στην πλευρά β και δ1 η εσωτερική διχοτόμος της γωνίας Γ. Να δειχθεί ότι το τρίγωνο ΑΒΓ είναι ισόπλευρο. (Ευάγγελος Οικονο μίδη ς - Κάτω Πατήσια ) 247. Ένας μαχαραγιάς μοίρασε όλα τα μαργαριτάρια του στις κόρες του ως εξής:

Η ι Η κόρη

και το

..!_ ν

τα

Η

..!_

των ν 2Η κόρη παίρνει 2k μαργαριτάρια

παίρνει k μαργαριτάρια και το

υπολοίπων.

των υπολοίπων.

Η

3Η κόρη παίρνει 3k

ι , , μαργαριταρια και το - των υπολοιπων κ.ο.κ. ν Δεδομένου ότι όλες οι κόρες πήραν τον ίδιο αριθμό μαργαριταριών να βρεθούν συναρτήσει των k και ν: Ι) Ο συνολικός αριθμός των μαργαριταριών. 2) Ο αριθμός των μαργαριταριών που πήρε κάθε μία από τις κόρες. 3) Πόσες ήταν οι κόρες του μαχαραγιά. (Η ρα κλή ς Ευ αγγελινός - Γλυφάδα ). την Ομάδα Λυκείου

λύσης

προβλημάτων

του

Ψηλογέφυρο Αγρινίο«Παναγ υ, ία Προυσιώτισσα>> Ιδιωτικού

είναι

Γα; με

(Π αναγιώτη ς Κουμάντος - Αθήνα

Από

-χ - y + 7z = 1 600

είναι τέλεια

i = ι, 2, 3, . . . , η που ικανοποιούν τη συνθήκη : 2 2 + Γα: + . . . + .Jα: = ξ + θ .

Λ

ρ.Ji , οπότε είναι

jξj + jθj ::f:. Ο , οι θετικοί αριθμοί

κέντρο του C 2 . Επειδή Β = Γ = 45 ° είναι και με ΓΚ

-----

λάβαμε ασκήσεων: 215, 216, 220, 221, 223. Η

στήλη

τις

λύσεις

εύχεται σε όλους καλή δύναμη για τη νέα

Σ zολική χρονιά που μόλις ξεκινάει. ! ! !

Β ' 93

των

ΕΥΚΛΕΙΔΗΣ τ.l/74


Ένα μ ι κρό σχόλ ιο με αφορμή μ ι α σημαντ ι κή παρουσία Γ.Σ.

Τασσόπουλος

Στο τρίτομο έργο «ΟΥΣΙΩΔΗ ΜΑΘΗΜΑτΙΚΆ» των εκλεκτών συναδέλφων Μανώλη Γ. Μαρα'γ1Cάιcη και Μιχάλη Ν. και κατά την θα αποβεί μνημειώδες για τα Ελληνικά Μαθηματικά χρονικά, διάβασα μεταξύ άλλων φιλοσοφημένων αποφθεyμάτων το που περιέχονται στην εισαγωγή του με τίτλο: εξής ενδιαφέρον, που ίσως ανεπαίσθητα έχει περάσει από το μυαλό μας. Όταν ο Bruce Winstein ρώτησε τον Ricbard Feynman <<Αν μπορούσατε να κάνετε κάτι διαφορετικό στη σταδιοδρομία σας, τι θα ήταν αυτό;» η απίστευτη αυτή περίπτωση χαρισματικού ανθρώπου απάντησε χωρίς κανένα δισταγμό ως εξής: «θα προσπαθούσα να ξεχάσω πως είχα λύσει ένα πρόβλημα. Έτσι κάθε φορά που θα εμφανιζόταν το ίδιο πρόβλημα, ίσως το έλυνα με διαφορετικό τρόπο- δεν θα σκεφτόμουν πώς το είχα λύσει προηγουμένως. Ήθελα να ξεχνώ τις λύσεις, Η μεγαλύτερη χαρά μου είναι να σκέφτομαι με νέο τρόπο τα πράγματα . . . ».

ταπεινή μου -yνώμη

«Αγαθόν το εξομολΟ'yείσθαι»

• Ένα πολύ -yνωστό πρόβλημα που ήρθε τότε στο μυαλό μου είναι η εύρεση του πλήθους των διαγωνίων κυρτού ν-γωνου. Τα περισσότερα βιβλία Γεωμετρίας επαναλαμβάνουν τη -yνωστή διαδικασία, σύμφωνα με την οποία κάθε κορυφή πρέπει να συνδεθεί με όλες τις άλλες, εκτός από την ίδια και τις δυο προσκείμενες, δηλαδή με (ν-3 ) κορυφές για να δώσει διαγώνιο, (κάτι που όντως δυσκολεύει τους μαθητές) ή καταφεύγουν στη Μαθηματική επαγωγή. Απαλλασσόμενοι όμως από αυτές τις διαδικασίες θα μπορούσαμε πολύ απλά να διαπιστώσουμε αρχικά ν (ν - 1) τμήματα ότι με άκρο κάθε ένα από τα σημεία Α1 , Α2, , Αν ορίζονται (ν- 1 ) τμήματα δηλαδή συνολικά

2

• • •

(

)

(αφού π.χ. το Α1Α2 λαμβάνεται και ως Α2Α1) Από αυτά τα ν- τμήματα είναι πλευρές. Οι διαγώνιες λοιπόν θα είναι ν (ν - 1) ν (ν - 3 ) ν-1 ν=ν τα υπόλοιπα τμήματα. -1 = •

2

2

2

Ένα άλλο θέμα που θέλησα να αντιμετωπίσω διαφορετικά με αφορμή αυτή την προτροπή του

R.

Feynman

είναι το εξής: Ως -yνωστόν, αν τα σημεία Β, Γ βρίσκονται προς το ίδιο μέρος μιας ευθείας (ε) , τότε από τα σημεία Α της (ε) αυτό που καθιστά τη διαφορά

IAB - AΓI

μέγιστη είναι το Α 1 (σημείο τομής της (ε) με τη ΒΓ) .

Β Ε Ρ Ρ Ο LEV I

Π Ρ Ο Λ Ο ΓΟ Σ - Ε Π Ι Μ ΕΛΕ Ι Α

Από μαθητής θυμάμαι ότι ο μόνος που είχε ασχοληθεί με

την περίπτωση ΒΓ I /(ε) και είχε αποδείξει ότι τότε δεν

υπάρχει μέγιστη τιμή της διαφοράς .. ήταν ο αείμνηστος Σπύρος

Κανέλλος.

Φωτεινό

παράδειγμα

μεγάλου

δάσκαλου και ερευνητή που μας άφησε παρακαταθήκη ένα

πληρέστατο βιβλίο Ευκλείδειας Γεωμετρίας. Στη λύση του, που είχαμε δημοσιεύσει σε παλαιότερο τεύχος του Ευκλείδη

Β ·, θεωρούσε, στα παρακάτω σχήματα, τα συμμετρικά των Α , Α1 ως προς τη μεσοκάθετο (δ) του ΒΓ, κάτι για το

οποίο δεν έβρισκα να συντρέχει ιδιαίτερος λfyyoς. Σκέφτηκα λοιπόν, μήπως θα μπορούσαμε να δώσουμε λύση χωρίς

ΕΥΚΛΕΙΔΗΣ Β' 93 τ.l/75

ΤΕΥΚΡΟΣ Μ Ι ΧΑΗΛΙΔΗΣ


αυτά τα συμμετρικά. Πράγματι, προέιαιψε η ακόλουθη πιο σ6νtομη λύση.

Θεωρο6με τα σημεία Α , Α1 στο ημιεπίπεδο ((δ) , Β) . Αρκεί λοιπόν να δείξουμε ότι ΑΝ < Α1 Ν => ΑΓ - ΑΒ < Α1 Γ - Α1 Β , δηλαδή β - γ < β 1 - γ1 • Επειδή από σχέσεις της μορφής

β<λ γ<μ

}

mθανόν να προκύψει

σχέση της μορφής β --' γ < λ - μ ή της μορφής β - γ � λ - μ , γι αυτό θα αναγάγουμε τη διαφορά β - γ στο

άθροισμα β + γ μέσω του δευτέρου θεωρήματος διαμέσων. Έχουμε λοιπόν:

β2 - γ2 ΜΗ β+γ

<

=

2 α · ΜΗ => β - γ

ΜΗ ι β . + γι

ΜΗ ΜΗ ι

<

=

2α · ΜΗ

β+γ

β+γ β . + γι

, . 0 μοιως, β 1

-

χ

β+γ

x+y

β . + γι

, ή __ <

γ1 =

2α ΜΗ 1 ·

. Αρκει

,

, , επομενως να δείξουμε οτι:

� +�

. Για να απαλλαγο6με από τα β 1 , γ1 παρατηρο6με ότι :

1 1 β+γ β+γ > β 1 < β + y , γ1 < γ + y => β 1 + γ 1 < β + γ + 2 y => -- > => , οπότε αρκεί να β . + γι β + γ + 2y β . + γι β + γ + 2y

δείξουμε ότι : __ < •

Αν το

Αν το

Η Η

χ

β+γ

x+y

β + γ + 2y

, ή β χ + β y + 2 xy < β χ + γχ + β y + γy , ή 2χ < β + γ .

είναι σημείο του τμήματος ΒΜ (σχήμα α ) ,τότε προφανώς χ :::;;

, εκτος , του ΒΜ (σχημα , β), τοτε , : ειναι

Σε πόσα άλλα άραγε θέματα που χωρίς ιδιαίτερη σκέψη επαναλαμβάνουμε μηχανικά να διδάσκουμε, δεν θα ήταν δυνατή μια νέα τοποθέτηση ! ! ! Αναλογιστείτε λοιπόν τι τεράστιο πεδίο έρευνας μας ανοίγει αυτή η σοφή φράση του μεγάλου δασκάλου. Τελειώνουμε με τα υπέροχα λόγια που έχουν του 1° του εmλεγεί «στο Μαθηματικο6 περιοδικο6

Οδυσσέα Ελύτη Τεύχος Θεαίτητος»

ΗΜ

=

,..;

Τόv κα ιρό

ΗΒ + ΗΓ 2 δέν

<

ιωιαλάιkιινο ιό

αυJ.λοy ιοι ιιtός _.,.,ιαμιiς.

vό ΙΙΙΜiμιι μιδαμιvή όnόσrC118J nού έν ιι.ιΙΙσ vό καί μέ .ιiφο. ιόν σιίιόν

αιίοι

rήν

<8ιέρανιη

2

ΑΒ + ΑΓ

,_...,. .ώ. llψίίμιι ι . μσίί Αiyαιιι οι ι δέν εΙιrσ fqlά νό μεισισnισβϋ καιό Ενα θiμι.

διαrρέ�

α

y ιό

ο:α ί

2

<

β+γ 2

=> 2χ < β + γ .

γ+β => χ < -- => 2 χ < β + γ 2 θ Ε Α ΙΤ Η Τ Ο Σ

πψοiiιι..-iJ t.ι.:ώση

μιιθι:Jμαru.:ιiιι· mtσυδι.ίτ.ι•

ίιιf

ιίικ1ιu ιέ.., ι: μιJu; σά η,..., ιφήσι.., wά ιό όνι ι σι� σιίιό οiμρσ; κσί άπό rήw' -.ιρ ίζε ι

μεριά rήv διι:ή μιιι; ..,; � οιά

πιι ιδιό Οι ι μιό διαιορε r ιιι:ή άιιιί μΕρσυς

,..ana- ιύιηιοι;

ιουι;

8ά ιn:ιροijαε nιiA ι νό ιό

δ ισu: ίpι«ι riίtt οrο ιχε ίυw ιίjι;

...

...t:.+t2.

11"ι..:" '1� Ι>•rι.ιi,>Ι-.ής

III A!'to'0.'\:1-f! ι· M . f'AI '.:

Ι!νόζε ι αέ όu.ιiiς

ιιύιιrηρά ι:οί όu.ιiiς dnιδε ιι:: ιέο μιιιιη.ιι ιώ;

Μια παράλειψη και ένας πλεονασμός Στο άρθρο του αγαπητού φίλου, Σχολικού Συμβούλου Πρόδρομου Ελευθερίου στο τεύχος 92 σελίδα 60απο δικιά μας αβλεψία;, παραλήφθηκε η συνθήκη f(O)=O, προκειμένου να ισχύει και η αντιστροφή της συνεπαγωγής: (f περιττή => f άρτια). Συγκεκριμένα ο συνάδελφος είχε γράψει: (f άρτια και f(O) => f περιττή), το οποίο και είχε αποδείξει. Εξάλλου στη σελίδα 62 αντί του: f t [ β] και μη σταθερά καλό είναι να γραφεί: f t ( α, β] και επομένως μη σταθερά Φtλικότατα Γεώργιος Σ. Τασσόπουλος Λ

α,

Λ

ΕΥΚΛΕΙΔΙΙΣ τ.t/76 Β' 93


Γ ρ ίφοι Ο 4ψ ήφ ιος

«Η δικαιοσύνη εlναι τετράγωνη,

«Η παιδεία εlναι σαν την πλούσια χώρα,

ι

Ανεβαίνοντας από τη βάση του σταυρού προς τα πάνω ή πάνω και δεξιά ή πάνω και αριστερά μετράμε 7 σπίρτα. Μπορείτε να κάνετε σταυρό με 2 σπίρτα λιγότερα χωρίς να μεταβληθεί το προηγούμενο άθροισμα;

Μ ά ντ ε ψ ε Μάντεψε το σύνολο των πόντων 3,4 ή 5 χαρτιών

μιας τράπουλας. Ζήτησε από κάποιον να πάρει 3,4 ή 5 χαρτιά στην τύχη από μια τράπουλα, σε κάθε ένα να προσθέσει τόσα φύλλα από την τράπουλα ώστε η τιμή που αναγράφει το κάθε φύλλο (ντάμα, ρήγα και βαλέ= 1 Ο) και τα φύλλα που θα προσθέσει να κάνουν 1 0. Σας ανακοινώνει πόσα φύλλα πήρε στην αρχή και πόσα είναι τα φύλλα που περίσσεψαν από την τράπουλα και λέτε το άθροισμα των πόντων των 3,4 ή 5 φύλλων που πήρε στην τύχη. Πώς γίνεται αυτό; Ο ι σ τ ά μνες

Μας έδωσαν δύο ίδιες στάμνες. Η μία περιέχει 50 λευκά μπαλάκια και η άλλη 50 μαύρα. Θα τραβήξουμε ένα μπαλάκι στα τυφλά από μία τυχαία στάμνα και εάν είναι λευκό κερδίζουμε το παιχνίδι, ενώ εάν είναι μαύρο χάνουμε. Έχουμε δικαίωμα από πριν να μετακινήσουμε όσα μπαλάκια θέλουμε από τη μία στάμνα στην άλλη, αφήνοντας όμως μέσα στις δύο στάμνες και τα 1 00 μπαλάκια. Πως θα κατανείμουμε τα μπαλάκια ώστε να μεγιστοποιήσουμε την πιθανότητα να τραβήξουμε λευκό μπαλάκι; :

••

σ' όλα τα μέρη lση και όμοια» Πυθαγόρας

παράγει όλα τα αγαθά>> Σωκράτης

Ο αριθμός διαιρείται με 13 όταν: πάρουμε τον αριθμό χωρίς το τελευταίο του ψηφίο και του προσθέσουμε τέσσερις φορές το τελευταίο ψηφίο, το αποτέλεσμα να διαιρείται με το 1 3 . Εφαρμόστε τον κανόνα αυτό συνέχεια μέχρι να έχετε διψήφιο αριθμό που εύκολα φαίνεται αν είναι πολλαπλά­ σιο του 1 3 . (όπως: 2466 1 , 2466+4. 1 =2470, 247+4.0=247, 24+4.7=24+28=52 που είναι 52=4. 1 3 άρα ο 2466 1 διαιρείται με 1 3 είναι 2466 1 = 1 897. 1 3). • Ο αριθμός διαιρείται με 17 όταν: πάρουμε τον αριθμό χωρίς το τελευταίο του ψηφίο και του αφαιρέσουμε πέντε φορές το τελευταίο ψηφίο, το αποτέλεσμα να διαιρείται με το 1 7. Εφαρμόστε τον κανόνα αυτό συνέχεια μέχρι να έχετε διψήφιο α­ ριθμό που εύκολα φαίνεται αν είναι πολλαπλάσιο του 1 7. (όπως: 5372, 53 7-5 .2=527, 52-5 .7=523 5= 1 7, άρα ο 5372 διαιρείται με 17 είναι 5372=3 1 6. 1 7). • Ο αριθμός διαιρείται με 19 όταν: πάρουμε τον αριθμό χωρίς το τελευταίο του ψηφίο και του προσθέσουμε δύο φορές το τελευταίο ψηφίο, το αποτέλεσμα να διαιρείται με το 1 9. Εφαρμόστε τον κανόνα αυτό συνέχεια μέχρι να έχετε διψήφιο α­ ριθμό που εύκολα φαίνεται αν είναι πολλαπλάσιο του 1 9. (όπως: 73568, 7356+2.8=7356+ 1 6=7372, 737+2.2=737+4=74 1 , 74+2. 1 =76 που είναι 4. 1 9 άρα ο 73568 διαιρείται με 19 είναι 73568=3872. 1 9). Η γνωστή φράση του Αρχιμήδη •:• «Δως μοι πα στω και τα Γαν κινάσω» (Δώσε μου μέρος να σταθώ και θα μετακινήσ ω τη Γη), αν και ακούγεται απίθανο κάτι τέτοιο, όμως μελέτησαν κάποιοι το θέμα και πιστεύουν ότι ο Αρχιμήδης ίσως κάτι να είχε στο μυαλό του. Όμως με ένα μοχλό πρώτου (Βάρος­ είδους Υπομόχλιο-Δύναμη) θέλουμε μήκος μοχλού 88. 1 021 μέτρων! (5 . 1 0 10 είναι η απόσταση Γης­ Ήλιου). •:• Αφού ως μονάδα μέτρησης των επιφανειών όρισαν το τετράγωνο, αμέσως μπήκε το ερώτημα του τετραγωνισμού του κύκλου. Ο πρώτος που •

Ένας 4ψήφιος είναι τέλειο τετράγωνο και τα 2 πρώτα ψηφία του καθώς και τα 2 τελευταία του είναι ίδια. Ποιος είναι ο αριθμός; Σταυ ρός

Παναγιώτης Π. Χριστόπουλος

Ξέ ρ ετε ότι: Κριτήρια διαιρετότητας με 13, 17, 19

ΕΥΚΛΕΙΔΗΣ Β' 93

τ. l / 7 7


-------

Τα Μαθηματικά μας Διασκεδάζουν

ασχολήθηκε με τον τετραγωνισμό του κύκλου είναι ο Αναξαγόρας ο Κλαζομένιος (500-428 π.Χ), φίλος και δάσκαλος του τετραγώνισε που Περικλή. Ο πρώτος μεικτόγραμμες επιφάνειες είναι ο Ιπποκράτης ο Χίος (470-400 π.Χ). ·:· Στα τέλη του 5ου π. Χ. αιώνα το πρόβλημα του τετραγωνισμού του κύκλου ήταν πολύ δημο­ φιλές. Ακόμα και ο κωμικός ποιητής Αριστοφάνης έκανε ένα αστείο σχετικά με αυτό στις Όρνιθες. Φέρνει στη σκηνή τον αστρονόμο Μέτωνα, ο ο­ ποίος λέει: <<με το ορθό ραβδί αρχίζω να μετρώ

ώστε να γίνει ο κύκλος τετράγωνος για χάρη σου και στο κέντρο του θα είναι η αγορά στην ο­ ποία θα οδηγούν όλοι οι δρόμοι, όπως σ ' ένα αστέ­ ρι, που ενώ είναι κυκλοτερές στέλνει παντού ευθείες ακτίνες λαμπερές». Ο αρχηγός των Ορνίθων Πει­ σθέταιρος τον χλευάζει «Αλήθεια, ο άνθρωπος εί­ ναι Θαλής!», και οδηγεί εκτός σκηνής τον Μέτωνα

κακήν κακώς. : Στο πανεπιστήμιο του Γκέτινγκεν το 1 853, ο Gauss ζήτησε από τον φοιτητή του G. Riemann να ετοιμάσει και να παρουσιάσει μια διατριβή επί υφη­ γεσία πάνω στα θεμέλια της Γεωμετρίας. Τον επόμενο χρόνο ο Riemann έδωσε τη διάλεξή του, παρά το φόβο και τη δειλία που είχε να μι­ λά δημόσια, τίτλος της ήταν • •

ϋber die Hypothesen welche der Geometrίe zu Grunde liegen («Επί των υποθέσεων που βρίσκονται στα θε­

μέλια της Γεωμετρίας»). Ανέπτυξε δηλαδή τη θεωρία

του για τη Ριμάνεια Γεωμετρία, όπως λέyεται σήμε­ ρα. Βρήκε δηλαδή τον σωστό τρόπο να επεκτείνει σε «V)) διαστάσεις τη Διαφορική Γεωμετρία των επι­ φανειών με τον Τανυστή καμπυλότητας. Σε μια επι­ φάνεια, ο τανυστής αυτός μπορεί να αναχθεί σε έναν αριθμό, θετικό, αρνητικό ή μηδέν. Ο Riemann μας οδήγησε έτσι στα μοντέλα των γνωστών μη Ευκλεί­ δειων γεωμετριών. Η ιδέα του Riemann ήταν να ει­ σαγάγει ένα σύνολο αριθμών για κάθε σημείο του χώρου που θα περιγράφουν το πόσο καμπυλωμένος είναι. Βρήκε ότι χρειάζονται 1 Ο αριθμοί σε κάθε ση­ μείο για την πλήρη περιγραφή των ιδιοτήτων μιας πολλαπλότητας, όσο παραμορφωμένη και να είναι. :

• •

Η

Ε κτυ π ωτής αν τι κε ι μ έν ω ν ,

3-διάστατη εκτύπωση είναι πλέον γεγονός. Κατασκεύασαν εκτυπωτή που μπορεί να εκτυπώνει αντικείμενα. π.χ. να αντιγράφει ένα εργαλείο ή ένα εξάρτημα μηχανής. Ο άνθρωπος που επινόησε τον εκτυπωτή είναι ο Charles W. Hull και τον ονόμασε Stereolithography, δίπλωμα ευρεσιτεχνίας το 1 984. Η 3-διάστατη εκτύπωση μπορεί να έχει βαθιά επίδραση στη ζωή μας, όπως

-------­

ο ερχομός της aτμομηχανής, του τυπογραφείου, του εργοστασίου, του τρανζίστορ κ.α. Με 3D εκτυπωτές θα μπορούμε να φτιάξουμε ανθρώπινα όργανα, όπλα, κοσμήματα, παπούτσια, ακόμη και οι αστροναύτες με ένα εκτυπωτή στο διάστημα θα φτιάχνουν ότι τους χρειάζεται.

Μάλιστα στο Διεθνή Διαστημικό Σταθμό (ISS) έχουν την ISSpresso μηχανή καφέ από γνωστή ιταλική βιομηχανία καφέ, ο καφές όμως δεν σερβίρεται σε φλιτζάνι αλλά σε διαφανή σακουλάκια. Για τις επισκευές του έχουν 3διάστατο εκτυπωτή. Ευχαριστούμε τους Νiκο Ταπεινό, Αποστόλη Κακαpά, Ιπ­ ποκράτη Δάκοyλου πολιτικό Μηχανικό και Αρ'yύρη Καvτε­ μiρη όπως επiσης και όλους όσους στηρίζουν τη στήλη αυτή. Ο ι απαντήσεις στα μαθηματικά μας διασκεδάζουν

Γρίφ οι Ο 4ψήφιος Αν

ο αριθμός είναι :ΧΧψψ=λ2 τότε ι ΟΟΟΧ+ ι ΟΟΧ+ ι ΟΨ+Ψ=ι ι ΟΟΧ+ l ι Ψ= ι ι ( Ι ΟΟΧ+Ψ)=λ2 δηλαδή ο 3ψήφιος ι ΟΟΧ+Ψ πρέπει να είναι ίσος με ι ι .ρ2 • Για να διαιρείται όμως με ι ι πρέπει το μεσαίο ψηφίο να είναι μηδέν (κριτήρια διαιρετότητας με l l τ. 92). Άρα X+Ψ=l l , όμως μόνο η λύση Χ=7, Ψ=4 δίνει το l l ρ2 = l l .82 •

Σταυρ ός I

Μ άντ εψε

Αφαιρούμε 2 σπίρτα από τα πλαϊνά και τα άλλα τα ανεβάζουμε μια θέση πάνω.

'

Πολλαπλαmάστε τον αριθμό των χαρτιών που πήρε τυχαία στην αρχή με ι ι , τώρα προσθέστε τα χαρτιά που έμειναν και αφαιρέστε το σύνολο των χαρτιών που έχει η τράπουλα(π.χ. 52). Αυτό είναι το άθροισμα των πόντων. Εξήγηση: αν κ είναι ο αριθμός των χαρτιών που πήρε τυχαία τότε ο αριθμός των χαρτιών που τοποθέτησε πάνω είναι ι οκ-Σ , όπου Σ το σύνολο των πόντων που πρέπει να μαντέψετε. Έτm αν μ είναι τα χαρτιά που απέμειναν μ=52-( 1 0κ-Σ+κ) ή Σ= 1 ι κ+μ-52. Ο ι στ άμ ν ε ς Μετακινούμε όλα τα λευκά μπαλάκια εκτός από ένα στην άλλη στάμνα. Έτm εάν διαλέξουμε αυτή τη στάμνα, είναι σίγουρο πως θα τραβήξουμε λευκό μπαλάκι. Αν όμως διαλέξουμε την άλλη έχουμε 49/99 mθανότητα να τραβήξει ένα λευκό. Έτm η συνολική mθανότητα να κερδίσουμε γίνεται περίπου 75%.

ΕΥΚΛΕΙΔΗΣ Β' 93

τ.Ι/78


ΑΣΚΗ Σ Ε Ι Σ

ΔΙΑΓΩΝ Ι ΣΜΟΙ ΟΛΥ Μ Π ΙΑΔΕ Σ

Ο Ουρανό�

διακοπών του καλοκαιριού, για τα όμορφα τοπία, τη

Καθημερινά παίρνουμε επιστολές και εργασίες αναγνωστών του περιοδικού. Πολλές από αυτές προκα­ λούνται από τα άρθρα που δημοσιεύουμε. Πρόσφατα πήραμε από τον κ. Κ. Μαυρομμάτη πρόεδρο της Εται­ ρείας Αστρονομίας και Διαστη­ μικής πήραμε: «( ) πρώτα να σας συπαρώ για τα δυο άρθρα που γράφτηκαν από τον Π. Χρι­ στόπουλο στα τελευταία τε6χη του & που α­ ναφέρονται .στο GPS και τα βα­ για το ότι ρυτικά κύματα. Ύστερα τα άρθρα σας αυτά μου έδωσαν αρκετά καινούργια στοιχεία, αν και ασχολούμαι συστηματικά με τα θέματα Αστρονομίας και Διαστημικής. Τέλος να σας γνωστο­ ποιήσω ότι η και Διαστήματος, της οποίας έχω την τιμή να προεδρε6ω, εκδίδει εδώ και 20 χρόνια το περιοδικό με παρόμοια θέμα­ τω> ( ) . Η αλήθεια είναι ότι γνωρίζουμε το έργο που επιτελείται χρόνια τώρα με την Εταιρεία Αστρονομίας και Διαστημικής. Διαβάζουμε στο περιοδικό σας «ΟΥ­ ΡΑΝΟΣ>>, τα εξαίρετα άρθρα για το Σύμπαν αλλά και τις καθημερινές προσπάθειες του ανθρώπου να το γνω­ ρίσει και να το . . . περπατήσει. Η προσφορά σας στη νεολαία της Ελλάδας και όχι μόνο, με την διεξαγωγή είναι με­ των γάλη. Το Υπουργείο Παιδείας δυστυχώς έχει αφαιρέσει εντελώς από το σχολείο την όσο για τη εί­ ναι στις άγνωστες λέ­ ξεις. Τα Ελληνόπουλα νομίζω ότι άρχισαν να θεωρούν τη Γη ως επί­ πεδη και ότι τον τρόπο για να πάει κανείς από τη Γη στη Σελήνη τον περιγράφει ο Ιούλιος Βερν. Δυστυχώς το δεν διδάσκει σχεδόν καθόλου την Πληροφορι­ κή, τη Διαστημική και την Αστρονομία, ενώ η επιστήμη κάνει καθημερινά άλματα. Είναι σε ένα ωρολό­ γιο σχολικό πρόγραμμα (3 5 ώρες) να Αστρονομίας σε μια Λυκειακή τάξη . . . ·

. . •

«Ευκλείδη Α Β» να σας ευχαριστήσω Εταιρεία Αστρονομίας "Ουρανός"

• • •

μαθητικών διαΎωνισμών «ΕΥΔΟΞΟΣ», Αστρονομί Διαστημιακή, ,

σχολείο ώρα

ΓΕΓΟΝΟΤΑ

. �άλασσα και τις ωραίες βραδιές στην ακρογιαλιά. Όμως

ολοι μας μικροί και μεγάλοι εκείνο που έχουμε θαυμά­ σει περισσότερο, μακριά από τα φώτα των πόλεων, εί­ ναι η Στη γαλήνη και τη σιγαλιά της νύχτας πολλές φορές έχουμε σηκώ­ σει τα μάτια στην μαγεία του έναστρου ουρανού και έ­ χουμε αφήσει τη μας Έχουμε νιώσει δέος και θαυμασμό, έχουμε μείνει για αρκετή ώρα άφωνοι και σκεπτικοί. Άφωνοι και σκεπτικοί, γιατί αισθανόμαστε αυτό που προβάλλεται ανάγλυφα μπρο­ στά μας, το άπειρο. Ποιος μπορεί λοιπόν να το αγνοήσει αυτό; Πριν 1 5 χρόνια στο τε6χος αυτό του περιοδικού υπήρχε αφιέρωμα για τα 30 χρόνια από το πρώτο βήμα του ανθρώπου στη Σελήνη. Παράλληλα όμως είχε ανα­ φορά για τα διαστημικά προγράμματα που τότε ξεκι­ νούσαν για την μελέτη των Μετεωριτών κλπ. Ήταν πρωτοπόρο γιατί 5 χρόνια μετά έφυγε το πρώτο μη ε­ πανδρωμένο ευρωπαϊκό διαστημόπλοιο για τον κομήτη που ύστερα από 10 ο Νοέμβρη 20 1 4 θα επιχειρήσει την κάθοδο του στον κομήτη. Αυτό το πείραμα γίνεται για πρώτη φορά και στόχος είναι να συλλεγούν πολύτι­ μες πληροφορίες για το διάστημα και ειδικότερα την ι­ στορία του ηλιακού μας συστήματος.

καταπληκτική εικόνα του ουρανού. φαντασία να ταξιδέψει.

«Ροζέτα» σ Γ κερασι μ tνκο ουρι ό μοφ Τ ταξίδι ετών τ

Αρχα ία Ολυμπ ία - Μουσείο Αρχ ι μήδη

Το μεράκι ενός ανθρώπου είναι αρκετό για να ζωντα­ νέψει την Εmσκε­ φτήκαμε το καλοκαίρι στην Αρχαία Ολυμπία το Μου­ σείο Αρχιμήδη. Ο κ. μας υποδέχτη­ κε και μας ξενάγησε στο χώρο. Είναι ο άνθρωπος που, όταν για πρώτη φορά φοιτητής στην Πάτρα μελέτησε για τους μοχλούς και τις σκέ­ φτηκε να αφιερώσει τη ζωή του ώστε να «ζωντανέψει» τις μηχανές και τα ρομποτικά συστήματα των Αρχαίων Ελλήνων που οι επιδόσεις τους στον τομέα αυτό ήταν σχετικά rι'\1\ιr.v•,,.,,.

τεχνολο-yία των Αρχαίων Ελλήνων. Κώστας Κοτσανάς μηχανές του Αρχιμήδη,

κρίμαμη χωράει μια

Ινδία : ομάδα Ερατοσθένη

Στην Ινδία παιδιά Γυμνασίου από όλο τον κόσμο, που είναι στην ομάδα «Ερατοσθένης», μετρούν τη σκιά του ήλιου στην αυλή του σχο­ λείου τους και με το γεωμετρι­ κό μοντέλο του Ερατοσθένη βγάζουν συμπεράσματα για το μέγεθος της Γης, για τη θέση του τόπου τους πάνω σε αυτή κ.ά. Μπράβο τους. Ο άνθρωπος πάντα θα ενδιαφέρεται για το χώρο που ζει και πάντα θα αναζητά κάτι και­ νούργιο. Με την Αστρονομία γεννήθηκαν οι εmστήμες και δεν είναι δυνατόν να την αγνοούν τα παιδιά μας. Άλλωστε όλοι μιλάμε με χαρά για τις ωραίες μέρες των

Στο έργο του αυτό έχει βοηθό και την σύζυγό του.

�ήμερα διατηρεί μια έκθεση στην Αρχαία Ολυμπία, μια

εκθεση στο Κατάκολο και μια περιοδε6ουσα σε άλλες πόλεις και χώρες. Όλα τα εκθέματα και το υποστηρικτι­ κό τους υλικό έχουν δημιουργηθεί από τον ίδιο. Πρό­ κειται για την ση Η τεχνολογία των αρχαίων Ελλήνων είχε αγγίξει τα όρια της τεχνολογίας της σημερινής επο­ χής.

πα-yκοσμίωε-yκυρότερη ς. και την πληρέστερη έκθε­


Η έκθεση ξαναζωντανεύει 300 περίπου εξαιρετι­ κές εφευρέσεις του αρχαιοελληνικού τεχνολογικού θαύματος (από το ρομπότ - υπηρέτρια του Φίλωνος μέ­ χρι τον κινηματογράφο του Ήρωνος και από το αυτόμα­ το ωρολόγιο του Κτησιβίου μέχρι τον που καλύπτουν την περίο­ δο από το 2000 π.Χ. μέχρι το τέλος του αρχαίου ελλη­ νικού κόσμου. Η έκθεση αποδεικνύει περίτρανα πόσα περισσότερα από όσα νομίζουμε ο σύγχρονος Δυτικός Τεχνολογικός Πολιτισμός Από τα εξαρτήματα της μηχανής ενός σύγχρονου αυτοκινήτου (κοχλίες, περικόχλια, οδοντωτοί τροχοί, τροχαλίες, ιμάντες, υδραυλικοί ελεγκτές, βαλβίδες, αυ­ τόματοι πλοηγοί) μέχρι έναν ικανό να παρουσιάζει αυτόματα την Ακόμη αυτοκινούμενα οχή­ ματα με αυτόματη πλοήγηση για ψυχαγωγία, λειτουργι­ κά ρομπότ, την αρχή του aτμοστροβίλου, πολύπλοκα αστρονομικά μετρητικά όργανα ακριβείας, ευφυείς μη­ χανές με κερματοδέκτη, ανυψωτικά μηχανήματα, ωρο­ λόγια (και ξυπνητήρια) ικανά να λειτουργούν αυτόματα και αδιάκοπα χωρίς ανθρώπινη παρέμβαση, κ.ά Απλά χρειάστηκε πάνω από μια χιλιετία για να ε­ πανακτήσει λησμονημένη τεχνολογία.Δεκάδες σχολεία και πανεπι­ στήμια Ελληνικά και ξένα, επισκέπτονται κάθε χρόνο το μουσείο. Τεράστιο το ενδιαφέρον των ξένων ΜΜΕ. Στο δημοφιλή γερμανόγλωσσο ταξιδιωτικό οδηγό είναι στους 10 προορι­ σμούς της Πελοποννήσου και στους δύο σημαντικότε­ ρους της Ηλείας. Το Νοέμβρη του 20 1 4 στο 31° που θα γίνει στη Βέροια ελπίζουμε να έχουμε την ευκαιρία και την χαρά να θαυμάσουμε αυτή τη μοναδική έκθεση.

καθηγητών Κ. ο οποίος χαρακτήρισε «βάλ­ σαμο τη διάκριση κάτω από αυτές τις συνθήκες που βιώνει τόσο το συνέχισε λέ­ γοντας ότι «οι καθηγητές και ο ίδιος προτά­ ξαμε τη

Βαρώτσου,

αναλογικό υπο­ διδα­έ­ χρωστά χή της στην Αρχαία Ελλά­ όχιμπνευσης και­ στεί της δα. ρας γνώση. «κινηματογράφο» πλοκή νούμενη εικόνα και ήχο. ενός μύθου με κι­ κάΙ. Εμμανουήλ,κουλτούρας του Έλληνα Μαθηματι­ εθελοντική συνεισφορά των καθηγητών η ανθρωπότητα αυτή την αξιοθαύμαστη συνεργαπολύσίακαλή ς «PELOPONNES» κορυφαίους Συνέ­ Δη­ δριο της ΕΜΕ μάκος, λογιστή των Αντικυθήρων)

Ολυμπ ι άδα Μαθηματ ι κών Ι ΜΟ

20 1 4

Στιγμιότυπο από την Ολυμπιάδα που έγινε στο Kape Town τον Ιούλιο του 20 14. Ενδεικτικό του διαγω­ νισμού, του χώρου και των συνθηκών που επικρατού­ διεξαγωγή του.

Προσπαθήσαμε να εμπνεύσουμε και το πετύχαμε». Στη συνέχεια ζήτησε από τους φοιτητές να φτάσουν τόσο ψηλά, ώστε οι καθηγητές του να γίνουν μαθητές τους. Ο ο οποίος χαρακτήρισε τα μέρος της και τόνισε ότι «αποτελούν την κατάλληλη γλώσσα για την περιγραφή, μελέτη και τη βαθύτερη κατανόηση της φύσης». Στη συνέχεια αναφέρθηκε στη φιλοσοφική διάσταση των Μαθηματικών αλλά και στην της σχολής στην εκγύμναση των φοι­ τητών για τους διαγωνισμούς και τέλος τόνισε την

που υπάρχει με­ ταξύ Μαθηματι­ κού Αθήνας και Μαθηματικής Εταιρείας. Στη συνέχεια ο πρόεδρος της Ε.Μ.Ε., Γ. αφού συvεχάρη τους βραβευόμενους, αναφέρ­ θηκε στο ιστορικό του Μαθηματικού Τμήματος Αθή­ νας του ΕΚΠΑ η οποία χρονικά προηγήθηκε και αυτής ακόμα της Ακαδημίας Αθηνών. Μετά αναφέρθηκε στην ιστορία και το ρόλο της Ένωσης Μαθηματικών Νοτιοα­ νατολικής Ευρώπης και το διαγωνισμό SEEMOUS. Φυσικά δεν παρέλειψε να αναφερθεί στο σύνολο των διαγωνισμών της ΕΜΕ Κλείνοντας ζήτησε από την πολιτεία να δώσει βαρύτητα στη χρησι­ μοποίηση όπως είπε, που θα βοη­ θήσουν στην

στο διαγωνισμό του Μκάνοντας ικρού Ευκλείιδιαίδτη.ερη αναφορά «τωνανάπτυξ νέων ημυαλών», της χώρας. Euromath

κα ι

Le - Math 20 1 4

Έγινε τον Ιούλιο του 20 1 4 σε κεντρικό ξενοδοχείο της Αθήνας η παρουσίαση δύο εργαλείων Μαθηματι­ κών, από την ευρωπαϊκή ομάδα μελέτη ς της Euromath, που

Seemous 20 1 4

Τετάρτη Ιουλίου

Την 9 2014 στο Αμφιθέατρο του Πα­ νεπιστημίου Αθηνών (κεντρικό κτίριο) έγινε η τελετή βράβευσης των φοιτητών που διακρίθηκαν στο Seemous 20 14. Βραβεύτηκαν οι φοιτητές των ΕΚΠΑ και ΕΜΠ: Μπόλκας Ελευθέριος, Ψαρομιλίγκος Κων­ σταντίνος, Ζαρίφης Νικόλαος, Δοσούλας Γεώργιος, Οι­ κονόμου Ανάργυρος. Ενδεικτικές οι προσφωνήσεις των

θα στην βοηθήσουν επι μ όρφωση καιδευτιτηνκώνκατάρτιση των εκπαι­

που διδάσκουν σε μα­ θητές (9- 1 8 ετών). Α. ΜΑTHeatre [Μαθηματικά και Μαθη ματικό θέατρο] Β . MATHFactor [Μαθηματικά και επικοινωνία] Πολλές πληροφορίες θα βρείτε για αυτά τα εργαλεία και για το Euromath 20 1 4 που έγινε τον 2014 στη Λευκωσία, στην ιστοσελίδα: www.Ie-matb.eu

Απρίλιο του


αλησμόνητος καθηγητής Πρόεδρος της Ελληνικής Μαθη ματικής Εταιρείας Στράτος Γαλανής Ο

Στις 5 Αυγούστου 2014 έγινε η νεκρώσιμη ακολουθία για τον Καθηγητή του Μετσόβιου Πολυτεχνείου Στράτο Γαλανή . Τον aποχαιρέτισε όλοι η μαθηματική κοινότητα και τα μέλη του Δ.Σ. της Ε.Μ.Ε. Εκφωνήθηκε ο παρακάτω επικήδειος λόγος: αλησμόνητος καθηγητής μας Στράτος Γαλανής Γεννήθηκε στο Βουλγαρέλι της Άρτας πριν από 79 χρόνια. Πτυχιούχος Μαθηματικών του Πανεπιστημίου Αθηνών και Διδάκτωρ του Πανεπιστημίου του Cambridge. Διετέλεσε καθηγητής από το 1 974 στο ΕΜΠ, πρόεδρος του Γενικού Τμήματος του ΕΜΠ από το 1 982- 1 987, Αντιπρύτανης στο ΕΜΠ, καθώς και Πρόεδρος της ΕΜΕ από το 1 983- 1 98 5 . Κατά τη διετία της προεδρίας του επεδίωξε και πραγματοποίησε την επέκταση και αναβάθμιση των περιοδικών και των επιστημονικών εκδόσεων της ΕΜΕ, τη μετάφραση αξιόλογων ξένων πανεπιστημιακών συγγραμμάτων (που και σήμερα ακόμη διδάσκονται σε πολλά πανεπιστήμια), τη θεμελίωση και ίδρυση της Βαλκανικής Μαθηματικής Ολυμπιάδας, και τη συγκρότηση της νέας σειράς των Πανελληνίων Συνεδρίων Μαθηματικής Παιδείας (το Πρώτο έγινε το 1 984 το 20 1 4 το 3 1 ο ). Δύο ήταν οι άξονες στα πλαίσια των οποίων αγωνίστηκε με όλες του τις δυνάμεις. Ο πρώτος ήταν η προσφορά υψηλής μαθηματικής παιδείας. Πίστευε ότι η Μαθηματική παιδεία δεν μεταδίδει στατικές γνώσεις και πέρα από τη μάθηση επιδιώκει την ολοκληρωμένη μόρφωση του ατόμου. Θεωρούσε ότι η Μαθηματική παιδεία αποτελεί μια ακατάλυτη ανθρωποπλαστική δύναμη που τείνει στην πνευματική ανανέωση του ανθρώπου. Πίστευε ότι είναι aσύγκριτη η ομορφιά της λογικής που περικλείει κάθε μαθηματική θεωρία και συνεπώς αναντικατάστατη για τη διαμόρφωση μιας ολοκληρωμένης προσωπικότητας. Για την υλοποίηση του πρώτου άξονα ανέπτυξε τις ακόλουθες δραστηριότητες: συστη ματική και εντατική προετοιμασία των Ελλήνων μαθητών για τις Ολυμπιάδες, διοργάνωση σεμιναρίων κάθε χρήσιμου κλάδου, ποιοτικό εμπλουτισμό των περιοδικών εκδόσεων της ΕΜΕ και έκδοση βοηθημάτων για κάθε εκπαιδευτική βαθμίδα. Όλες οι παραπάνω δραστηριότητες αποτέλεσαν οδηγό δράσης για τις επόμενες δεκαετίες της ΕΜΕ. Ο δεύτερος άξονας ήταν ο aνυποχώρητος και συνεχής αγώνας για ειρήνη - αφοπλισμό και ατομικά δικαιώματα ανά την υφήλιο. Πίστευε ότι ο πνευματικός άνθρωπος δεν έχει σαν μοναδικό σκοπό το ατομικό συμφέρον και δεν πρέπει να υποτάσσει σε αυτό τα πάντα. Είχε πίστη στην κοινωνική αποστολή του ανθρώπου που αποτελεί την ηθική πηγή του καθήκοντος, αφού μέτρο της αξίας ενός ατόμου πρέπει να είναι η κοινωνική προσφορά και καταξίωσή του. Στο πρώτο συνέδριο της ΕΜΕ το 1 984 ο αλησμόνητος φίλος Στράτος είχε αναφέρει τα ακόλουθα λόγια του Γέρου του Μοριά « Η προκοπή σας και μάθησή σας να μη γίνει σκεπάρνι μόνο για το άτομό σας αλλά να κοιτάζει το καλό της κοινότητας γιατί μέσα στο καλό αυτό βρίσκεται και το δικό σας». Επίκαιρο μήνυμα ακόμα και στην εποχή μας. Ο φίλος και συνάδελφος Στράτος υπήρξε συνεπής, αξιοπρεπής άνθρωπος του μέτρου, της συνεννόησης, της συναίνεσης. Μια ισχυρή προσωπικότητα που στηριζόταν στη λογική της δικαιότητας, της κοινωνικής προσφοράς, της ηπειρώτικης επιμονής και όλα αυτά διαχέονταν από ένα ανεξάντλητο χιούμορ. Αποτέλεσε για μας τους νεώτερους ζωντανό παράδειγμα προτύπου, ακεραιότητας και εθελοντικής δράσης Το Δ.Σ. της ΕΜΕ εκφράζει τα θερμά συλλυπητήριά του στην αγαπημένη του γυναίκα Άννα και στο μοναχογιό του και συνάδελφο Γιώργο τους οποίους υπεραγαπούσε και οι οποίοι στάθηκαν δίπλα του με αγάπη και αφοσίωση σε όλη τη διάρκεια της ζωής του. Φίλε Στράτο καλό ταξίδι στην αιώνια ζωή " . ''Ο

Τον επικήδειο λόγο εκφώνησε ο Πρόεδρος της Ελληνικής Μαθηματικής Εταιρείας Καθηγητής κ. Γεώργιος Δημάκος



Issuu converts static files into: digital portfolios, online yearbooks, online catalogs, digital photo albums and more. Sign up and create your flipbook.